50 Sitting Arrangements Sets

You might also like

Download as pdf or txt
Download as pdf or txt
You are on page 1of 348

50 important seating arrangment sets with

explanations for SBI PO 2019

Directions: Study the following information carefully and answer the


questions given beside.

Set-1

Nine persons Anu, Bablu, Cheenu, Dona, Esha, Faria, Gaurav, Harish and
Ishita are sitting in a row and all are facing north. It is known that Cheenu
sits exactly in the middle and there is no person to the right of Ishita. Dona
is fourth to the right of Faria. Gaurav and Harish are sitting next to each
other. Esha is the neighbor of Dona but not of Cheenu.

Harish doesn't sit at any extreme corner.Dona is not sitting adjacent to


either Cheenu or Ishita.Anu is second to the right of Harish.

1. Who is sitting to the immediate left of Cheenu?


A. Faria B. Anu C. Harish
D. Dona E. None of these
2. Who is sitting between Bablu and Esha?
A. Dona B. Faria C. Gaurav
D. Cheenu E. None of these
3. Four of the following five are alike in a certain way and thus forms a
group. Which of the following does not belong to that group?
A. Gaurav and Harish B. Cheenu and Bablu C. Faria and Gaurav
D. Dona and Esha E. Esha and Ishita
4. Who is sitting third to the right of Harish?
A. Bablu B. Dona C. Faria
D. Cheenu E. None of these
5. Who is sitting at the left most seat of the row?

A. Faria B. Bablu C. Gaurav

D. Dona E. None of these


Set-2

L, M, N, O, P, Q, R and S are sitting around a circular table facing the centre


but not necessarily in the same order.

They all are having discussion on an important topic.

N is an immediate neighbor of both L and R, who is an immediate


neighbour of P. P sits second to the left of N. Only two persons sit between
P and S. M is an immediate neighbor of O. Only one person sits between M
and Q.

6. What is the position of O with respect to S in the above arrangement?

A. Third to the left B. Immediate left C. Second to the right

D. Immediate right E. None of these

7. Four of the following five are alike in a certain way and thus form a
group. Which is the one that does not belong to that group?

A. MS B. QL C. NM

D. RP E. OM

8. Who among the following sits second to the left of the one who is on
the immediate right of O?

A. Q B. M C. O

D. S E. None of these

9. Which of the following persons sits between M and Q?

A. O B. L C. N

D. S E. None of these

10. Who among the following sits third to right of the one who sits
opposite of N?
A. L B. S C. R

D. O E. None of these
Set-3

There are eight persons - Mita, Niya, Om, Pari, Qiya, Rati, Shri and Tiya. All
of them are sitting in a straight row and are facing in the South direction
but not necessarily in the same order. There are only two persons between
Rati and Pari. The number of persons between Qiya and Mita is same as the
number of persons between Pari and Shri. Om is not a neighbor of Shri who
is immediate left of Niya. Qiya is second to the left of Rati who is fourth
from the right corner. Mita sits at any corner of the row. Tiya sits
immediate right of Qiya.

11. How many persons are seated to the left of Niya?

A. One B. Three C. Four


D. Six E. None of these
12. Four of the following five are alike in some way and thus form a
group. Which of the following does not belong to the group?

A. Om B. Rati C. Shri
D. Qiya E. Pari
13. Who among the following is/are immediate neighbour(s) of Qiya?

A. Tiya B. Om C. Both A and B


D. Either A or B E. Shri
14. Who among the following sits second to the left of the person who
sits third from the right end?

A. Mita B. Tiya C. Pari


D. Niya E. None of these
15. Who among the following sits third to the left of Rati?

A. Om B. Pari C. Niya
D. Mita E. None of these
Set-4

Eight persons – Amar, Ansh, Arun, Amit, Ashu, Anup, Arif and Anil, are
sitting around a circular table facing the centre but not necessarily in the
same order. Arif is to the immediate left of Ashu. Anup is to the immediate
left of Arun. Amar is the neighbour of Ansh. Amit is between Anup and
Ansh. Arif is not the neighbour of Ansh or Anil.

16. Who among the following is sitting third to right of Amar?


A. Arif B. Anup C. Arun
D. Ansh E. None of these
17. How many persons were sitting between Amit and Anil, when counted
from the left of Anil?
A. None B. One C. Two
D. Three E. More than three
18. What is the position of Arif with respect to Amit?
A. Third to the right B. Second to the left C. Fourth to the right
D. Fourth to the left E. None of these
19. Which of the following person sits opposite to the one who sits third
to the left of Anil?
A. Anup B. Arif C. Amar
D. Arun E. None of these
20. Which of the following pairs of persons is sitting adjecent of Arif?
A. Anil, Ashu B. Amit, Arun C. Anup, Ashu
D. Arun, Ashu E. Arun, Anil
Set-5

Eight persons namely J to Q are sitting on the square table but not
necessarily in the same order. Four persons are sitting at the corner of the
table and facing away from the center. Four persons are sitting at the
middle of the table and facing towards the center. The persons are sitting
at the corner of the table likes different fruits among Banana, Apple,
Grapes and Mango. The persons are sitting at middle of the table likes
different vegetables among Cucumber, Brinjal, Beans and Cabbage. All the
above information is not necessarily in the same order.

K sits third to the right of O. One person sits between K and Q. One who
likes Beans sits third to the left of Q. One who sits second to the right of the
one who likes Beans is facing N. One who likes Cabbage sits second to the
right of the one who likes Cucumber. The person J is the immediate
neighbor of the one who like Brinjal and O. There are as many persons sit
between P and M as between P and L. One who likes Mango sits third to
the right of M. One who likes Beans and one who likes Mango are not
immediate neighbors. J likes Banana. L and one who like Apple are not
immediate neighbors.

21. Who among the following likes Grapes?

A. M B. Q C. K

D. N E. None of these

22. Who among the following person sit opposite to O?

A. The one who like Mango

B. The one who like Grapes

C. The one who like Cucumber

D. The one who like Cabbage

E. None of these
23. Which among the following combination is true?

A. K-Mango B. M-Apple C. N-Banana

D. L-Grapes E. None is true

24. What is the position of N with respect to the one who likes Banana?
A. Third to the right B. Second to the left C. Second to the right

D. Immediate left E. None of these

25. Four of the following five are alike in a certain way and hence form a
group. Which of the following does not belong to the group?

A. K B. J C. Q

D. M E. P
Set-6

Eight students Aadvik, Tanish, Jiral, Vaibhav, Yashith, Bhavin, Rithwik and
Kiansh sit in a square table. Four persons sit in each corners of the table
and four persons sit in each sides of the table but not necessarily in the
same order. All are facing outside. Aadvik is immediate neighbor of Kiansh.
Bhavin does not sit third to the left of Tanish who doesn’t sit at the corner.
Bhavin sits fourth to the right of Yashith. There are three persons sit
between Tanish and Jiral. Jiral sits third to the left of Vaibhav. One person
sits between Jiral and Kiansh. Rithwik and Kiansh are not immediate
neighbors.

26. Four of the following five are alike in a certain way and hence form a
group. Which of the following does not belong to the group?

A. Aadvik B. Vaibhav C. Bhavin

D. Yashith E. Rithwik

27. Tanish is related to Kiansh in certain way, Bhavin is related to Aadvik


then in the same way Jiral is related to which among the following?

A. Rithwik B. Tanish C. Vaibhav

D. Bhavin E. Aadvik

28. Who among the following sits fifth to the right of Kiansh and
immediate right of Rithwik?

A. Aadvik and Vaibhav B. Only Bhavin C. Vaibhav and Yashith

D. Tanish and Yashith E. Only Vaibhav

29. Which of the following person sits between Jiral and Kiansh?
A. Tanish B. Rithwik C. Aadvik

D. None E. None of these


30. Which of the following person sits opposite to the one who sits
second to the right of Rithwik.

A. Bhavin B. Jiral C. Tanish

D. Yashith E. None of these


Set-7

Some persons are sitting in a row facing north. Hina sits second to right of
Mera. Desh sits fifth to left of Fany. Pinu is second to right of Rani. Hina and
Asha have three persons between them. Desh and Rani have two persons
between them. No one sits between Asha and Fany. Number of persons
towards right of Asha is one less than the number of persons sitting
between Pinu and Asha. Number of persons towards left of Rani is equal to
the number of persons sitting between Fany and Hina.
31. How many persons were sitting in the row?
A. 17 B. 20 C. 16
D. 19 E. 22
32. How many persons were sitting between Pinu and Asha?
A. Three B. Five C. Seven
D. Eight E. Six
33. What is the position of Fany with respect to Hina?
A. Third to the left B. Fourth to the left C. Fifth to the left
D. Sixth to the left E. Can’t be determined
34. What is the total number of persons that were sitting on the right of
Hina and on the left of Rani?
A. 5 B. 8 C. 10
D. 11 E. Can’t be determined
35. If Riya is sitting in the middle of the row then how many persons were
sitting between Riya and Rani?
A. 5 B. 7 C. 6
D. 8 E. 4
Set-8

Eight persons – Doly, Lali, Mona, Joya, Pinu, Usha, Chaya and Tara, are
sitting in a straight line but not necessarily in the same order such that
some of these persons are facing south while some are facing north.

Doly faces north. Mona sits third to the left of Usha. Pinu sits third to the
right of Mona. Joya sits to the immediate right of Pinu. Joya does not sit at
any of the extreme ends of the line. Only one person sits between Chaya
and Lali. Pinu sits second to the left of Chaya. Tara sits second to the right
of Joya. Both the immediate neighbours of Mona face the opposite
direction. Tara faces the same direction as Joya and Doly faces the same
direction as Chaya. Usha sits at one of the extreme ends of the line.

36. Who among the following is sitting second to the right of


Mona?
A. Doly B. Usha C. Pinu

D. Joya E. None of these

37. Four of the following five are alike in some way and hence
form a group. Which of the following is the one that does not
belong to the group?
A. Tara B. Usha C. Mona

D. Lali E. Joya

38. How many person(s) sit between Joya and Doly?


A. Three B. Two C. One

D. None E. More than three

39. How many persons are facing north direction?


A. Two B. Three C. Four
D. Five E. Six

40. Who among the following is sitting third to the left of Lali?
A. Usha B. Joya C. Chaya

D. No one E. None of these


Set-9

Twelve persons are sitting in two parallel rows in such a way that the
persons sitting in Row-1 faces the persons sitting in the Row-2. Six persons
A to F are sitting in the Row-1 and facing north direction. Six persons P to U
are sitting in the Row-2 and facing south direction. All the above
information is not necessarily in the same order. E sits second to the right
of F. Two persons sit between F and B, who is facing R. Q and R are
immediate neighbors. U sits second to the left of Q. The person who sits
second to the right of P is facing C. More than one person sits between F
and D. T does not sit at the extreme end of the line.

41. Who among the following person sits third to the right of R?

A. S B. P C. Q D. U E. None of these

42. How many persons sit between C and A?

A. One B. None C. Two D. Three E. None of the above

43. If only Row-1 persons arranged in alphabetical order from left to right,
then who among the following faces Q?

A. E B. C C. D D. F E. A

44. If S is related to P, Q is related to U, then in the same manner E is


related to ________ .

A. B B. C C. F D. D E. A

45. Who among the following is sitting diagonally opposite to C?

A. P B. Q C. R D. T E. U
Set-10

Eight persons – Mona, Moti, Meera, Mali, Mitra, Mansi, Megha and Mrig,
were sitting in a circle facing towards the center. Each of them was born in
a different city — Shimla, Manali, Agra, Nanital, Pune, Goa, Mumbai and
Delhi, but not necessarily in the same order.

Two persons were sitting between the one, who was born in Nanital and
Mitra. Mona was born in Shimla and sits opposite to Mitra. The one, who
was born in Pune, sits opposite to Mali. Mrig was born in Mumbai and sits
second to the right of the one, who was born in Nanital. Meera was born in
Pune and was an immediate neighbor of the one, who was born in Goa.
Megha sits third right to Moti. Mali was born in Nanital. The one, who was
born in Delhi, sits adjacent to the one, who was born in Nanital. Mitra was
born in Manali and Megha was born in Delhi.

46. Mansi was born in which of the following cities?

A. Agra B. Mumbai C. Delhi

D. Goa E. None of these

47. Who among the following was sitting third to the right of Megha?

A. Mitra B. Meera C. Moti

D. Mona E. None of these

48. Who among the following was born in Agra?

A. Moti B. Mansi C. Megha

D. Mrig E. None of these

49. Who among the following was sitting in the front of Moti?
A. The one who was born in Manali

B. Meera
C. The one who was born in Mumbai

D. Mitra

E. Both option A and D

50. What is the position of Meera with respect to the one who was born
in Shimla?

A. Second to the right B. Third to the right C. Fourth to the left

D. Second to the left E. None of these


Set-11

Eight persons – Ankit, Kavit, Manit, Vinit, Charit, Sumit, Ganit, and Ronit are
sitting in a straight line (but not necessarily in the same order) such that
three of them are facing south and rest of them are facing north. Each of
them works in a different bank – SBI, UCO, PNB, BOB, SEBI, RBI, BOI, and
NHB (but not necessarily in same order).

Four persons sit between Ronit and Charit, who works in RBI. Sumit sits
right of Manit, who works in BOI. Ganit and Charit face opposite direction
to each other. The one, who is sitting on the immediate right of Vinit, works
in NHB. Kavit sits third from one of the extreme end of the row. Ronit sits
third to the right of Manit. The one who works in PNB sits immediate right
of the one who works in UCO. Manit doesn’t face north direction. Ganit is
an immediate neighbor of Manit and Charit both. Vinit sits second to the
right of Ganit. Manit sits on the immediate left of Kavit. The one who works
in SBI sits at an extreme end. The one who works in BOB sits between
Charit and Ankit. Sumit does not work in PNB.

51. Kavit works in which of the following banks?

A. UCO B. PNB C. SEBI

D. NHB E. None of these

52. Which of the following combinations is correct?

A. Sumit – BOI – South B. Kavit – UCO – South C. Vinit – BOB – North

D. Ronit – NHB – North E. Ankit – SBI – South

53. How many person(s) sit between Charit and Sumit?

A. None B. One C. Two

D. Three E. More than three


54. What is the position of the one who works in BOB with respect to the
one who works in PNB?

A. Third to the right B. Fourth to the left C. Fifth to the right

D. Fourth to the right E. None of these

55. Who among the following works in SEBI?

A. Ankit B. Kavit C. Vinit

D. Sumit E. None of these


Set-12

Eight persons – Shirish, Suresh, Sajal, Sashi, Sanjay, Sunder, Srajit and
Shankar (not necessarily in the same order) are sitting around a circular
table in a way such that four of them are facing towards the centre and the
others are facing away from the centre. Each of these persons have a
different surname – Gaur, Gail, Gopal, Gandhi, Girish, Guman, Garv and
Goyal, but not necessarily in the same order.

Sajal’s surname is Gaur and is facing towards the centre.

The immediate neighbours of Sajal faces away from the centre and their
surnames are Gail and Gopal.

Shankar faces away from the centre and the immediate neighbours of
Shankar faces a direction opposite to each other.

Srajit’s surname is Garv, is facing towards the centre and sits third to the
right of Sashi, whose surname is Gail.

The one whose surname is Girish is not neighbouring the one whose
surname is Gail or Srajit.
Shirish is sitting opposite to Sanjay and neither of them has a surname
Gopal but surname of one of them is Girish.
Both Shirish and Sanjay are facing in the same direction (for example, if one
is facing outside then other is also facing outside).
Shankar is not neighbouring either the one whose surname is Garv or the
one who is neighbouring the one whose surname is Guman but is sitting
opposite to the one whose surname is Gandhi, who is neighbouring Shirish.

Sunder’s surname is not Goyal and is facing the centre.

56. Who among the following has surname Goyal?

A. Suresh B. Shirish C. Sanjay

D. Shankar E. Sunder
57. Who among the following sits second to the right of Shankar?

A. Suresh B. Shirish C. Sanjay

D. Sashi E. Sajal

58. Who among the following is sitting third to the right of the one, who is
sitting second to the left of the one, whose surname is Gandhi?

A. Sunder B. Shirish C. sanjay

D. Shankar E. Sashi

59. Four of the following five are alike in some way and hence form a
group. Which of the following is the one that does not belong to the
group?

A. Sunder Gopal B. Shirish Girish C. Sashi Gaur

D. Sajal Garv E. Suresh Gail

60. Who among the following is the neighbour of Suresh?

I. Srajit II. Sanjay III. Shirish

A. Only I B. Only II C. Only III

D. Both I and II E. Both I and III


Set-13

Seven persons – Naina, Shalu, Amar, Rohan, Doshi, Brij and Vinod, are
sitting on a bench in a church, facing south. Each of them has a different
profession – Manager, Engineer, Boxer, Politician, Singer, Painter and Actor.

Amar sits second to the right of Vinod. Brij is not a Boxer. There is one
person sitting between Amar and Rohan. Naina is an immediate neighbour
of Doshi, who sits at an extreme end. The one who is an Engineer sits third
to the left of Shalu, who is a Painter. Vinod sits third from left end. The one
who is a Singer sits at an extreme end and immediate neighbor of Shalu.
The one who is an Actor sits second to the left of Rohan. Naina is neither
Manager nor Boxer.

61. Who among the following is a Politician?

A. Doshi B. Naina C. Brij

D. Amar E. None of these

62. What is the profession of the one who sits in the middle of the row?

A. Boxer B. Painter C. Engineer

D. Manager E. None of these

63. Who among the following sits third to the left of the one who is an
Actor?

A. Naina B. No one (As the one who is an Actor sits second


from the left end of the row)

C. Vinod D. Amar E. Brij

64. How many person(s) sits between Shalu and Naina?

A. None B. One C. Two

D. Three E. More than three


65. Which of the following combinations is/are correct?

A. Shalu – Singer B. Doshi – Manager C. Amar – Actor

D. All are correct E. None is correct


Set-14

Eight persons namely Hetav, Herin, Hevin, Himan, Hiran, Hiyan, Henik and
Herat are sitting in square table in such a way that four of them sit at the
corner of the table and four of them sit at the middle of the table. The
persons sitting at the corner of the table are facing away from the centre
and the persons sitting at the middle of the table are facing towards the
centre. Each of them likes different chocolates among Milky Bar, Munch,
Perk, Kitkat, 5 Star, Dairy Milk, Barone and Bournville. All the above
information is not necessarily in the same order.

Herin and Hetav are facing same direction. Herat sits third to the right of
Hetav. The one who likes Munch and the one who likes Milky Bar are facing
each other. Hiyan likes Perk and sits second to the right of the one, who
likes Barone. Herat and Himan sit opposite to each other but does not face
each other. Henik sits to the immediate right of the one who likes Milky Bar
and to the immediate left of the one who like 5 star chocolates. The one
who likes Bournville sits third to the left of the one who likes Kitkat. Hiran
sits third to the right of the one who likes Dairy Milk. Herin is an immediate
neighbor of the one who likes Barone. One who sits to the immediate right
of Herat does not like Bournville.

66. Who sits second to the right of the one who likes Kitkat?

A. Herin B. Henik

C. The one who like 5 star

D. The one who like Milky Bar

E. None of these

67. Who among the following likes 5 star chocolate?

A. Hevin B. Hiran C. Herin

D. Hiyan E. None of these


68. Who among the following likes Bournville chocolate?

A. Himan B. Hiran C. Herin

D. Hiyan E. None of these

69. Who among the following persons facing each other?

A. Hetav and the one who like Dairy Milk

B. Herin and the one who like Bournville

C. Hiran and the one who like 5 stars

D. Hevin and the one who like Barone

E. None of these

70. Who sits third to the right of Henik?

A. The one who likes Bournville

B. The one who likes Kitkat

C. The one who likes Munch

D. The one who likes Perk

E. None of these
Set-15

Twelve people O, P, Q, R, S, T, U, V, W, X, Y and Z are seated on parallel


rows. Such a way that O, P, S, U, V, Z are seated on row 1 and other persons
are seated on row 2 but not necessarily in the same order. Some people are
facing north and some people facing south in both the rows. O sits second
from the left of S who is not facing north. P sits immediate right of S. P sits
straight to T facing opposite direction to each other. V sits second to the
right of P. O and R faces each other. Y sits second to the left of R. Two
persons sit between Q and X. W and U are seated at corners and facing
opposite direction to each other. Neither Q nor X is sit at the extreme ends
of the row. Z and U are facing same direction as Q and X. V is facing same
direction as R. Q is not a neighbor of R. Not more than five persons facing
south.

71. Who among the following persons facing south in Row-1?

A. S and O B. V and S C. S and P

D. S and U E. Cannot be determined

72. In which direction neighbor of S faces?

A. One person faces north and other persons faces south

B. Both the persons faces North

C. Both the person faces South

D. Immediate left faces north and immediate right faces south

E. Cannot be determined

73. Find the following person’s position in the rows respectively?

I. Who sits immediate right of O?

II. Who sits third to the right of Q?


A. Y and V B. V and X C. V and Y

D. Z and R E. Cannot be determined

74. Find the odd one out among the following.

A. U B. Z C. Y

D. X E. R

75. If U is related to P and S is related to Z in same way Y is related with:

A. U B. Z C. X

D. Q E. R
Set-16

Eight persons – Navya, Farukh, Ankur, Mangal, Ravan, Danav, Simar, and
Gopal are seated around a circular table such that only three of them are
facing away from the centre and the rest are facing towards the centre (not
necessarily in the same order).

Navya sits second to the left of Ankur. Farukh is not an immediate


neighbour of both Ankur and Navya. One of the immediate neighbours of
Ankur faces opposite direction of Ankur. Gopal sits third to the left of
Navya. Danav sits second to the left of Gopal. Mangal is not an immediate
neighbour of Farukh. Simar sits third to the left of Mangal. Ravan and
Farukh face the same direction as Mangal faces. Farukh doesn’t sit opposite
to both Ankur and Navya.

76. Who among the following sits second to the right of Danav?

A. Navya B. Simar C. Mangal

D. Gopal E. Gopal

77. Who among the sits in front of Ravan?

A. Farukh B. Mangal C. Navya

D. Ankur E. None of these

78. If the all persons are made to according to their names appear in
dictionary in clockwise direction starting from Ankur then position of how
many person(s) will remain unchanged including Ankur?
A. One B. Two C. Three

D. Four E. Five

79. Who among the following sits third to the left of Simar?

A. Mangal B. Danav C. Navya

D. Ravan E. None of these


80. How many person(s) sit between Ravan and Navya when counted
from the right of Navya?

A. One B. Two C. Three

D. Four E. Five
Set-17

A row of persons facing north direction consist of equal number of boys


and girls such that boys are sitting at the extreme ends. All the boys except
one sit at even positions and all the girls except one sit at odd positions.
Extreme left end of the row is positioned as 1 and the place immediate next
to it as 2 and so on.

 There is no one to the left of A who is fourth to the left of J who is


immediate left of W.

 P sits at an extreme end and is third to the right of F who is a girl.

 Only three boys are there to the right of U, who is a girl.

 T and O are immediate neighbours and T is to the immediate left of P


with T being a girl.

 Only two persons are between W and F.

 H who is not a boy is sitting adjacent to A and X.

 Z is fourth to the right of B who is on immediate right of X.

81. Which of the following girls sit at an even numbered position?

A. X B. J C. H

D. U E. None of these

82. How many girls sit between X and O?

A. Four B. Three C. Six

D. Five E. None of these

83. Which of the following is an immediate neighbour of B and W?

A. U B. X C. F

D. J E. None of these
84. How many boys and girls are there in the row?

A. 6 boys, 6 girls B. 7 boys, 7 girls C. 5 boys, 5 girls

D. 4 boys, 4 girls E. Either A or C

85. Four of the following five are alike in a certain way and thus form a
group. Which of the following does not belong to the group?

A. B B. W C. O

D. Z E. J
Set-18

Eight persons – Heer, Rani, Nora, Anup, Doha, Mrig, Gaur and Shera are
sitting around a square table such that four persons sits on the fours corner
the table facing towards the center and the rest of the four persons sits on
the middle of the sides facing away from the centre.

Gaur neither sits at corner nor opposite to Shera. Nora sits third to the left
of Rani. Only two persons sit between Mrig and Shera, who sits immediate
left of Nora. Anup sits second to the left of Heer, who sits third left of Gaur.

86. Who among the following sits in front of Anup?

A. Nora B. Gaur C. Shera

D. Rani E. None of these

87. How many person(s) sit between Rani and Shera, when counted from
the right of Shera?

A. One B. Two C. Three

D. None E. More than three

88. If the names of all the persons are arranged according the order in
which they will appear in dictionary in anticlockwise order starting from
Anup, then position of how many person(s) will be remain unchanged
excluding Anup?

A. One B. Two C. Three

D. None E. More than three

89. Who among the following sits third to the left of Doha?

A. Shera B. Rani C. Heer


D. Anup E. None of these
90. Who among the following sits second to the right of Mrig?

A. Nora B. Gaur C. Doha

D. Anup E. None of these


Set-19

Eight persons are sitting on two parallel benches namely Bench 1 and
Bench 2, each four on a bench. The persons who are sitting on bench 1
facing north and those on bench 2 are facing south such that each person
of both the benches faces each other.

Gayu faces Hina, who does not sit at any of the extreme ends. Ela and Bani
neither sits in the same row nor faces each other. Hina sits second to the
right of Dia. Fiza sits in the same row in which Ela sits. Bani faces the person
who is second to the right of Ahana. Caira sits on bench 1

91. Who among the following faces Ela?

A. Bani B. Hina C. Gayu

D. Dia E. None of these

92. Four of the following five are alike in a certain way and thus form a
group. Which of the following does not belong to the group?

A. Hina B. Gayu C. Fiza

D. Ela E. Ahana

93. Which of the following pairs consist of persons who sit at the extreme
ends?

A. Ela - Hina B. Caira - Gayu C. Ela – Caira

D. Ahana - Bani E. Bani - Caira

94. What is the position of Fiza with respect to Ahana?

A. Second to the left B. Second to the right C. Immediate left

D. Immediate right E. None of these


95. In which of the following pairs, persons do not face each other?

A. Bani - Fiza B. Caira - Ahana C. Dia – Ela

D. Hina - Gayu E. Hina – Ahana


Set-20

Ten People- A, B, C, D, E, P, Q, R, S and T are sitting in two parallel rows but


not necessarily in same order. Four of them like white colour , four of them
like black colour and two like red colour. No two people likes same colour
sit opposite or next to each other. Row 1 consist of- A, B, C, D, E but not
necessarily in same order and Row 2 consist of- P, Q, R, S and T but not
necessarily in same order. People sitting in both rows face each other.

T is an immediate neighbour of R. People who like Red colour do not sit in


the same row. Two people sit between Q and T. E sits third to the right of B.
A does not like black colour. Person who likes red colour is not an
immediate neighbour of D. Person facing B and sitting next to him, Both like
Black colour. D does not face P. People liking Red colour sit at extreme ends
of the rows. R sits at one of the extreme ends. People who like Red colour
do not face each other. Q does not like red colour. S and A like same colour.
B likes red colour.

96. Find the Odd one out?

A. B B. Q C. D

D. C E. R

97. Which of the following pair face the one who likes red?

A. C, S B. A, P C. E, T

D. D, R E. Q, D

98. A is related to C and E in some way , in the same way S is related to P


and Q then E is related to whom following the same pattern?

A. C, B B. A, D C. E, R

D. D, C E. Q, D
99. Which of the following is not false?

A. S-Black B. D-Red C. Q-White

D. All are false E. All are not false

100. Find the wrong pair?

A. C, S B. E, T C. A, R

D. B, Q E. None of these
Set-21

Six friends Prakash, Qadir, Rithesh, Surya, Tarini and Umesh are sitting
around the square table. All of them belong to different profession among
Engineer, Doctor, Scientist, Banker, Lawyer and Teacher but not necessarily
in the same order. Some of them are facing inside and some of them are
facing outside. Four of them are sitting at the corner of the table and two
are sitting on the opposite edges of the table. Not more than two
consecutive sitting persons are facing the same direction.

Tarini, whose profession is Scientist, sits second to the left of Qadir and to
the left person whose profession is Lawyer. Qadir sits at corner and facing
away from the centre of the table. The person sitting to the immediate
right of Qadir is sitting at the corner of the table. Umesh, who is not the
immediate neighbor of Qadir, is facing the person, whose profession is
Doctor. Surya, whose profession is Teacher, is an immediate neighbor of
the person whose, profession is Lawyer. Surya is not the immediate
neighbor of Tarini. Rithesh is not sitting at the edge of the table. The
immediate neighbors of Prakash are facing different directions. The one
whose profession is Scientist is not sitting to the immediate right of the
person whose profession is Banker. The person whose profession is Banker
does not sit at the edge of the table. The person sitting at the edges are
facing towards each other.

101. Who is sitting to the immediate left of Prakash?

A. The one who is Banker

B. The one who is Scientist

C. Surya D. Rithesh E. None of these

102. What is the profession of Rithesh?

A. Banker B. Lawyer C. Engineer

D. Doctor E. None of these


103. Who is sitting to the immediate right of Surya?

A. Umesh B. The one who is Banker

C. Rithesh D. The one who is Lawyer

E. Cannot be determined

104. Who is Doctor?

A. Qadir B. Rithesh C. Prakash

D. Tarini E. Cannot be determined

105. Who is sitting to the immediate right of the person whose profession
is Engineer?

A. Qadir B. Tarini C. Surya

D. Rithesh E. None of these


Set-22

Twelve persons are sitting in two parallel rows containing six persons each,
in such a way that there is an equal distance between adjacent persons. In
row 1 – Joshi, Jaya, Jeet, Jamal, Juber and Jina are seated and all of them
are facing south. In row 2 – Prem, Pinu, Pal, Punj, Poran and Pooja are
seated and all of them are facing north. Therefore, in the given seating
arrangement each member seated in a row faces another member of the
other row.

Only one person sits between Jaya and Jina. Joshi sits third to the left of
Juber. Neither Prem nor Poran faces Juber or Joshi. Prem does not sit at an
extreme end. Jeet does not face Prem and Jeet does not sit at an extreme
end of the line. Prem sits second to the right of Poran. Only one person sits
between Pooja and Pal. Neither Joshi nor Juber sit’s at extreme end of the
line. Pooja is not an immediate neighbour of Pinu and Prem does not face
Jina. Neither Pooja nor does Pal face Juber. Pal does not sit at the extreme
end.

106. How many person(s) sit(s) between Juber and Joshi?

A. None B. One C. Two

D. Three E. More than three

107. Four of the following five are alike in some way and hence form a
group. Which of the following is the one that does not belong to the
group?

A. Jaya B. Poran C. Punj

D. Joshi E. Jeet

108. Who among the following faces Pinu?

A. Jaya B. Joshi C. Jamal

D. Juber E. None of these


109. Who among the following sits to the immediate left of the one who
faces Punj?

A. Joshi B. Jamal C. Jeet

D. Juber E. None of these

110. Which of the following statement(s) is/are correct?

A. Poran faces Jaya

B. Jina sits between Jaya and Jamal

C. Punj sits second to the left of Prem

D. Pal faces Jina

E. None is correct
Set-23

Eight friends Hiram, Henry, Sofia, Paul, John, Betty, Rachel and Andrew are
reading different Sci-Fi novels Fugitive, Killer, Outlaw, Judgement,
Shadowfell, Shadowhunters, Legends, Scarlet. Four out of eight novels are
written by one particular author while rest four novels are not written by
the same author. They are sitting around a circular table and but not
necessarily in same order and reading novels but not necessarily in same
order. People who are reading novel by same author face away from the
table while others don’t.

John is sitting the second position to right of Betty and is reading Legends.
Sofia is sitting at the second position to the right of Rachel. Paul is reading
Judgement and is sitting third position to the right of Betty. Henry is
reading Fugitive and is sitting opposite to that of Hiram, who is facing away
from the center. Betty is reading Shadowfell. The person reading Legends is
sitting at the second position to the left of the person who is reading
Shadowhunters. Rachel who is reading Outlaw, is sitting at the third
position to the right of Henry and is reading one of the novels by the same
author. Both Paul and Henry are reading novels by same author. Hiram is
not reading Scarlet.

111. What is the position of Paul with respect to Rachel?

A. Third to the left B. Second to the right C. Third to the right

D. Fifth to the right E. Fouth to the left

112. Who sits third to right of Andrew?

A. Sofia B. Hiram C. Rachel

D. John E. Paul

113. If Betty and John interchange their positions and similarly Sofia and
Rachel interchange their positions while keeping their new directions
same as original ones, what is the position of Henry with respect to
Rachel?

A. Fourth to the left B. Fifth to the left C. Second to the left

D. Immediate Left E. None of these

114. Find the odd one out among the following.

A. Fugitive B. Killer C. Rachel

D. Rachel E. Andrew

115. Which of the following novels is read by the person who sits opposite
to Andrew.

A. Fugitive B. Killer C. Judgement

D. Outlaw E. Shadow hunters


Set-24

Twelve friends are sitting in two parallel rows containing six people in each
in such a way that there is an equal distance between two adjacent
persons. In row-1 Sriram, Tushar, Uma, Vivek, Wasim and Xanil are seated
and all of them are facing south direction. In row-2, Mano, Nirmal, Oviya,
Parthiv, Queen and Ranjith are seated and all of them are facing north
direction. Therefore in the given seating each member seated in a row
faces another member of the other row. No two persons, whose names
start with consecutive letters of English alphabet, are immediate neighbors
of each other. For example, Binu is an immediate neighbor of neither Aman
nor Chitra and so on. Each of them also likes different Kollywood actors -
Rajni, Kamal, Vijay, Ajith, Surya, Karthi, Dhanush, Vikram, Prasanth, Jeeva,
Arya and Prabhu. All the above information is not necessarily in the same
order.

Nirmal does not like Ajith. Tushar sits at an extreme end of the line. Only
two people sit between Tushar and the one who likes Dhanush. Nirmal sits
second to the left of the one who faces the one who likes Dhanush. The one
who likes Vijay sits second to the right of Oviya. Tushar does not face the
one who likes Vijay. Only two persons sit between Parthiv and Ranjith. Uma
does not sit at an extreme end of the line. Xanil is not an immediate
neighbor of Tushar. Vivek faces the one who likes Prasanth. Only three
people sit between the one who like Prasanth and Rajni. Ranjith likes
Karthi. One of the immediate neighbors of Mano faces the one who likes
Arya. The one who likes Kamal sits to the immediate right of Sriram. Parthiv
sits second to the right of the one who likes Jeeva. Only three people sit
between the ones who like Vikram and Prabhu respectively. Tushar does
not like Prabhu.

116. Which among the following actors was liked by Wasim?

A. Vikram B. Ajith C. Prabhu

D. Arya E. Cannot be determined


117. Four of the following five are alike in a certain way and hence form a
group. Which of the following does not belong to the group?

A. Vikram B. Rajni C. Surya

D. Arya E. Vijay

118. Which among the following combination is definitely true?

A. Nirmal-Karthi B. Parthiv-Rajni C. Sriram-Dhanush

D. Wasim-Kamal E. None of the above

119. Which among the following statements is definitely true?

A. Wasim and the one who like Dhanush are immediate neighbors.

B. The one who likes Kamal faces the one who sits third to the right of
Queen.

C. Only two persons sit between Mano and the one who likes Karthi

D. The one who likes Surya is an immediate neighbor of Ranjith and Oviya

E. None of the above

120. Which of the following persons sits immediate right of the person
who sits opposite to Ranjith?

A. Uma B. Wasim C. Sriram

D. Oviya E. Queen
Set-25

Some persons were watching ‘Zero’ in a multiplex, where several rows has
different number of seats. The rows were numbered such that the nearest
row from the screen was numbered 1 and the row just above it was
numbered 2 and so on. The seats were numbered from left to right such
that the leftmost seat was numbered as 1, its adjacent seat as 2 and so on.
All the persons were facing towards the screen which was in the north
direction. The below mentioned information is regarding few persons only.

The film was house-full and none of the seats was vacant.

The total number of rows does not exceed 7 and the total number of seats
was five times the total number of rows.

Mihir’s row number and seat number were same and it was an even prime
number.

Ranu sits second to the left of Shreya.

The number of persons to the right of Hema who was at seat number 3 in
the row number 5 was 1 more than the total number of rows.

Shreya is on the immediate right of Sanu who sits in row number 4.

Tina was seated at the left most seat in the last row, which was not prime
numbered.

Total number of persons in the top 3 farthest rows (from screen) was equal
to 18.

There were three seats between Saba and Tina.

Kavya was on the last seat of the first row.

The total number of persons in row 1 was thrice of the total number of
persons in row 4.

Shagun was seated in an odd numbered row except row 1.


121. How many seats were there in the multiplex?

A. 35 B. 25 C. 30 D. 20

E. Cannot be determined

122. What is the difference between the seat numbers of Kavya and
Sanu?

A. 2 B. 5 C. 6 D. 7 E. None of these

123. Who among the following is immediately ahead of Ranu?

A. Shagun B. Mihir C. Tina D. Kavya E. None of these

124. Four of the following five are alike in a certain way and thus form a
group. Which of the following does not belong to the group?

A. Tina B. Hema C. Kavya D. Saba E. Sanu

125. What is the total number of persons in the top two nearest rows?

A. 13 B. 11 C. 10 D. 12 E. None of these
Set-26

Certain number of persons is sitting around the circular table.

All of them are facing towards the center.

Distance between any two adjacent persons is same.

Akhil is facing Pravin, who is sitting second to the right of Tahir.

Sriman is facing Lokesh, who is sitting fourth to the right of Chahal.

None among Sriman, Chahal and Lokesh is an immediate neighbor of


Pravin.

The person, who is sitting four places away from Tahir, is not immediate
neighbor of Chahal.

Manoj, who is sitting seven places away from Tahir, is not immediate
neighbor of Akhil.

At most 13 people sit around the circular table.

126. How many people are sitting around the table?

A. 11 B. 9 C. 10

D. 13 E. None of these

127. How many people are sitting between Manoj and Akhil, when
counted from left of Akhil?

A. One B. Two C. Three

D. Four E. None of these

128. Which of the following statements is true?

A. Akhil sits third to the left of Lokesh

B. Sriman sits third to the right of Pravin


C. Tahir sits fifth to the left of Sriman

D. Chahal sits to the immediate right of Manoj

E. None is true

129. In certain way Akhil is related to Chahal and Tahir is related to


Lokesh and in same way that, who among the following person is related
to Sriman?

A. Chahal B. Manoj C. Pravin

D. Lokesh E. None of these

130. Who sits at an even numbered place?

A. Chahal B. Manoj C. Lokesh

D. Akhil E. None of these


Set-27

Anil, Bihu, Charu, Disha, Esha, Farah, Geet and Hina are sitting around a
square table such that four of them sit on each of the corners and four on
each sides of the table. All of them are facing towards the centre of the
table but not necessarily in the same order. Everyone is related to Hina as
father, mother, brother, spouse, daughter, son and niece.

Hina’s husband sits third to the right of Bihu who is a male. Only one
person sits between Farah and Anil, who is the son of Geet and Anil.. Bihu is
not adjacent to any female. Geet sits immediate right of Hina. Disha is the
granddaughter of Anil but not the daughter of Charu. Hina’s mother sits to
the immediate left of Disha. Bihu belongs to the third generation. Esha sits
adjacent to her father Charu. Hina sits second to the right of her daughter
Esha.

131. If Hina's brother sits immediate right of Bihu then who sits second to
the left of Anil?

A. Hina B. Hina's niece C. Hina's mother

D. Charu E. None of these

132. How is Anil related to Charu?

A. Father B. Father-in-law C. Son

D. Cannot be determined E. No relation

133. How is Hina related to the one who sits opposite to Hina?

A. Aunt B. Daughter C. Son

D. Mother E. None of these

134. Which of the following person sits third to the right of person who
sits opposite to disha?

A. Esha B. BIhu C. Geet


D. Anil E. None of these

135. Find the odd one out.

A. Geet –Hina B. Charu – Esha C. Geet - Disha

D. Hina – Bihu E. None of these


Set-28

Eight soldiers from A to G of Indian Army were seated around a round table
facing towards the center. They were of different ranks and have killed
different number of terrorists in an undercover operation.

Below given is the distribution of soldiers as per descending order of their


ranks.

H(24) > C(68) > F(74) > B(31) > A(79) > D(20) > E(52) > G(82)

Some additional information is given below:

A is neither adjacent to the one whose rank is the lowest nor to the one
whose rank is the second highest. The second highest ranked soldier was
sitting opposite to the one who killed the highest number of terrorists. F
was immediate right of the soldier who was third lowest in rank. The one
who killed the fourth lowest number of terrorists was sitting between G
and the one who was third highest ranked. D and the one whose number of
terrorists killed is a prime number were sitting opposite to each other.

136. Who was third to the left of the one who killed second highest
number of terrorists?

A. The one who killed lowest number of terrorists


B. The one whose rank was second lowest
C. The one who was highest ranked
D. The one who killed the lowest number of terrorists
E. Cannot be determined
137. Four of the following five are alike in a certain way and thus form a
group. Which of the following does not belong to the group?

A. D – G B. F – B C. B - C

D. A – F E. C - E
138. Who was seated opposite to E?

A. The one who killed second lowest number of terrorists

B. C C. The one who was third highest ranked

D. B E. None of these

139. Which of the following statements is false?

A. A sits second to the right of the one who killed the lowest number of
terrorists.

B. H sits adjacent to the one who killed lowest number of terrorists.

C. The one who was least ranked was immediate left of the one who killed
third lowest number of terrorists.

D. The one who killed second highest number of terrorists was opposite to
F.

E. All are false

140. Who were the immediate neighbours of the one who killed the
lowest number of terrorists?

A. E and F B. E and F C. C and F

D. H and C E. None of these


Set-29

Eight persons – A, B, C, D, E, F, G and H – from eight different fields viz.


Agriculture, Banking, Business, Army, Medicine, Navy, Software and
Teaching sit in two different rows, four persons in each facing each other
but not necessarily in the same order. The following information is given.

(a) B and the person from Teaching are adjacent to each other and one
among them sits at one of the ends.

(b) The person from Banking and G are opposite each other. H is not from
Banking.

(c) Either C or the person from Navy sits at one of the ends and both sit in
the different rows.

(d) Only one person sits to the left of G.

(e) H who is not from Medicine is the neighbour of the persons from Army
and Software.

(f) The person from Business who is not D is adjacent to A and opposite E,
who faces north.

(g) C sits to the right of the person from Army.

141. H is from which field?

A. Business B. Banking C. Agriculture

D. Teaching E. Navy

142. Who among the following are at the ends of the row?

A. H, the person from Business

B. B, E

C. G, D
D. The person from Teaching, E

E. F, A

143. Who sits opposite the person from Army?

A. B B. G C. The person from Banking

D. F E. The person from Business

144. Who is from banking field?

A. B B. A C. C

D. H E. None of these

145. Find the odd one out.

A. F-E B. A-G C. B-H

D. C-B E. None of these


Set-30

Eight persons Padmesh, Quincy, Rachna, Udvita, Wimal, Xavier, Yatin and
Zenith are sitting around a circular table. No two persons have same age.
Only the person whose age is in even number is facing inside the circle. All
the above information is not necessarily in the same order.

The one whose age is 17 years old sits second to the left of Padmesh. Two
persons sit between Padmesh and Quincy. The one who sits immediate
right of Quincy is 41 years old. One of the immediate neighbors of Quincy is
facing inside. Udvita sits second to the right of Quincy. Three persons sit
between the one whose age is 33 years old and the one whose age is 19
years. Quincy’s age is neither 33 nor 19. Wimal is an immediate neighbor of
Quincy, but not an immediate neighbor of Udvita. Yatin is 22 years old. The
one who sits second to the left of Udvita is 25 years old. Padmesh is 7 years
younger than Quincy. Neither Yatin nor Wimal is an immediate neighbor of
Zenith. Rachna is 3 years younger than Wimal, but not the youngest person
in the group. Two persons sit between the one who is the youngest and
Udvita.

146. Who among the following persons is the eldest in the group?

A. Wimal B. Zenith C. Udvita

D. Either A or C E. Cannot be determined

147. Persons in which of the following pairs face outside?

A. Padmesh and Quincy B. Wimal and Udvita C. Zenith and Xavier

D. Rachna and Padmesh E. None of these

148. How many persons sit between the one who is the oldest and the
one who is the youngest in the group?

A. One B. Two C. Three

D. Either C or B E. Cannot be determined


149. Four of the following five are alike in a certain way and hence form a
group. Which of the following does not belong to the group?

A. Quincy-22 B. Xavier-41 C. Wimal-33

D. Padmesh-36 E. Yatin-25

150. If all the persons are facing the opposite direction with respect to the
original direction, then who among the following persons sits third to the
left of the one who is the second oldest in the group?

A. The one who is 18 years old

B. The one who is 17 years old

C. The one who is 25 years old

D. The one who is 22 years old

E. None of these
Set-31

Irfan, Harish, Govind, Francis, Edwin, Diwan, Chandra, Bala and Ashwin are
sitting in two parallel rows. Each row has six seats. Nine persons are sitting
in nine seats and there are three vacant seats. Therefore in the given
arrangement, each member seated in a row faces another member of the
other row or vacant seat. Vacant seats are not facing each other. Each of
them like different mobile phones like Redmi, Lenovo, Samsung, Sony,
Micromax, Moto, Asus, Oppo and Vivo. The person sit in row-1 facing south
direction and the person sit in row-2 facing north direction. Row-1 is
followed by Row-2. All the above information is not necessarily in the same
order.

The person, who is sitting third to the right of Diwan, is facing the person or
vacant seat who/which is immediate right of the one who likes Oppo.
Govind is an immediate neighbor of vacant seats. Only one person sits
between the one who likes Asus and the one who likes Oppo. Diwan and
the one who likes Samsung are facing each other. Edwin is second to the
right of the one who likes Sony. One who likes Lenovo and the one who
likes Moto are immediate neighbors of each other. Ashwin, who likes
Micromax, is not sitting adjacent to any of the vacant seats. One who likes
Lenovo and one who likes Vivo are facing each other. Harish and Irfan, who
likes Samsung, are diagonally opposite to each other. The number of
persons to the right of Edwin and the number of persons to the left of
Francis are same. Diwan likes Sony mobile phone. One, who likes Redmi,
sits third to the right of the one who likes Lenovo. Bala likes neither Oppo
nor Asus. Chandra sits right of Govind, who does not face south direction.

151. Who among the following persons likes Asus mobile phone?

A. Francis B. Govind C. Chandra

D. Edwin E. Cannot be determined


152. Which of the following combinations is definitely true?

A. Bala-Redmi B. Govind-Oppo C. Irfan-Vivo

D. Edwin-Moto E. None of these

153. How many persons sit between the one who likes Redmi and the one
who likes Sony?

A. Four B. Three C. Two

D. One E. None of these

154. The person who sits second to the left of Francis is facing whom
among the following persons?

A. Bala B. Edwin C. Harish

D. Diwan E. None of these

155. Who among the following sits opposite to the person who sits third
to the left of Edwin?

A. Chandra B. Francis C. Govind

D. Irfan E. None of these


Set-32

Eight people A, B, C, D, E, F, G and H are sitting in two parallel rows


containing four people each. B, D, F and H are sitting in row-I facing North
and A, C, E and G are sitting in row-2 facing South (but not necessarily in
the same order). Thus, each person sitting in row-I faces another person
sitting in row-2. Each of the persons lives on different floors of a building
.i.e. floor 1 to 8.

B lives on 4th floor and the person who lives on the 1st floor is not the
neighbour of G in the row. H sits to the opposite of the person who sits to
the immediate right of the person who lives on 2ndfloor. The person who
lives on 3rd floor sits exactly between D and the person who lives on
8thfloor. G lives on even prime numbered floor. C lives neither on 1st floor
nor on 5th floor. G sits opposite to F, who lives on the 8th floor. E sits
between A and G. D sits at the left end of the row and lives on 6th floor.

156. Which of the following represents the people sitting at extreme ends
of both the rows?

A. A, C and B, D B. A, G and D, B C. F, D and A, C

D. A, G and F, D E. None of these

157. Who amongst the following sits to the immediate left of the person
who lives on 2nd floor?

A. B B. C C. E

D. A E. F

158. Which of the given statements is true with respect to the given
arrangement?

A. A sits opposite to H

B. H lives on second floor


C. C sits opposite to B

D. F lives on seventh floor

E. All are true

159. How many people live between C and H?

A. None B. Two C. Four

D. Three E. None of these

160. Who is/are the immediate neighbours of the person who is sitting
opposite to H?

A. G,C B. A,G C. E,C

D. D,F E. None of these


Set-33

Sixteen players Virat, Dhawan, Rohit, Rahul, Vijay, Rayudu, Dhoni, Dinesh,
Chahal, Yadav, Raina, Bumrah, Pandya, Ishant, Pujara and Umesh were
seated on eight sofas which were arranged around the circular table to
attend the BCCI annual meeting. There were two persons on every sofa and
all the players were seated facing in the centre but not necessarily in the
same order.

Virat was third to the left of Umesh. Umesh and Raina were seated at a gap
of one person. Rahul was to the immediate left of Pandya. Ishant and
Rayudu were on the same sofa and there were two persons between
Pandya and Rayudu. Yadav was to the immediate right of Bumrah and they
were on different sofas. Dhawan's sofa was neighboring Chahal’s sofa and
Umesh’s sofa. Rahul and Umesh were on different sofas. Dhoni was second
to the right of Dinesh. Ishant was not neighbouring Rohit's sofa. Chahal and
Virat were not on the same sofa. Rahul was second to the right of Dhawan.
Virat and Dhawan were neither on the same sofa nor on the neighboring
sofas. Pandya is sixth to the right of Dinesh. Pujara was not seating with
Pandya.

161. Four of the following five are alike in a certain way and hence form a
group. Which of the following does not belong to the group?

A. Pujara-Bumrah B. Pandya-Vijay C. Rohit-Chahal

D. Dinesh-Umesh E. Virat-Raina

162. Who sits with Pandya?

A. Rahul B. Dinesh C. Dhoni

D. Vijay E. Cannot be determined

163. Which among the following pairs represent the immediate neighbors
of Virat?

A. Dhoni-Vijay B. Rohit-Raina C. Bumrah-Pujara


D. Bumrah-Pujara E. Cannot be determined

164. Who sits third to the left of Dinesh?

A. Rohit B. Yadav C. Chahal

D. Pujara E. Cannot be determined

165. How many persons sit between Umesh and Ishant, if counted from
right of Umesh?

A. Six B. Eight C. Seven

D. Eleven E. None of these


Set-34

Preet's family had 6 members Preet, Geet, Shiva, Roop, Daya and Moni.
Each of them was related to Preet in some manner: Mother, Father, Sister,
Wife and Brother. They were seated in a row, facing east. They were aged
differently. It is known that any child and his/her spouse would be younger
to his/her parents. Any husband would not necessarily be elder to his wife.

Preet's wife was 32 years old.

Preet's father was 7 years younger than Preet's mother.

Geet was 29 years old and was to the immediate left of Preet's brother.

Daya was seated at one of the extreme ends at a gap of 3 from Preet's
father.

Preet's father was to the immediate right of the one who was 30 years old.

Daya was 33 years elder to Moni.

Preet was 3rd to the left of his sister.

The one who was 34 years old was neighboring Daya.

Roop was to the immediate left of Moni.

166. Who among the following is the youngest in the family?

A. Preet B. Preet’s wife C. Preet’s sister

D. Shiva E. Preet’s brother

167. What was the difference between the ages of the persons sitting at
the extreme ends?

A. 20 B. 35 C. 33

D. 28 E. 30
168. What is the position of Preet’s wife with respect to the eldest person
in the family?

A. Immediate right B. Second to the right C. Fourth to the left

D. Third to the left E. Fourth to the right

169. How is Geet related Preet?

A. Mother B. Wife C. Father

D. Sister E. None of these

170. Who sits second to left of Geet if Geet changes her/his position with
Moni?

A. Roop B. Preet C. Shiva

D. Daya E. None of these


Set-35

Sachin, Thara, Udhay, Aravind, Baskar, Manju, Chinnu and Vinoth are sitting
around a square table in such a way that four of them sit at four corners of
the square table facing outwards the centre of the table while four of them
sit in the middle of the four sides facing centre of the table. Each of them
likes different colors like Pink, Red, Orange, Blue, Yellow, Black, White and
Green. Each of them was born in different months January, February,
March, April, May, June, July and August of the same year. The person who
was born in the consecutive months will not sit together i.e. The person
born in January can not be an immediate neighbor of the person who was
born in February. All the above information is not necessarily in the same
order.

Vinoth likes neither Black nor Orange. The one who was born in March sits
on the immediate left of Chinnu. Manju does not like Black color. The one
who likes Pink was born in the month which has less than 30 days. Thara
likes Blue and sits opposite to the one who likes Green. Vinoth was born in
January and is an immediate neighbor of Aravind and Sachin. Chinnu likes
Red and was born in June. The one who was born in April sits opposite to
the one who likes Pink. Vinoth doesn’t sit on the middle sides of the table.
Chinnu doesn’t face outside the table. Aravind was born in August month.
Manju is an immediate neighbor of the one who was born in June and
Thara. Manju was born in the month which has more than 30 days. Udhay
likes Yellow and was born on the month which has 30 days and is not an
immediate neighbor of Chinnu.

171. Who was born in July month?

A. Sachin B. Thara C. Manju


D. Either A or B E. Cannot be determined
172. Which of the following persons is/are immediate neighbor(s) of the
one who likes Pink color?
A. The one who was born in June and Vinoth
B. The one who was born in June and the one who was born in July
C. The one who was born in March and Chinnu
D. The one who was born in June and the one who was born in May
E. None of these
173. Four of the following five are alike in a certain way and hence form a
group. Which of the following does not belong to the group?

A. The one who likes Green


B. The one who likes Red
C. The one who likes White
D. The one who likes Blue
E. The one who likes Black
174. Which of the following combinations is true?

A. Baskar-February-Pink
B. Chinnu-Red-June
C. Thara-Blue-July
D. Aravind-Black-August
E. All of these
175. Who are the immediate neighbours of the person who sits third to
the left of Sachin?

A. Thara – Arvind B. Bhaskar – Vinod C. Manju - Bhaskar

D. Chinnu – Thara E. None of these


Set-36

Eight persons Aashvi, Bhairav, Chavez, Dhruvi, Eeshan, Faizan, Geetika and
Hendrik are sitting in a straight line facing north direction. They all have
different ages such as 8, 12, 14, 21, 28, 36, 44 and 56.

Two persons are sitting between Chavez and the one whose age is 56 years.
The one whose age is 14 years is sitting second to the left of Chavez.
Bhairav sits to the immediate left of the one whose age is 56 years. One
person is sitting between Hendrik and the one whose age is 56 years. Either
Hendrik or the one whose age is 56 years is sitting at the extreme end.
There is a difference of 6 years between the ages of Faizan and Geetika and
two persons are sitting between them. The one whose age is 12 is sitting
second to the left of Dhruvi. Dhruvi is not sitting at the extreme end. There
is one person sitting between Faizan and the one whose age is 44 years.
Aashvi is an immediate neighbor of the one whose age is 44 years. Eeshan
and the one whose age is 21 years are immediate neighbors. Hendrik is
older than Aashvi.

176. How many persons are sitting between the people who are having
age difference of 4 years?

A. One B. Two C. None

D. Four E. None of the above

177. What is the age difference between Faizan and Dhruvi?

A. 8 years B. 7 years C. 5 years

D. 6 years E. None of these

178. What is the age of Hendrik?

A. 44 years B. 21 years C. 36 years

D. 56 years E. 14 years
179. What is the sum of the ages of the persons who sit at extreme end?

A. 52 B. 50 C. 42

D. 68 E. 64

180. What is the name of the person whose age is 56 years?

A. Faizan B. Bhairav C. Dhruvi

D. Eashan E. None of these


Set-37

Eight persons Rishab, Tanay, Virak, Nikhil, Yash, Jagat, Harsha and Liyan are
sitting around a square table, four of them sit at the corner of the table and
four of them sit at the sides of the table. Among eight persons some of
them are facing centre and some of them are facing away from the centre.
Each of them is born in different months of same year viz. January,
February, March, April, May, July, August and October.

Tanay sits third to the right of Rishab. There are three persons sitting
between Rishab and the one who was born in February. One who was born
in August is second to the right of Nikhil. Rishab was born in one of the
months before the month in which Virak was born. Tanay and the one who
was born in August are immediate neighbors. One who was born in October
and the one who was born in April are immediate neighbors of Rishab.
Liyan was born in May and sits second to the right of Tanay. Virak sits
fourth to the right of Yash. Virak and Yash are not facing each other. One
who was born in July sits third to the left of Virak. Jagat sits second to the
left of Liyan. Immediate neighbors of Rishab are facing direction opposite to
Yash. Virak is facing centre. Tanay sits at corner of the table.

181. Who among the following persons sits third to the left of Harsha?

A. Tanay B. Yash C. Nikhil


D. Liyan E. None of these
182. Which of the following combinations is definitely true?

A. Yash-February B. Rishab-July C. Virak-March


D. Jagat-April E. Harsha-October
183. Four of the following five are alike in a certain way and hence form a
group. Which of the following does not belong to the group?

A. Rishab B. Tanay C. Yash


D. Nikhil E. Liyan
184. What is the position of Nikhil with respect to the one who was born
on October?

A. Third to the right B. Third to the left C. Fourth to the


left

D. Immediate right E. Cannot be determined

185. Who among the following sits opposite to the person who sits fifth
to the right of Liyan?

A. Rishab B. Harsha C. Nikhil

D. Virak E. None of these


Set-38

Ten candidates Aravind, Ram, Atal, Arivu, Anbu, Vernish, Fazith, Ragu,
Nithya and Aruna are seated in a Row which contains twelve seats. All the
people facing the north direction and there are two vacant seats. Each of
them likes different Banks to work such as BOB, Dena Bank, BOI, Canara
Bank, LVB, Axis Bank, RBI, Corporation bank, HDFC Bank and IOB. All the
given information is not necessarily in the same order.

Anbu sits third to the left of one of the vacant seats. The only neighbor of
the person who likes Corporation Bank sits second to the left of Atal. Arivu
sits second to the right of one of the vacant seats. Ram does not like IOB.
Vernish sits immediate left of the person who likes RBI. There is no vacant
seat at the extreme ends. The person who likes BOI and Canara Bank are
immediate neighbors. Aravind sit second to the left of Ram. The person
who likes LVB and Axis Bank are immediate neighbors. Only one person sits
between Aruna and one of the vacant seats. The person who likes Dena
Bank and IOB are immediate neighbors. Fazith sits third from the left end
and likes RBI. The person who likes BOB sits second to the right of one of
the vacant seat. No person sits between Ram and Aruna. Nithya neither
likes IOB nor BOI. There is one vacant seat between Ragu and Nithya and no
other person sit between them. No person sits between Aravind and
Fazith. Five people seated between two vacant seats. Aruna is not a
neighbor of Anbu. Arivu does not like BOB. Ram is not a neighbor of the
vacant seat. Vernish likes Axis Bank. One of the immediate neighbors of
Ram likes Dena Bank. Only one person sits to the left of Vernish.

186. Who among the following likes Canara Bank?

A. Arivu B. Nithya C. Ragu

D. Ram E. Can’t be determined

187. Who sits third to the right of the person who likes Axis bank?

A. Aravind
B. The person who likes RBI

C. The person who likes Corporation Bank

D. Both A and C

E. Cannot be determined

188. Who among the following sits exactly between the person who likes
Corporation Bank and the person who likes IOB?

A. Aruna

B. The person who likes BOB

C. The person who likes Dena Bank

D. Ram

E. None of these

189. How many people sit to the right of the person who likes BOI?

A. None B. One C. Two

D. Three E. More than three

190. Find the odd one out.

A. Fazith - RBI B. Raghu - IOB C. Aruna – BOB

D. Anbu - LVB E. Nithya - Dena Bank


Set-39

Hara, Raga, Daha, Gada, Sada, Yaja, Paya, Waha, Edha, Baba and Taha are
eleven persons sitting in a row facing north. Each person likes different
flowers. They are Azalea, Camellias, Freesia, Hyacinth, Holy, Crocus, Lilac,
Daisy, Orchid, Bergamot and Begonia but not necessarily in the same order.

Gada and Edha are immediate neighbors. The one who likes Azalea and the
one who likes Camellias are immediate neighbors. There are as many
persons sit between Hara and Waha sits between the one who likes Lilac
and the one who likes Bergamot. Waha likes Freesia and sits either fourth
from one of the ends. Only two persons sit between Baba and Daha. The
one who likes Camellias sits exactly between Paya and Waha. Yaja likes
Bergamot and sits at one of the extreme ends. Paya is not an immediate
neighbor of the one who likes Bergamot. Daha sits second to the right of
the one who likes Camellias. There are as many person sits between Paya
and Yaja sits between Hara and Taha. Gada sits second to the left of Sada
who likes Daisy. Daha does not like Lilac. The person(s) sitting between the
one who likes Hyacinth and the one who likes Crocus is same as the
person(s) sitting between the one who likes Crocus and Sada. Edha likes
Holly and is not an immediate neighbor of the one who like Orchid or
Begonia. Neither Hara nor Taha is an immediate neighbor of the one who
likes Freesia.

191. Which of the following person sits exactly in the middle?

A. Paya B. Edha C. Gada

D. Daha E. Cannot be determined

192. Which of the following flowers is liked by Hara?

A. Bergamot B. Orchid C. Begonia

D. Hyacinth E. Cannot be determined


193. Which of the following persons like Crocus and Lilac respectively?

A. Gada and Baba B. Baba and Gada C. Gada and Taha

D. Daha and Raga E. Cannot be determined

194. Which of the following is definitely true?

A. Only one person sits between the one who likes Holly and the one who
likes Daisy

B. Raga sits third to the left of the one who likes Hyacinth

C. Hara likes Orchid and third to the left of Waha

D. Sada is an immediate neighbor of the one who like Begonia

E. None of these

195. Which of the following person sits between Edha and Baba?

A. Daha B. Gada C. Taha

D. Paya E. None of these


Set-40

Eight friends Anab, Manov, Bayaz, Takbir, Neaha, Pakija, Qumila and Vitna
went to State bank of India for depositing different amount in their savings
accounts and they all were asked to sit around a circular table facing
outside the centre. All of them had different amount (in Rs) to deposit such
as 4000, 6000, 15000, 1000, 2000, 3000, 9000 and 7000, but not necessarily
in the same order.

The one who has the lowest amount to deposit sits opposite to the one
who has the highest amount to deposit. Qumila deposited an amount
which is sum of the amount deposited by his/her immediate neighbors. The
one who deposited thrice the amount of Vitna sits opposite to Vitna. Pakija
is an immediate neighbor of Neaha and Vitna. Pakija is neither an
immediate neighbor of the one who deposited 6000 nor of the one who
deposited 9000. Only one person sits between Vitna and the one who has
an amount 9000 to deposit. The sum of the amount deposited by Anab and
Vitna is equal to the amount deposited by Takbir, whose amount is multiple
of 6. There are as many person(s) sitting between the one who has the
lowest amount to deposit and Vitna as sitting between Neaha and Anab.
Qumila has the highest amount to deposit in the savings account. The
amount deposited by Vitna is not a multiple of 6. The person Bayaz has only
3000 Rupees to deposit. The one who has amount 7000 to deposit sits
immediate left of Anab.

196. What is the average amount (In Rs.) deposited by Pakija and Manov?

A. 4000 B. 3000 C. 2000

D. 1500 E. None of these

197. How much amount (in Rs.) did Takbir deposit in savings account?

A. 6000 B. 1000 C. 7000

D. 9000 E. Cannot be determined


198. Who sits third to left of the one who deposited Rs.6000?

A. Qumila B. Pakija C. Bayaz

D. Either Qumila or Pakija E. Either Pakija or Bayaz

199. Which of the following statements is true?

A. Qumila sits to the immediate left of Takbir

B. The difference between the amount deposited by Anab and Pakija is


Rs.2000

C. The one who deposited Rs.3000 is not an immediate neighbor of Vitna

D. Vitna deposited thrice of Pakija

E. Neaha deposited Rs.7000

200. Who among the following persons sits second to the left of Neaha?

A. The one who deposited Rs.6000

B. The one who deposited Rs.2000

C. The one who sits opposite to Bayaz

D. The one who deposited Rs.7000

E. The one who deposited Rs.7000


Set-41

Nine persons – Lata, Prem, Deep, Avi, Roop, Geet, Kunj, Joy and Meet are
sitting in a row facing in North and South directions not necessarily in the
same order. All of these persons comprise a family divided among three
generations.

The following information is also known about them:

Deep who is Lata’s father sits fourth to the right of Prem who is the only
daughter of Roop and they both face in the opposite directions. Avi sits
third to the right of Geet who is Prem’s grandmother and one of them sits
adjacent to Meet. Avi is unmarried and faces in a direction similar to that of
Kunj. Roop who is the sister of Avi sits at one of the ends with a female.
Deep’s neighbours are females and face in opposite directions. Geet’s
spouse Deep faces in the North direction. Meet who is the wife of Kunj sits
exactly in the middle of the row facing in South direction. Lata, who is male,
is the only sibling of Kunj sits exactly between Avi and Prem’s cousin who is
a male. The people sitting at ends face in North direction. Lata faces in
same direction as that of Geet. Deep sits adjacent to Meet.

201. How is the one who sit at the extreme right of the row related to
Deep?

A. Daughter B. Son C. Wife

D. Son in law E. Daughter in Law

202. Who among the following sits third to the right of Kunj?

A. No one B. Kunj’s Wife C. Kunj’s Daughter


D. Kunj’s Father E. Kunj’s Sister in law
203. How many females sit between Lata’s spouse and Prem’s cousin?

A. None B. Three C. More than three

D. One E. Two
204. What is difference between the number of males facing south
direction and the number of females facing north direction?

A. Zero B. One C. Two

D. Three E. Four

205. Four of the following five are alike in a certain way and hence form a
group. Which of the following does not belong to the group?

A. Geet B. Roop C. Lata

D. Kunj E. Meet
Set-42

Eight persons P to W are seated around a circular table in such a way that
four of them are facing towards the centre and four of them are facing
away from the centre but not necessarily in the same order. Each of them
likes different cricket players among Sachin, Dravid, Sehwag, Dhoni, Virat,
Ganguly, Yuvraj and Dhawan but not necessarily in the same order.

R sits second to the left of V who likes Yuvraj. Three persons sit between R
and the one who likes Dravid, both of them face each other. Two persons
sit between the one who likes Dravid and U. Immediate neighbors of U face
opposite to each other (If one facing the center other facing away from the
center or vice versa). W sits third to the right of Q, who is an immediate
neighbor of U. T and P are immediate neighbors, one of them is an
immediate neighbor of V. W likes Ganguly. The ones who like Virat and
Dhoni sit opposite to each other, both of them are facing away from the
center. The one who likes Dhawan sits to the immediate right of the one
who likes Dhoni. Not more than two persons facing same direction sit
together. U and Q do not like Sehwag.

206. Who among the following persons likes Virat?

A. T B. P C. S

D. Q E. Cannot be determined

207. Who among the following persons sits third to the right of the person
who likes Sachin?

A. The one who likes Ganguly B. The one who likes Dhawan

C. The one who likes Virat D. The one who likes Yuvraj

E. None of these

208. If T is one of the immediate neighbors of V, then who among the


following persons sits to the immediate right of the person who likes
Dravid?
A. P B. T C. V

D. Either T or P E. Cannot be determined

209. How many persons sit between the one who likes Sehwag and V, if
counted from the right of V?

A. One B. Two C. Three

D. Four E. None of the above

210. Which of the following statements is definitely true?

A. T sits opposite to the one who likes Dhawan.

B. P and U sit opposite to each other, both of them are facing away from
the centre.

C. The one who likes Yuvraj and the one who likes Sachin sit opposite to
each other, both of them are facing towards the centre.

D. Number of persons sit between U and T is same as P and R.

E. None of the above


Set-43

Seven persons – Mani, Basu, Shiv, Teer, Jadu, Pari and Ramu, were sitting in
a straight line, facing towards north. These persons were born in different
years – 1975, 1982, 1986, 1989, 1991, 1996 and 1999, but not necessarily in
same order. There persons were born in different cities – Mandi, Jaipur,
Agra, Ajmer, Noida, Jhansi and Pune, but not necessarily in same order.

Person who was born in 1986 in Ajmer was sitting on the extreme right end
of the row.

Jadu was born in 1999 and was born in Pune.


Two persons were born between the persons who were born in Noida and
Agra.
Basu, who was born in Jhansi, was sitting on the left of the person born in
Jaipur.
Pari, who was born after Teer, was sitting on the left of Teer, who was born
in Mandi.
At most three persons were sitting on the left of Mani, who was born in a
leap year.
Teer wasn’t sitting third from the left end of the row.
Three persons were sitting between Mani and Shiv, who was born in 1975.
Pari wasn't born in 1991.
Shiv wasn’t born in Noida.
Only 1 person was born between person who was sitting on the immediate
left of Shiv and the person who was sitting in the middle of the row.
211. Who among the following was sitting in the middle of the row?

A. The one born in 1996 B. The one born in 1982


C. The one born in 1975 D. The one born in 1999
E. The one born in 1989
212. Which of the following combinations is/are correct?

A. Mani – 1996 – Noida B. Pari – 1989 – Jaipur

C. Teer – 1999 – Mandi D. Shiv – 1975 – Agra

E. Basu – 1986 – Ajmer

213. How many person(s) were sitting between Jadu and Teer?

A. None B. One C. Two

D. Three E. More than three

214. Who among the following was sitting on the extreme left end of the
row?

A. Mani B. Basu C. Jadu

D. Pari E. None of these

215. What is the position of the one born on 1982 with respect to Ramu?

A. Immediate left B. Third to the right C. Immediate right

D. Second to the left E. None of these


Set-44

Eight persons – Ankit, Ankush, Amjad, Akash, Akshay, Ashish, Ashad and
Arman, were sitting around a circular table, facing towards the centre.
These persons were playing cards and have won a different amount of
money.

Amjad was sitting neighboring Ankush and Ashish and won Rs. 40. The
person, who was sitting opposite to the person neighboring Ankit, won Rs.
30. One of the persons won Rs. 50. Ankush was sitting third to the right of
the person, who won the highest amount. Ankit was sitting exactly opposite
to the one, who won the highest amount. Every person won at least Rs. 20.
Sum of the money won by Ashad and Arman was more than the money
won by Ankush but less than the one who won the highest amount. The
sum of the amount won by Ankit and his neighbors was Rs. 200. Ankit won
the less money than his neighbors. Akash won Rs. 80, and was not an
immediate neighbour of the one who won highest amount of money.
Ankush and his neighbours and the person sitting opposite to him did not
win the least amount of money. Arman won more money than only one
person. One of the persons won Rs. 20. The person sitting opposite to
Amjad won Rs. 60 which is half of the sum of the amount won by the ones,
who won the highest and the lowest amount of money.

216. How many person(s) were sitting between the ones who won the
highest amount and the least amount, when counted from the left of the
one who won the least amount?

A. None B. One C. Two

D. Three E. More than three

217. What is the amount won by one, who was second to the right of the
one, who won the least amount?

A. Rs. 45 B. Rs. 90 C. Rs. 70

D. Rs. 85 E. Rs. 60
218. Difference of the amount won by Akash and Ankush is equal to the
amount won by whom among the following persons?

A. Ashish B. Arman C. Ankit

D. Amjad E. None of the persons

219. Who among the following was sitting third to the right of the one
who won the least amount?

A. Ashad B. Ankit C. Amjad

D. Ankush E. Either Amjad or Ankush

220. Who among the following persons won highest amount of money?

I. Ashish II. Akshay III. Ankush

A. Only III B. Either II or III C. Either I or III

D. Only II E. Other than the given options


Set-45

Eight students Anaika, Artika, Advita, Ankita, Asvitha, Ashwika, Anvita and
Aadvika like different subjects like Hindi, Physics, Chemistry, Botany,
Zoology, Maths, English and History but not necessarily in the same order.
All of them are sitting around the square table in such a way four of them
sitting in the middle of each sides of the table and facing inside and
remaining four of them are sitting at corner of the table and facing outside.

Anaika is facing inside the table and not an immediate neighbor of the one
who likes Hindi. Anvita is sitting second to the right of the one who likes
Botany and the one who is sitting at corner of the table. There are three
persons between the one who like Botany and Aadvika. The one who likes
Physics sits third to the right of Anvita. Artika sits exactly opposite to the
one who likes Maths subject. Aadvika sits third to the right of Ashwika, who
like Chemistry subject. The one who likes Zoology sits exactly opposite to
the one who likes Chemistry. Aadvika likes History subject. Anvita sits third
to the right of the one who like Physics subject. Anvita and Advita face
opposite direction and Ankita sits third to the right of Advita. The one who
like Hindi subject is sitting exactly opposite to the one who is immediate
left of the one who like English subject. The number of people sitting
between the one who likes English subject and the one who likes History
subject when counted in anti-clockwise direction from the one who likes
English subject are one more than the number of people sitting between
the one who like Maths subject and the one who like Hindi subject when
counted in anti-clockwise direction from the one who likes Hindi subject.

221. Four of the following five are alike in a certain way and hence form a
group. Which of the following does not belong to the group?

A. Asvitha-Physics B. Anvita-Maths C. Ankita-Botany

D. Artika-Zoology E. Anaika-English
222. Who among the following persons likes Zoology?

A. Anaika B. Advita C. Artika

D. Ashwika E. Asvitha

223. Which of the following persons are sitting on the same side of the
square table?

A. Anaika, Ankita and Ashwika

B. Asvitha, Artika and Ashwika

C. Ashwika, Advita and Anvita

D. Ankita, Anaika and Anvita

E. Aadvika, Advita and Artika

224. Which of the following statement is definitely true?

A. Anaika sits second to the left of the one who like Chemistry subject

B. Aadvika sits second to the right of the one who like Maths subject

C. Advita sits second to the left of the one who like English subject

D. Advita sits third to the left of the one who like Hindi subject

E. All are true


Set-46

A group of eight friends – Avdhesh, Baadal, Chaand, Dinesh, Eshan,


Faneesh, Ganesh and Harshit are sitting in a straight line facing north. Each
of them has different professions viz. Doctor, Banker, Businessman,
Teacher, Lawyer, Engineer, Pilot and CA. Each of them like different colours
viz. Pink, Yellow, Red, Black, Blue, Orange, White and Green, but not
necessarily in the same order.

Avdhesh, who is a lawyer, sits third to the left of Faneesh. Neither Avdhesh
nor Faneesh sit at the extreme ends of the line. Eshan, who likes pink color,
is a businessman. Eshan is not an immediate neighbor of either Avdhesh or
Faneesh. One person is sitting between Harshit and Chaand, Harshit likes
orange color. Chaand, who is a pilot, likes Red colour. Ganesh, who is a
doctor, sits at the extreme end of the line and likes white colour. Baadal,
who is a teacher likes Green colour and sits to the immediate left of
Avdhesh. Avdhesh does not like either Blue or Yellow colour. One who is a
CA sits to the immediate left of the Doctor. Faneesh, who is a Banker, does
not like Yellow colour.

225. Which one of the following likes Black color?

A. Dinesh B. Faneesh C. Harshit

D. Avdhesh E. None of these

226. Who is sitting between Chaand and Avdhesh?

A. Dinesh

B. Baadal

C. Harshit

D. The one who likes Black color

E. None of these
227. Which colour is liked by the one who is a Banker?

A. Yellow

B. Black

C. Blue

D. Cannot be determined

E. None of these

228. Which one of the following persons is an Engineer by Profession?

A. Harshit

B. The one who likes black color.

C. Dinesh

D. The one who likes Blue color.

E. None of these

229. Which one of the following persons is sitting third to the right of one
who likes yellow color?

A. Harshit

B. Faneesh

C. The one, who is a teacher.

D. The one who likes Blue color.

E. None of these
Set-47

Eight persons namely H, G, E, D, C, Y, X and Z are sitting around the circular


table but not necessarily in the same order. Some of them are facing inside
the circle while some of them are facing outside the circle. Each of them
likes different Rivers among Ganga, Yamuna, Saraswati, Gomti, Godavari,
Narmada, Kaveri and Mahanadi but not necessarily in the same order.

E doesn’t sit adjacent to X and do not like Ganga River. Only two persons sit
between G and H. One who sits immediate left of X likes Yamuna river and
faces inside the circle. Two persons sit between D and E. Both D and E are
facing same direction. The one who likes Narmada River sits immediate
right of the one who likes Kaveri River but not sitting opposite to the one
who likes Saraswati River. D sits second to the left of X. The one who like
Godavari River faces the one who like Mahanadi River. C sits opposite to Y
who sits to the immediate right of D. The one who likes Saraswati River sits
between E and Y. The one who likes Ganga River doesn’t sit to the
immediate right of E. X and Z are faces outside the circle. E faces opposite
direction to X. Z sits second to the left of G. The one who likes Mahanadi
River sits second to the right of the one who likes Saraswati River.

230. E likes which among the following River?

A. Ganga River B. Yamuna River C. Kaveri River

D. Gomti River E. None of the above

231. Who among the following likes Narmada River?

A. Z B. C C. D

D. H E. None of the above

232. How many persons sit between the one who like Saraswati River and
the one who like Kaveri River, if counted from the left of the one who like
Kaveri River?

A. Two B. One C. Three


D. Four E. None of the above

233. Who sits opposite to Y?

A. The one who like Narmada River

B. The one who like Yamuna River

C. The one who like Mahanadi River

D. The one who like Gomti River

E. None of the above

234. Which of the following statement is definitely true?

A. Only one person sit between H and the one who likes Ganga River, if
counted from the left of H.

B. The one who like Narmada River and the one who like Gomti River is
sitting adjacent to each other.

C. The one who like Godavari River sits second to the left X.

D. If Z and X interchanges their place, then X sits to the immediate left of Y.

E. None of these
Set-48

Seven persons namely Mahesh, Tahir, Fiyaz, Manoj, Tarun, Baskar and
Harish are sitting in a linear row and all are facing in north direction. The
total length of the linear row is 50m. All the persons, who are not sitting at
any end of the row, are not equidistant from their immediate neighbours.
Distance between two consecutive persons is among 5m, 6m, 10m and
12m.

Baskar is seated 23m to the left of Manoj. Mahesh, who is seated at one
end of the row, is 15m away from Tahir. Tarun is seated 27m to the left of
Fiyaz. Distance between Harish and the one who is seated at one end of the
row is 6m. Distance between Mahesh and its immediate neighbour is not
10m. Distance between Fiyaz and Manoj, is not 22m.

235. Who is seated two places to the left of Harish?

A. Baskar B. Manoj C. Tarun

D. Fiyaz E. None of these

236. How many persons are seated to the right of Baskar?

A. Three B. Four C. Five

D. Six E. None of these

237. What is the distance between Manoj and Mahesh?

A. 6m B. 27m C. 29m

D. 50m E. None of these

238. Who among the following sits to the immediate right of Fiyaz?

A. Manoj B. Harish C. Tahir

D. Mahesh E. None of these


239. What is the distance between Baskar and Tahir?

A. 5m B. 12m C. 17m

D. 18m E. None of these


Set-49

8 friends A, B, C, D, E, F, G and H sit around a circular table facing inside and


like basketball, football, cricket, badminton, kho kho, table tennis, hockey,
and volleyball.

The one who likes football sit between C and E. H sits 2nd to the right of the
friend who likes basketball. B is the neighbour of the one who likes
basketball. The friend who likes table tennis sits opposite to G. E likes
cricket and sits immediate right of F. A and D both don’t like either
badminton or table tennis. G likes kho kho and sits 2nd to the right of the
one who likes volleyball. F doesn’t like football. B likes hockey.

240. H likes which game?

A. Basketball B. Football C. Badminton

D. Table Tennis E. Volleyball

241. Who sits between the one who likes Basketball and the one who
likes Football?

A. G B. H C. F

D. E E. D

242. A likes which game?

A. Volleyball B. Basketball C. Football

D. Table Tennis E. Either option A or C

243. Who sits 2nd to the right of F?

A. A B. D C. F

D. C E. Can't be determined
244. How many friend(s) sit between F and the one who likes Football
when counted in anticlockwise direction from F?

A. 1 B. 2 C. 3

D. 4 E. 5
Set-50

Ten friends Amol, Darsh, Farhan, Isaac, Jason, Kabir, Laksh, Neel, Samar and
Ranbir are sitting in two parallel rows. Each row has six seats. Ten people
are sitting on ten seats and there are two vacant seats. Therefore in the
given seating arrangement each member seated in a row faces another
member of the other row. In Row-1, all are facing south and in Row-2 all
are facing north direction. In each row at least one vacant seat is there.
Each of them likes different food namely Almond, Bread, Corn, Fish,
Gumbo, Hot Dogs, Moose, Ostrich, Reuben and Toast. All the above
information is not necessarily in the same order.

Neel sits second to the left of Isaac, who sits opposite to the one who like
Reuben. The one who like Corn sits second to the right of the one who likes
Fish. Jason sits third to the right of the one who likes Toast. The one who
likes Almond sits immediate right of Isaac. Laksh sits second to the right of
vacant seat of Row-1. Ranbir sits opposite to the one who is an immediate
neighbour of Farhan. Two persons sit between the one who likes Reuben
and the one who likes Moose, who sits at extreme end. Vacant seats are
not at the extreme end. The one who likes Fish is sitting at extreme end.
The one who likes Corn and the one who likes Ostrich sit opposite to each
other. The one who likes Toast and the one who likes Gumbo sit opposite
to each other. Samar sits second to the left of Kabir and does not like either
Reuben or Gumbo. The one who likes Fish and the one who likes Bread sit
diagonally opposite to each other. Only one person sits between Amol and
Ranbir.

245. Who sits second to the left of the one who likes Ostrich?

A. Isaac B. Ranbir C. Laksh

D. Neel E. Samar

246. How many persons sit between the one who like Gumbo and the one
who sit opposite to Jason?
A. One B. Two C. Three

D. Either Two or Three E. None

247. Who sits second to the right of the one who sits opposite to the one
who like Almond?

A. Amol B. Neel C. Farhan

D. Kabir E. Vacant seat

248. Which of the combinations of persons are seated in same row?

A. Jason, Amol, Isaac B. Ranbir, Amol, Laksh C. Isaac, Darsh, Samar

D. Laksh, Neel, Amol E. Isaac, Darsh, Farhan

249. Four of the following five are alike in a certain way and hence form a
group. Which of the following does not belong to the group?

A. Farhan and the one who like Toast

B. Isaac and the one who like Ostrich

C. Laksh and the one who like Bread

D. Kabir and the one who like Almond

E. Jason and the one who like Corn


Correct answers:
1 B 31 D 61 B 91 D 121 C 151 A 181 E 211 E 241 A
2 A 32 E 62 D 92 A 122 D 152 B 182 C 212 D 242 E
3 C 33 C 63 A 93 C 123 A 153 B 183 D 213 B 243 E
4 D 34 A 64 D 94 B 124 E 154 E 184 E 214 B 244 B
5 C 35 E 65 C 95 E 125 B 155 D 185 C 215 D 245 B
6 C 36 D 66 E 96 D 126 E 156 A 186 B 216 A 246 A
7 C 37 B 67 A 97 E 127 B 157 B 187 D 217 C 247 D
8 E 38 A 68 A 98 B 128 D 158 C 188 D 218 E 248 E
9 D 39 C 69 C 99 D 129 A 159 D 189 A 219 B 249 C
10 C 40 D 70 B 100 C 130 B 160 B 190 D 220 D
11 D 41 E 71 A 101 A 131 C 161 C 191 B 221 D
12 E 42 A 72 B 102 C 132 B 162 D 192 E 222 B
13 C 43 C 73 B 103 E 133 D 163 B 193 A 223 D
14 B 44 C 74 C 104 C 134 C 164 A 194 E 224 E
15 A 45 E 75 D 105 B 135 D 165 A 195 B 225 D
16 B 46 D 76 C 106 C 136 B 166 C 196 E 226 A
17 E 47 B 77 D 107 B 137 D 167 B 197 A 227 C
18 A 48 A 78 B 108 D 138 A 168 D 198 B 228 C
19 C 49 C 79 A 109 B 139 B 169 D 199 E 229 A
20 D 50 E 80 A 110 E 140 C 170 B 200 B 230 D
21 C 51 B 81 C 111 C 141 C 171 B 201 E 231 B
22 C 52 C 82 B 112 D 142 D 172 D 202 A 232 C
23 E 53 D 83 D 113 B 143 C 173 C 203 C 233 A
24 A 54 B 84 A 114 E 144 B 174 E 204 B 234 E
25 D 55 E 85 E 115 C 145 D 175 D 205 A 235 A
26 E 56 A 86 C 116 B 146 B 176 A 206 E 236 A
27 A 57 D 87 A 117 C 147 C 177 B 207 A 237 D
28 E 58 B 88 A 118 C 148 C 178 C 208 B 238 B
29 C 59 C 89 D 119 E 149 C 179 B 209 B 239 B
30 B 60 D 90 B 120 B 150 D 180 D 210 E 240 C
| Explanations |

Set-1

Common explanation

Reference:

It is known that Cheenu sits exactly in the middle and there is no person to
the right of Ishita.

Inference:

There is no one sitting to the right of Ishita, means she is sitting at the right
most end of the row.

Reference:

Dona is not sitting adjacent to either Cheenu or Ishita.

Inference:

Reference:

Dona is fourth to the right of Faria.

Inference:

As Dona cannot sit adjacent to Cheenu and Ishita, so, therefore, there is
only place left i.e. third from the right end of the row for Faria.
Reference:

Gaurav and Harish are sitting next to each other. Harish doesn't sit at any
extreme corner.

Inference:

Reference:

Esha is the neighbor of Dona but not of Cheenu.

Inference:

Reference:

Anu is second to the right of Harish.

Inference:

Now, only Bablu is left to be placed at the only left position.

Final image:
Answers:

1.

From the following explanation we get “Anu is sitting to the immediate left
of Cheenu”.

Option B, is hence the correct answer.

2.

From the following explanation we get “Dona is sitting between Bablu and
Esha”.

Option A, is hence the correct answer.

3.

From the following explanation we get “Faria and Gaurav do not belong to
that group”.

Option C, is hence the correct answer.

4.

From the following explanation we get “Cheenu sits third to the right of
Harish”.

Option D, is hence the correct answer.

5.

From the following explanation we get “Gaurav is seated at the left most
corner of the row”.

Option C, is hence the correct answer.


Set-2

Common explanation

Reference:

N is an immediate neighbor of both L and R, who is an immediate


neighbour of P.

P sits second to the left of N.

Inference:

Reference:

Only two persons sit between P and S.

Inference:

Reference:

Only one person sits between M and Q.


Inference:

Answers:

6.

O is Second to the right of S.

Option C, is hence the correct answer.

7.

NM does not belong to the group.

Option C, is hence the correct answer.

8.

M sits second to the left of the one who is on the immediate right of O.

Option B, is hence the correct answer.

9.

In the following Common explanation it is clear that S sits between M and


Q.

Hence, option D is correct.


10.

In the following Common explanation it is clear that R sits third to right of


the one who sits opposite of N?

Hence, option C is correct.


Set-3

Common explanation

Reference:

Qiya is second to the left of Rati who is fourth from the right corner.

Tiya sits immediate right of Qiya.

There are only two persons between Rati and Pari.

Inference:

Based on the given hints following two cases can be drawn:

Case1 When Pari is third to the left of Rati

Row/Persons→
Rati Tiya Qiya Pari
(Facing South)↓

←Right end Left end→

Case2 When Pari is third to the right of Rati

Row/Persons→
Pari Rati Tiya Qiya
(Facing South)↓

←Right end Left end→

Reference:

The number of persons between Qiya and Mita is same as the number of
persons between Pari and Shri.

Mita sits at any corner of the row.

Om is not a neighbor of Shri who is immediate left of Niya.


Inference:

Case1 When Pari is third to the left of Rati and number of persons between
Mita and Qiya are 4

Row/Persons→
Mita Shri Rati Tiya Qiya Pari
(Facing South)↓

←Right end Left end→

In Case1, Positions of Om and Niya cannot be ascertained without violating


the third hint, thus Case1 Fails here.

Case2 When Pari is third to the right of Rati and number of persons
between Mita and Qiya is 1

Row/Persons→
Pari Niya Shri Rati Tiya Qiya Om Mita
(Facing South)↓

←Right end Left end→

Answers:

11.

From the following explanation it is clear that six persons are seated
towards the left of Niya.

Option D, is hence the correct answer.

12.

From the following explanation it is clear that Pari is the only one among
the given options, who sits at an extreme end, rest person sit in the
middle of the row.

Option E, is hence the correct answer.


13.

From the following explanation it is clear that Om and Tiya both are the
immediate neighbours of Qiya.

Option C, is hence the correct answer.

14.

From the following explanation it is clear that the person sitting third
from the right end is Shri and second to the left of Shri is Tiya.

Option B, is hence the correct answer.

15.

From the following explanation it is clear that Om is sitting third to the


left of Rati.

Option A, is hence the correct answer.


Set-4

Common explanation

Reference:

Eight persons – Amar, Ansh, Arun, Amit, Ashu, Anup, Arif and Anil, are
sitting around a circular table facing the centre but not necessarily in the
same order.

Inference:

We will keep this information in mind while solving the puzzle.

Reference:

Anup is to the immediate left of Arun.

Amit is between Anup and Ansh.

Amar is the neighbour of Ansh.

Inference:

After using the above hints, we can draw a following circular arrangement:

Reference:

Arif is to the immediate left of Ashu.

Arif is not the neighbour of Ansh or Anil.


Inference:

After using the above hints, we have:

Answers:

16.

Following the final solution, we can say that Anup is sitting third to right of
Amar.

Hence, the correct answer is option B.

17.

Following the final solution, we can say that four persons were sitting
between Amit and Anil, when counted from the left of Anil.

Hence, the correct answer is option E.

18.

Following the final solution, we can say that Arif is third to the right of Amit.

Hence, the correct answer is option A.

19.

In the following common it is clear that Amar sits opposite to the one who
sits third to the left of Anil.
Hence, the correct answer is option C.

20.

In the following Common explanation it is clear that Arun and Ashu are
sitting adjecent of Arif.

Hence, the correct answer is option D.


Set-5

Common explanation

References:

J likes Banana.

The person J is the immediate neighbor of the one who like Brinjal and O.

K sits third to the right of O.

One person sits between K and Q.

One who likes Beans sits third to the left of Q.

Inferences:

From above statements,

J likes Banana i.e. J likes fruits and sits at the corner of the table.

The person J is the immediate neighbor of the one who like Brinjal and O.
Here we get two possibilities as shown in seating.

Remaining statements are given directly; by using we get 3 initial cases as


follows

Case: 1
Case: 2

Case: 1-A

References:

One who sits second to the right of the one who likes Beans is facing N.

One who likes Cabbage sits second to the right of the one who likes
Cucumber.

One who likes Beans and one who likes Mango are not immediate
neighbors.

One who likes Mango sits third to the right of M.

Inferences:

From above statements,

One who sits second to the right of the one who likes Beans is facing N.
With respect to the position of the one who likes Beans, N can be located in
each of three cases as shown below.
One who likes Cabbage sits second to the right of the one who likes
Cucumber i.e. both are vegetables. So those persons are sitting in the
middle of the table. Already Beans and Brinjal is located in all the 3 cases.
So we get only one possibility to place Cabbage and Cucumber as per
statement.

One who likes Beans and one who likes Mango are not immediate
neighbors. One who likes Mango sits third to the right of M.

Here we get only one possibility in each case as per above statement.

Note: In Case 1-A and case-2, K doesn’t like Mango; if so then there is no
place for M (O is occupied already in both cases) as per given statement.

By using above information, we get the following seating as shown

Case: 1

Case: 2
Case: 1-A

References:

There are as many persons sit between P and M as between P and L.

L and one who like Apple are not immediate neighbors.

Inferences:

From above statements,

Case-1: P sits to the immediate left of M and L sits to the immediate right of
P i.e. No one sits between P and M & no one sits between P and L. As per
given statement, there is no place for Apple (L and one who like Apple are
not immediate neighbors). Hence this case become invalid and it can be
eliminated.

Case-1-A: Here, the 1st reference point (There are as many persons sit
between P and M as between P and L doesn’t get satisfied). Hence this case
become invalid and it can be eliminated.

Case: 1 [Eliminated] – No Place for Apple


Case: 1-A [Eliminated] There are as many persons sit between P and M as
betwen P and L.

Case-2: Here, P sits to the immediate left of O and L sits to the immediate
left of K. By this two persons sit between P &M and also P & L (given
condition satisfied). Given, L and one who like Apple are not immediate
neighbors. Therefore P likes Apple and K likes Grapes (only possibility).
Thus we get the completed seating as shown below.
Answers:

21.

Following the Common explanation, we get "K-Grapes",

Hence, option C is correct.

22.

Following the Common explanation, we get "The one who like Cucumber
i.e. L",

L sit opposite to O. Hence, option C is correct.

23.

Following the Common explanation, we get "None is true",

Hence, option E is correct.

24.

Following the Common explanation, we get "Third to the right",

N sits third to the right of the one who like Banana

Hence, option A is correct.

25.

Following the Common explanation, we get "M-sit in the middle of the


sides",

Remaining 4 persons sit at the corner of the table.

Hence, option D is correct.


Set-6

Common explanation

References:

There are three persons sit between Tanish and Jiral. Jiral sits third to the
left of Vaibhav.

Bhavin does not sit third to the left of Tanish who doesn’t sit at the corner.

One person sits between Jiral and Kiansh.

Rithwik and Kiansh are not immediate neighbors.

Inferences:

From above statements,

o Tanish does not sit at corner of the table, and then he must sit in the
middle of the table.

Based on the remaining statements we get the following cases as follows

Case: 1 Case: 2

References:

Aadvik is immediate neighbor of Kiansh.

Bhavin does not sit third to the left of Tanish who doesn’t sit at the corner.
Bhavin sits fourth to the right of Yashith.

Inferences:

From above statements, we get the following case,

Case: 1 [Eliminated] No place:


Bhavin sits fourth to the right
of Yashith.

Case: 2

Finally, Rithwik sits immediate right of Yashith. Thus we get the


completed seating arrangement as follows,
Answers:

26.

The following Common explanation, we get "Rithwik sit in the middle of the
table".

Remaining 4 persons sit at corner of the table.

Hence, option E is correct.

27.

The following Common explanation, we get "Rithwik".

Relation: Tanish is second to the left of Kiansh.

Bhavin is second to the left of Aadvik similarly

Jiral is second to the left of Rithwik

Hence, option A is correct.

28.

The following Common explanation, we get "Only Vaibhav".

Fifth to the right of Kiansh is Vaibhav and immediate right of Rithwik is also
Vaibhav.

Hence, option E is correct.

29.

In the following Common explanation it is clear that Aadvik sits between


Jiral and Kiansh.

Hence, option C is correct.


30.

In the following Common explanation it is clear that Jiral sits opposite to


one who sits second to the right of Rithwik.

Hence, option B is correct.


Set-7

Common explanation

Reference:

Hina sits second to right of Mera.

Hina and Asha have three persons between them.

Inference:

Here, we have two possible scenarios in which above hints can be used
accordingly.

Case 1:

Case 2:

Reference:

No one sits between Asha and Fany.

Desh sits fifth to left of Fany.

Inference:

At this point, we cannot fix the position of Desh and Fany according to the
above hints in case 1 so we can say that Case 1 is an invalid case.
Here, we have two possible scenarios in which above hints can be used in
case 2 accordingly.

Case 2-A:

Case 2-B:

Reference:

Desh and Rani have two persons between them.

Pinu is second to right of Rani.

Inference:

After using the above hints, we have:

If we fix the position of Rani on the right of Desh then we won't be able to
fix the position of Pinu second to the right of Rani. Then,

Case 2-A:

Case 2-B:
Reference:

Number of persons towards right of Asha is one less than the number of
persons sitting between Pinu and Asha.

Number of persons towards left of Rani is equal to the number of persons


sitting between Fany and Hina.

Inference:

At this point we can see that our case 2-B is contradicting with the above
hints so we can say that case 2-B is an invalid case.

Case 2-A:

Answers:

31.

Following the final solution, we can say that 19 persons were sitting in the
row.

Hence, the correct answer is option D.

32.

Following the final solution, we can say that six persons were sitting
between Pinu and Asha.

Hence, the correct answer is option E.

33.
Following the final solution, we can say that Fany was fifth to the left of
Hina.

Hence, the correct answer is option C.

34.

Following the final solution, we can say that number of persons that were
sitting on the right of Hina and on the left of Rani were 1 and 4 respectively.

Required Sum = 1 + 4 = 5

Hence, the correct answer is option A.

35.

Following the final solution and applying the given conditions, we have:

Here, four persons are sitting between Riya and Rani.

Hence, the correct answer is option E.


Set-8

Common explanation:

Reference:

Usha sits at one of the extreme ends of the line.

Mona sits third to the left of Usha.

Pinu sits third to the right of Mona.

Inference:

Here, we have two possible scenarios in which above references can be


used accordingly.

Case 1:

Case 2:

Reference:

Joya sits to the immediate right of Pinu.

Joya does not sit at any of the extreme end of the line.

Pinu sits second to the left of Chaya.

Inference:
After using the above references, we have:
Case 1:

Case 2:

Reference:

Tara sits second to the right of Joya.

Only one person sits between Chaya and Lali.

Tara faces the same direction as Joya.

Inference:

After using the above references, we have:

Case 1:

Case 2:

Reference:
Doly faces north.

Doly faces the same direction as Chaya.

Both the immediate neighbours of Mona face the opposite direction.

Inference:

At this point, we cannot use the second hint (Chaya faces south) in case 1
accordingly so we can say that case 1 is an invalid case.

Case 2:

Answers:

36.

Following the final solution, we can say that Joya is sitting second to the
right of Mona.

Hence, the correct answer is option D.

37.

Following the final solution, we can say that Usha is the one who does not
belong to the group because amongst all the persons given in the options,
Usha is the only one who faces North direction.

Hence, the correct answer is option B.

38.
Following the final solution, we can say that three persons sit between Joya
and Doly.

Hence, the correct answer is option A.

39.

Following the final solution, we can say that four persons are facing north
direction.

Hence, the correct answer is option C.

40.

Following the final solution, we can say that no one is sitting third to the
left of Lali.

Hence, the correct answer is option D.


Set-9

Common explanation

References:

Six persons A to F are sitting in the Row-1 and facing north direction.

Six persons P to U are sitting in the Row-2 and facing south direction.

E sits second to the right of F.

Two persons sit between F and B, who is facing R.

More than one person sits between F and D.

Q and R are immediate neighbors.

U sits second to the left of Q.

Inferences:

From above statements

Note: B can’t sit at any end of the row; if so F sits either at 3rd or 4th from
end of the row by this more than 1 person between D and F is not possible
(Refer 1st three reference points)

With respect to the last two statements, we get the initial table as follows,

Case-1: F sits extreme left end (Row-1). E and B sits second and third to the
right of F respectively. Q sits opposite to E (only possibility) and R sits
opposite to B (given). U sits to the immediate left of R. Here D sits either
second or 1st from right end in Row-1

Case-1
Person[Row-2]
Q R U
South

Person[Row-1]
F E B D/ D/
North
Case-2: F sits second from left end. E and B sits second and third to the
right of F respectively. Q sits opposite to E (only possibility) and R sits
opposite to B (given). U sits to the immediate left of R. Here D sits at
extreme right end of Row-1 [only possibility] i.e. 3 persons sit between F
and D.

Case-2
Person[Row-2]
Q R U
South

Person[Row-1]
F E B D
North

References:

The person who sits second to the right of P is facing C.

T does not sit at the extreme end of the line.

Inferences:

From above statements,

Case-1: There is no place for P as per given statement (The person who sits
second to the right of P is facing C). This case become invalid and it can be
eliminated.

Case-1 [Eliminated]
The person who sits second to the
right of P is facing C [not satisfied]
Person[Row-2]
Q R U
South

Person[Row-1]
F E B D/ D/
North

Case-2: As per given statement, C sits extreme left of the Row-1. P sits to
the immediate right of Q in Row-2. Given, T does not sit at the extreme end
of the line. Then T sits to the immediate right of P in Row-2. Finally, S sits
extreme at right end in Row-2 and the person A sit to the immediate right
of F. Thus all given conditions gets satisfied and we get the completed
seating as shown.

Case-2
Person[Row-2]
S T P Q R U
South

Person[Row-1]
C F A E B D
North

Answers:

41.

Following the Common explanation, we get "None of these".

T sits third to the right of R

Hence, option E is correct.

42.

Following the Common explanation, we get "One".

Only one person sit between C and A

Hence, option A is correct.

43.

Following the Common explanation, we get "D faces Q".


Case-2
Person[Row-2]
S T P Q R U
South

Person[Row-1]
C F A E B D
North
Alphabetical order A B C D E F
Hence, option C is correct.

44.

Following the Common explanation, we get "Second person is sitting


second to left of the first person", So, E is sitting second to the left of F.

Hence, option C is correct.

45.
Following the Common explanation, we get "U is sitting diagonally opposite
to C ".

Hence, option E is correct.


Set-10

Common explanation

Reference:

Eight persons – Mona, Moti, Meera, Mali, Mitra, Mansi, Megha and Mrig,
were sitting in a circle facing towards the center. Each of them was born in
a different city — Shimla, Manali, Agra, Nanital, Pune, Goa, Mumbai and
Delhi, but not necessarily in the same order.

Inference:

We will keep this information in mind while solving the puzzle.

Reference:

Two persons were sitting between the one, who was born in Nanital and
Mitra.

Mali was born in Nanital.

Mitra was born in Manali.

Mona was born in Shimla and sits opposite to Mitra.

Inference:

Here, we have two possible scenarios in which the above hints can be used
accordingly.

Case 1: Case 2:
Reference:

The one, who was born in Delhi, sits adjacent to the one, who was born in
Nanital.

The one, who was born in Pune, sits opposite to Mali.

Mrig was born in Mumbai and sits second to the right of the one, who was
born in Nanital.

Inference:

After using the above hints, we have:

Case 1: Case 2:

Reference:

Meera was born in Pune and was an immediate neighbor of the one, who
was born in Goa.

Megha sits third right to Moti and Megha was born in Delhi.

Inference:

At this point we cannot use the above hints in case 2 accordingly so we can
say that case 2 is an invalid case.
Case 1:

At this point we can say that Moti was born in Agra and Mansi was born in
Goa.

Case 1:

Answers:

46.

Following the final solution, we can say that Mansi was born in Goa.

Hence, the correct answer is option D.

47.

Following the final solution, we can say that Meera was sitting third to the
right of Megha.

Hence, the correct answer is option B.


48.

Following the final solution, we can say that Moti was born in Agra.

Hence, the correct answer is option A.

49.

Following the final solution, we can say that the one who was born in
Mumbai was sitting in the front of Moti.

Hence, the correct answer is option C.

50.

Following the final solution, we can say that Meera was third to the left of
the one who was born in Shimla.

Hence, the correct answer is option E.


Set-11

Common explanation

Reference:
Eight persons – Ankit, Kavit, Manit, Vinit, Charit, Sumit, Ganit, and Ronit are
sitting in a straight line (but not necessarily in the same order) such that
three of them are facing south and rest of them are facing north. Each of
them works in a different bank – SBI, UCO, PNB, BOB, SEBI, RBI, BOI, and
NHB (but not necessarily in same order).

Inference:

We will keep this information in mind while solving the puzzle.

Reference:

Ronit sits third to the right of Manit.

Manit doesn’t face north direction.

Four persons sit between Ronit and Charit, who works in RBI.

Inference:

After using the above hints, we have:

Reference:

Ganit is an immediate neighbor of Manit and Charit both.

Manit sits on the immediate left of Kavit.


Inference:

After using the above hints, we have:

Reference:

Kavit sits third from one of the extreme end of the row.

Vinit sits second to the right of Ganit.

Inference:

After using the above hints, we have:

Reference:

The one, who is sitting on the immediate right of Vinit, works in NHB.

Sumit sits right of Manit, who works in BOI.

Ganit and Charit face opposite direction to each other.

Inference:

After using the above hints, we have:


As we know that only three of these persons are facing towards south
direction so we can find out the direction faced by the rest of these
persons.

Reference:

The one who works in SBI sits at an extreme end.

The one who works in BOB sits between Charit and Ankit.

The one who works in PNB sits immediate right of the one who works in
UCO.

Sumit does not work in PNB.

Inference:

After using the above hints, we have:

Here, we can say that Ganit works in SEBI.

Answers:

51.
Following the final solution, we can say that Kavit works in PNB.

Hence, the correct answer is option B.

52.

Following the final solution, we can say that Vinit – BOB – North is the
correct combination.

Hence, the correct answer is option C.

53.

Following the final solution, we can say that three persons sit between
Charit and Sumit.

Hence, the correct answer is option D.

54.

Following the final solution, we can say that the one who works in BOB is
fourth to the left the one who works in PNB.

Hence, the correct answer is option B.

55.

Following the final solution, we can say that Ganit works in SEBI.

Hence, the correct answer is option E.


Set-12

Common explanation
Reference:
Eight persons – Shirish, Suresh, Sajal, Sashi, Sanjay, Sunder, Srajit and
Shankar (not necessarily in the same order) are sitting around a circular
table in a way such that four of them are facing towards the centre and the
others are facing away from the centre. Each of these persons have a
different surname – Gaur, Gail, Gopal, Gandhi, Girish, Guman, Garv and
Goyal, but not necessarily in the same order.
Inference:
We will keep this information in mind while solving the puzzle.
Reference:
Sajal’s surname is Gaur and is facing towards the centre.
The immediate neighbours of Sajal faces away from the centre and their
surnames are Gail and Gopal.
Srajit’s surname is Garv, is facing towards the centre and sits third to the
right of Sashi, whose surname is Gail.
Inference:
Here, we have two possible scenarios in which above hints can be used
accordingly.
Case 1: Case 2:
Reference:

The one whose surname is Girish is not neighbouring the one whose
surname is Gail or Srajit.

Shirish is sitting opposite to Sanjay and neither of them has a surname


Gopal but surname of one of them is Girish.

Inference:

At this point we cannot use the above hints in case 2 accordingly so we can
say that case 2 is an invalid case.

Here, we have two possible scenarios in which above hints can be used in
case 1 accordingly.

Case 1-A: Case 1-B:

Reference:

Shankar is not neighbouring either the one whose surname is Garv or the
one who is neighbouring the one whose surname is Guman but is sitting
opposite to the one whose surname is Gandhi, who is neighbouring Shirish.

Inference:

At this point we cannot use the above hints in case 1-B accordingly so we
can say that case 1-B is an invalid case.
After using the above hints in case 1-A, we have:

Case 1-A:

Reference:

Sunder’s surname is not Goyal and is facing the centre.

Both Shirish and Sanjay are facing in the same direction (for example, if one
is facing outside then other is also facing outside).

Shankar faces away from the centre and the immediate neighbours of
Shankar faces a direction opposite to each other.

Inference:

After using the above hints in case 1-A, we have:

Case 1-A:
At this point we can say that Suresh’s surname is Goyal and faces the
centre.

Case 1-A:

Answers:

56.

Following the final solution, we can say that Suresh has surname Goyal.

Hence, the correct answer is option A.

57.

Following the final solution, we can say that Sashi sits second to the right of
Shankar.

Hence, the correct answer is option D.

58.

Following the final solution, we can say that Shirish is sitting third to the
right of the one, who is sitting second to the left of the one, whose
surname is Gandhi.

Hence, the correct answer is option B.


59.

Following the final solution, we can say that Sashi Gaur is the one that does
not belong to the group because of all the options Sashi Gaur is the only
option in which name and surname is of adjacent persons.

Hence, the correct answer is option C.

60.

Following the final solution, we can say that both Srajit and Sanjay are the
neighbours of Suresh.

Hence, the correct answer is option D.


Set-13

Common explanation

Reference:

Seven persons – Naina, Shalu, Amar, Rohan, Doshi, Brij and Vinod, are
sitting on a bench in a church, facing south. Each of them has a different
profession – Manager, Engineer, Boxer, Politician, Singer, Painter and Actor.

Inference:

We will keep this information in mind while solving the puzzle.

Reference:

Vinod sits third from left end.

Amar sits second to the right of Vinod.

There is one person sitting between Amar and Rohan.

Inference:

After using the above hints, we can draw a following linear arrangement:

Reference:

Naina is an immediate neighbour of Doshi, who sits at an extreme end.

The one who is a Singer sits at an extreme end and immediate neighbor of
Shalu.

Inference:
After using the above hints, we have:

Reference:

The one who is an Actor sits second to the left of Rohan.

The one who is an Engineer sits third to the left of Shalu, who is a Painter.

Inference:

At this point, we can easily fix the position of Brij in the row.

Reference:

Naina is neither Manager nor Boxer.

Brij is not a Boxer.

Inference:

After using the above hints, we have:

Here, we can say that Naina is the Politician.


Answers:

61.

Following the final solution, we can say that Naina is a Politician.

Hence, the correct answer is option B.

62.

Following the final solution, we can say that the one who sits in the
middle of the row is a Manager.

Hence, the correct answer is option D.

63.

Following the final solution, we can say that Naina sits third to the left of
the one who is an Actor.

Hence, the correct answer is option A.

64.

Following the final solution, we can say that three persons sit between
Shalu and Naina.

Hence, the correct answer is option D.

65.

Following the final solution, we can say that Amar – Actor is the correct
combination.

Hence, the correct answer is option C.


Set-14

Common explanation

References:

Herat and Himan sit opposite to each other but does not face each other.

Herat sits third to the right of Hetav.

Inferences:

From above statements,

Herat and Himan sit opposite to each other but does not face each other
i.e. Herat and Himan are sitting at the corner of the table and faces away
from the centre.

By using above information, we get the initial seating as follows

References:

Herin and Hetav are facing same direction.

Herin is an immediate neighbor of the one who likes Barone.

Hiyan likes Perk and sits second to the right of the one, who likes Barone.

Inferences:

From above statements,


Hetav faces inside the table. Therefore, Herin also faces inside the table.
Thus we get 3 possibilities from above seating as shown below with respect
to other reference points.

Case-1: Here, Herin sits to the immediate right of Herat. Given, Herin is an
immediate neighbor of the one who likes Barone. If the one who likes
Barone sits to the immediate left of Herin, then there is no place for Hiyan
(Given, Hiyan likes Perk, sits second to the right of the one, who likes
Barone). Therefore, as per seating Herat likes Barone and Hiyan (likes Perk)
sits to the immediate right of Herin to satisfy the given statement.

Case-2: Here, Herin sits to the immediate right of Himan. Given, Herin is an
immediate neighbor of the one who likes Barone. If the one who likes
Barone sits to the immediate left of Herin, then there is no place for Hiyan
(Given, Hiyan likes Perk, sits second to the right of the one, who likes
Barone). Therefore, as per seating Himan likes Barone and Hiyan (likes
Perk) sits to the immediate left of Herin to satisfy the given statement.

Case-3: Here, Herin sits to the immediate left of Herat. Given, Herin is an
immediate neighbor of the one who likes Barone. If the one who likes
Barone sits to the immediate right of Herin, then there is no place for Hiyan
(Given, Hiyan likes Perk, sits second to the right of the one, who likes
Barone). Therefore, as per seating Herat likes Barone and Hiyan (likes Perk)
sits to the immediate right of Hetav to satisfy the given statement.

References:
The one who likes Munch and the one who likes Milky Bar are facing each
other.
Henik sits to the immediate right of the one who likes Milky Bar and to the
immediate left of the one who like 5 star chocolates.

Inferences:

From above statements,


The one who likes Munch and the one who likes Milky Bar are facing each
other i.e. both are sitting in the middle of the table.
With respect to the second reference point, we get only one possibility (to
locate for the one who like Milky bar) in each case as shown below.
Case: 1 Case: 2
Case: 3

References:

Hiran sits third to the right of the one who likes Dairy Milk.
The one who likes Bournville sits third to the left of the one who likes
Kitkat.
One who sits to the immediate right of Herat does not like Bournville.

Inferences:

From above statements,


Case-1: Given, Hiran sits third to the right of the one who likes Dairy Milk.
Here, Hiran likes Milky Bar and Himan likes Dairy Milk (Only possibility to
satisfy this statement). There is no place for the one who likes Kitkat as per
the given statement. Hence this case become invalid and it can be
eliminated.
Case: 1 [Eliminated] – No place for the one who like Kitkat.
Case-2: Here, there is no place for Hiran i.e. given, Hiran sits third to the
right of the one who likes Dairy Milk. Hence this case become invalid and it
can be eliminated.

Case: 2 [Eliminated] – No place for Hiran

Case-3: Given, Hiran sits third to the right of the one who likes Dairy Milk.
Here, Henik likes Dairy Milk and Hiran sits to the immediate right of Herat
(Only possibility to satisfy this statement). With respect to second and third
reference point, Himan likes Bournville and Hiran likes Kitkat (only
possibility). Finally, Hevin likes 5 star chocolates and sits to the immediate
left of Henik. Thus we get the completed seating as shown below.

Case: 3

Answers:

66.

The following Common explanation, we get "None of these".


Herin (likes Milky Bar) sits second to the right of Hiran, who likes Kitkat

So answer is both A and D

Hence, option E is correct.

67.

The following Common explanation, we get "Hevin-5 star chocolates".

Hence, option A is correct.

68.

The following Common explanation, we get "Himan-Bournville chocolates".

Hence, option A is correct.

69.

The following Common explanation, we get "Hiran and the one who like 5
stars (Hevin)".

Hence, option C is correct.

70.

The following Common explanation, we get "Hiran sits third to the right of
Henik and Hiran likes Kitkat".

Hence, option B is correct.


Set-15

Common explanation

References

O, P, S, U, V and Z are seated on row 1 & Q, R, T, W; X and Y are seated in


row 2.

O sits second from the left of S who is not facing north.

P sits immediate right of S. P sits straight to T facing opposite direction to


each other.

V sits second to the right of P.

O and R face each other. Y sits second to the left of R.

W and U seated are at corners and facing opposite direction to each other.

Inferences

From above statements, we get

Note: In each row persons may face north/south.

We don’t know the position of S from extreme ends. So we solve as per


the statements without considering number of persons in row as follows,

i. S faces south and his/her immediate right is P.

ii. O sits second from the left of S. V sits second to the right of P.

iii. Note: if P faces south, then V and O are sit at extreme ends of row-1.
Therefore no place for U to sit at corner. Thus P must face north direction.
Then V sits immediate left of S.
iv. P must face north direction sits opposite to T who faces south direction.
[opposite direction to each other means, if one faces north then the other
faces south or vice versa]

v. O and R face each other. Y sits second to the left of R. [If R faces south, T
and Y sit at extreme ends of row-2. Therefore no place for W to sit at
corner. Thus R faces north direction and O faces south direction. Both are
facing each other. Therefore Row-2 faces Row-1 as shown below.

vi. Y sits second to the left of R. Thus we get the table as follows,

Direction North South South


Person(Row-1) P S V O

Person (Row-2) T Y R
Direction South North

References

Neither Q nor X is sit at the extreme ends of the row. Q is not a neighbor of
R.

Two persons sit between Q and X.

V is facing same direction as R.

W and U are seated at corners and facing opposite direction to each other.

Z and U are facing same direction as Q and X.

Not more than five persons facing south.

Inferences

Note: Clearly from above diagram (we placed only 4 seating in each row), Q
is not sit at any ends and not neighbor to R. If Q sits second to the right of
R, then T and Q sit at extreme ends. Then there is no place for W to sit at
corner in row 2 which is not possible. Thus Q must sit immediate right of T
and X sits immediate left of R. Only person in row 2 is W who sits extreme
corner and faces the person U.

i. V is facing same direction as R. R faces north, then V also faces north


direction.

ii. Only remaining person in row-1 is Z who sits immediate left of P.

iii. Note: [Given: Z, U, Q and X are facing same direction but none of them
direction is given directly]

iv. Hint: Not more than five persons facing south. [If Z, U, Q and X are
facing south direction, total persons facing south direction becomes 7
including T, S and O which is not possible.

v. Therefore Z, U, Q and X are facing north direction

vi. W and U are seated at corners and facing opposite direction to each
other. [U faces north, and then W faces South direction.

vii. Y direction is unknown. He/she may face North/South direction.

Thus we get the completed seating as shown below,

Direction North North North South North South


Person (Row-1) U Z P S V O

Person (Row-2) W Q T Y X R
Direction South North South North/South North North

Answers:

71.

The following Common explanation, we get "S and O”.


Hence, option A is correct.

72.

The following Common explanation, we get "Both the persons faces North
“.

Hence, option B is correct.

73.

The following Common explanation, we get "V and X”.

Immediate right of O is V & Third to the right of Q is X.

Hence, option B is correct.

74.

Following Common explanation, we get "Y”.

Except Y, the directions of all the others are confirmed.

Hence, option C is correct.

75.

Following Common explanation, we get "Q”.

P is seated second to the right of U and in same way Z is seated second to


the right of S and in same way Y is seated second to the right of Q.

Hence, option D is correct.


Set-16

Common explanation

Reference:

Eight persons – Navya, Farukh, Ankur, Mangal, Ravan, Danav, Simar, and
Gopal are seated around a circular table such that only three of them are
facing away from the centre and the rest are facing towards the centre (not
necessarily in the same order).

Inference:

We will keep this information in mind while solving the puzzle.

Reference:

Navya sits second to the left of Ankur.

Farukh is not an immediate neighbour of both Ankur and Navya.

Farukh doesn’t sit opposite to both Ankur and Navya.

Inference:

Here, we have two possible scenarios in which we can use the above
information accordingly.

Case 1: Case 2:

.
Reference:

Gopal sits third to the left of Navya.

Danav sits second to the left of Gopal.

One of the immediate neighbours of Ankur faces opposite direction of


Ankur.

Inference:

After using the above references, we have:

Case 1: Case 2:

Reference:

Mangal is not an immediate neighbour of Farukh.

Simar sits third to the left of Mangal.

Inference:

After using the above references, we have:

Case 1: Case 2:
Reference:

Ravan and Farukh face the same direction as Mangal faces.

Inference:

At this point we cannot fix the position of Ravan and Farukh facing same
direction as Mangal in case 2 according to the given hints so we can say
that case 2 is an invalid case.

After using the above hints case 1 can be redrawn as:

Case 1:

Answers:

76.

Following the final solution we can say that Mangal sits second to the right
of Danav.

Hence, the correct answer is option C.

77.

Following the final solution we can say that Ankur sits in front of Ravan.

Hence, the correct answer is option D.

78.
Following the final solution and applying the given conditions, we get:

Here, we can see that position of two persons is unchanged including


Ankur.

Hence, the correct answer is option B.

79.

Following the final solution we can say that Mangal sits third to the left of
Simar.

Hence, the correct answer is option A.

80.

Following the final solution we can say that only one person sit between
Ravan and Navya when counted from the right of Navya.

Hence, the correct answer is option A.


Set-17

Common explanation

Reference:

A row of persons facing north direction consist of equal number of boys


and girls such that boys are sitting at the extreme ends. All the boys except
one sit at even positions and all the girls except one sit at odd positions.
Extreme left end of the row is positioned as 1 and the place immediate next
to it as 2 and so on.

Inference:

We will make a mental note of these informations while solving the puzzle.

Reference:

 There is no one to the left of A who is fourth to the left of J who is on


immediate left of W.
 H who is not a boy is sitting adjacent to A and X.
 P sits at an extreme end and is third to the right of F who is a girl.
 Only two persons are between W and F.
 neighbours and T is to the immedT and O are immediateiate left of P
with T being a girl

Inference:

 With the hint that No one is to the left of A, it is clear that A sits at
position no. 1 i.e. extreme left end and is the boy that sits at odd
number position. Rest of the boys will sit at even numbered places.
 The second hint thus confirms P to be sitting at extreme right end and
he is also a boy. Thus the position at extreme left end must be an even
number.
 Using all the hints we can identify that there are 12 persons in the row,
out of which 6 must be boys and 6 must be girls.
 With the hint that H is a girl and is adjacent to X, position of H can be
fixed as between A and X. Thus H sits at 2nd number and X at 3rd
number.
 Now we have assigned all the odd numbered places to girls except one
and thus occupied six places for girls and 6 for boys.

Reference:

 Only three boys are there to the right of U, who is a girl.


 Z is fourth to the right of B who is on immediate right of X.

Inference:

Thus position of B can be fixed as between X and J and U is thus seated as in


between U and F.

Answers:

81.

From the following explanation it is clear that H is the only girl that sits on
an even numbered position.

Hence option C is correct.

82.

From the following explanation it is clear that three girls sit between X and
O.

Hence option B is correct.


83.

From the following explanation it is clear that J is an immediate neighbour


of B and W.

Hence option D is correct.

84.

From the following explanation it is clear that there are 6 boys and 6 girls in
the row.

Hence option A is correct.

85.

From the following explanation it is clear that J is the only girl among the
five given options. Thus the odd one out.

Hence option E is correct.


Set-18

Common explanation

Reference:
Eight persons – Heer, Rani, Nora, Anup, Doha, Mrig, Gaur and Shera are
sitting around a square table such that four persons sits on the fours corner
the table facing towards the center and the rest of the four persons sits on
the middle of the sides facing away from the centre.

Inference:

We will keep this information in mind while solving the puzzle.

Reference:

Nora sits third to the left of Rani.

Only two persons sit between Mrig and Shera, who sits immediate left of
Nora.

Inference:

Here, we have two possible scenarios, in which we can use the above hints
accordingly.

Case 1: Case 2:

Here, we will make a mental note that only two persons sit between Mrig
and Shera.
Reference:

Gaur neither sits at corner nor opposite to Shera.

Anup sits second to the left of Heer, who sits third left of Gaur.

Inference:

After using the above hints, we have:

Case 1: Case 2:

Initially, we have a hint that only two persons sit between Mrig and Shera
at this point we cannot fix the position of Mrig according the hint in case 1.
So, we can say that case 1 is an invalid case.

Case 2:

At this point we can easily fix the position of Doha in case 2. Then,
Case 2:

Answers:

86.

Following the final solution we can say that Shera sits in front of Anup.

Hence, the correct answer is option C.

87.

Following the final solution we can say that only one person sit between
Rani and Shera, when counted from the right of Shera.

Hence, the correct answer is option A.

88.

Following the final solution and applying the given condition we have:
Here, we can see that position of only persons i.e. Rani is unchanged
excluding Anup.

Hence, the correct answer is option A.

89.

Following the final solution we can say that Anup sits third to the left of
Doha.

Hence, the correct answer is option D.

90.

Following the final solution we can say that Gaur sits second to the right of
Mrig.

Hence, the correct answer is option B.


Set-19

Common explanation

Reference:

The persons sitting on bench 1 are facing north and those on bench 2 are
facing south such that each person of both the benches faces each other.

Inference:

On the basis of above hint we can infer the arrangement in which benches
are situated.

Reference:

Gayu faces Hina, who does not sit at any of the extreme ends.

Hina sits second to the right of Dia.

Inference:

Here we can have following two cases.

Case 1 - When Gayu sits on bench 2.

Case 2- When Gayu sits on bench 1.


Reference:

Bani faces the person who is second to the right of Ahana.

Ela and Bani neither sits in the same row nor faces each other.
Inference:

Note- We cannot place Bani on Bench 2 in Case 1, because in that case


second hint will get violate i.e. Ela and Bani will face each other.

Note- We cannot place Bani on Bench 1 in Case 2, because in that case


second hint will get violate i.e. Ela and Bani will face each other.

Reference:

Fiza sits in the same row in which Ela sits.

Caira sits on bench 1.


Inference:

Clearly we can see that in Case 2, we cannot place Caira at bench 1. Thus it
gets violated.

Thus we obtain the final arrangement through Case 1.

Answers:

91.

From the following explanation it is clear that Dia and Ela face each other.

Hence option D is correct.

92.

From the following explanation it is clear that Hina sits on Bench 1. Thus is
the odd one out because all other persons sit on becnh 2.

Hence option A is correct.

93.

From the following explanation it is clear that Ela- Caira is the pair of
persons who sit at extreme ends.

Hence option C is correct.

94.

From the following explanation it is clear that Fiza is second to the right of
Ahana.
Hence option B is correct.

95.

From the following explanation it is clear that Hina-Ahana are not facing
each other.

Hence option E is correct.


Set-20

Common explanation

Reference:

 People who like Red do not face each other.


 People liking Red sit at extreme ends of the rows.
 B likes red.
 People who like Red do not sit in the same row.
 E sits third to the right of B.

Inference:

Case(I): When B faces North

Color Red

Persons Row 2
facing South
Persons Row 1
B E
facing North

Color Red

Case(II): When B faces South

Color Red

Persons Row 1
E B
facing South
Persons Row 2
facing North
Color Red

Reference:

 People facing B and sitting next to him both like Black.


 No two people liking same color sit opposite or next to each other.

Inference:

With the help of second hint, position of colors can be determined in both
the cases.

Case (I): When B faces North

Color Black White Black White Red

Persons Row 2
facing South
Persons Row 1
B E
facing North

Color Red Black White Black White

Case (II): When B faces South

Color White Black White Black Red

Persons Row 1
E B
facing South
Persons Row 2
facing North
Color Red White Black White Black

Reference:

 R sits at one of the extreme ends.


 T is an immediate neighbour of R.
 Two people sit between Q and T.
 Q does not like red.

Inference:

Here, R cannot sit opposite to B, because this will violate the last hint in
both the cases.

Case (I): When B faces North

Color Black White Black White Red

Persons Row 2
Q T R
facing South
Persons Row 1
B E
facing North

Color Red Black White Black White

Case (II): When B faces South

Color White Black White Black Red

Persons Row 1 facing


E B
South
Persons Row 2
R T Q
facing North
Black
Color Red White Black White

Reference:

 A does not like black.


 S and A like same color.
 Person who likes red is not an immediate neighbour of D.
 D does not faces P.

Inference:

As A does not like black and, S and A like same color, thus A and S both like
White.

In this way both the cases satisfy the given hints and the final arrangement
looks like following.

Case(I): When B faces North

Color Black White Black White Red

Persons Row 2
Q S P T R
facing South
Persons Row 1
B C A E D
facing North

Color Red Black White Black White

Case (II): When B faces South


Color White Black White Black Red

Persons Row 1
D E A C B
facing South
Persons Row 2
R T P S Q
facing North

Color Red White Black White Black

Answers:

96.

As Derived in the Common explanation we can see that B, D, Q and R all of


them sit at an extreme ends but C does not.

Hence Option D is correct.

97.

As Derived in the Common explanation we clearly see that Q and D face the
people who like red.

Hence Option E is correct.

98.

As Derived in the Common explanation we clearly see that A sits exactly in


between C and E , S sits exactly in between P and Q Therefore, E sits exactly
in between A and D.

Hence Option B is correct.

99.

As Derived in the Common explanation we clearly see that all the given
options are false.
Hence Option D is correct.

100.

As derived in the Common explanation we can see that all the pairs face
opposite to each other except A and R.

Hence option C is correct.


Set-21

Common explanation

References:

Qadir sits at corner and facing away from the centre of the table.

The person sitting to the immediate right of Qadir is sitting at the corner of
the table.

Tarini, whose profession is Scientist, sits second to the left of Qadir and the
person whose profession is Lawyer.

Umesh, who is not the immediate neighbor of Qadir, is facing the person,
whose profession is Doctor.

Inferences:

From above statements,

By using above information, it is clearly observed that, no one sits to the


immediate right of Qadir.

Tarini, whose profession is Scientist, sits second to the left of Qadir and
also, Tarini sits second to the left of the one whose profession is Lawyer (as
per seating, Lawyer faces inside the table)

Umesh faces Doctor. Umesh is not an immediate neighbor of Qadir. Here


we get two possibilities as shown in figure.

By using above information, we get the initial seating as follows,

Case: 1 Case: 2
References:

Surya, whose profession is Teacher, is an immediate neighbor of the person


whose, profession is Lawyer.

Surya is not the immediate neighbor of Tarini.

Rithesh is not sitting on the edge of the table.

Inferences:

From above statements,

Rithesh sits corner of the table. By this case: 2 get eliminated as shown in
figure.
Case: 1

In Case: 2 there is no place for Rithesh at any corner. Hence this case is
eliminated.

Case: 2 [Eliminated]

References:

The immediate neighbors of Prakash are facing different directions. The


person sitting on the edges are facing towards each other.

The one whose profession is Scientist is not sitting to the immediate right of
the person whose profession is Banker.
The person whose profession is Banker does not sit at the edge of the table.

Inferences:

From above statements,

Prakash is a doctor. Prakash and Umesh (both are sitting on edges) are
facing each other. One of the immediate neighbors of Prakash is Qadir who
faces outside and other one is Tarini, who faces inside the table.

Therefore, Rithesh must face outside the table (Maximum 2 persons can sit
together who faces same direction)

The one whose profession is Scientist is not sitting to the immediate right
of the person whose profession is Banker.
Here, Rithesh is Engineer and Qadir is banker. Finally, Surya direction
cannot be determined, he face either inside or outside the table. Thus we
get the completed seating as shown,

Case: 1

Answers:

101.

The following Common explanation, we get "Immediate left of Prakash is


Qadir, who is a Banker".
Hence, option A is correct.

102.

The following Common explanation, we get "Rithesh-Engineer".

Hence, option C is correct.

103.

The following Common explanation, we get "Cannot be determined".

Surya faces either inside or outside and his immediate right is either Qadir
or Umesh.

Hence, option E is correct.

104.

The following Common explanation, we get "Prakash-Doctor".

Hence, option C is correct.

105.

The following Common explanation, we get "Rithesh, who is Engineer and


his immediate right is Tarini".

Hence, option B is correct.


Set-22

Common explanation

Reference:

Twelve persons are sitting in two parallel rows containing six persons each,
in such a way that there is an equal distance between adjacent persons. In
row-1, Joshi, Jaya, Jeet, Jamal, Juber and Jina are seated and all of them are
facing south. In row- 2, Prem, Pinu, Pal, Punj, Poran and Pooja are seated
and all of them are facing north. Therefore, in the given seating
arrangement each member seated in a row faces another member of the
other row.

Inference:

We will keep this information in mind while solving the puzzle.

Reference:

Joshi sits third to the left of Juber.

Neither Joshi nor Juber sits at extreme end of the line.

Inference:

After using the above hints we have:

Reference:

Prem sits second to the right of Poran.

Neither Prem nor Poran faces Juber or Joshi.


Prem does not sit at an extreme end.

Inference:

After using the given hints above figure can be redrawn as:

Reference:

Jeet does not face Prem and Jeet does not sit at an extreme end of the line.

Inference:

Here, we can fix the position of Jeet in the above figure as:

Reference:

Only one person sits between Pooja and Pal.

Neither Pooja nor Pal faces Juber.

Pal does not sit at the extreme end.

Inference:

After using the above hints we have:


Reference:

Only one person sits between Jaya and Jina.

Pooja is not an immediate neighbour of Pinu and Prem does not face Jina.

Inference:

After using the above hints we have:

At this point we can easily fix the position of Jamal and Punj in their
respective rows.

Answers:

106.

Following the final solution, we can say that two persons sit between Juber
and Joshi.

Hence, the correct answer is option C.


107.

Following the final solution, we can say that Poran is the one that does not
belong to the group because of the persons Poran is the only one who sits
at an extreme end.

Hence, the correct answer is option B.

108.

Following the final solution, we can say that Juber faces Pinu.

Hence, the correct answer is option D.

109.

Following the final solution, we can say that Jamal sits on the immediate
left of the one faces Punj.

Hence, the correct answer is option B.

110.

Following the final solution, we can say that none of the given statements is
correct.

Hence, the correct answer is option E.


Set-23

Common explanation

Reference:

1. Rachel who is reading Outlaw, is sitting at the third position to the right
of Henry and is reading one of the novels by same author.

2. Sofia is sitting at the second position to the right of rachel.

3. Henry is reading Fugitive and is sitting opposite to that of Hiram, who is


facing away from the center.

4. Both Paul and Henry are reading novels by same author.

Inference:

 Henry and Hiram are reading the novel by the same author.
 Paul and Henry are also facing away from the center.
 Direction and book read by Rachel is given to us, So we will begin with
her.

Reference:

1. Paul is reading Judgement and is sitting third position to the right of


Betty.

2. Betty is reading Shadow fell.


3. The person reading Legends is sitting at the second position to the left of
the person who is reading Shadow hunters.

Inference:

Now if we want to place Betty, there will be two Cases according to


mentioned conditions and since four of them are facing away from the
center, that makes paul fouth one who is facing away from the center.
Hence Betty is facing center of the table.

Case (I):

Case (II):

Reference:

1. John is sitting the second position to right of Betty and is reading


Legends.
Inference:

From reference we can say that case II will be eliminated because to the
second right of Betty in case 2 the place is already occupied.

Reference:

1.The person reading Legends is sitting at the second position to the left of
the person who is reading Shadow hunters.

2. Hiram is not reading Scarlet.

Inference:

Using the hints given above as reference, we get the final seating
arrangement as:

Answers:

111.

Following the Common explanation, it is clear that option C is correct.

112.

Following the Common explanation, it is clear that option D is correct.


113.

Following the Common explanation, and modifying it according to


condition in the question It is clear that option B.

114.

Following the Common explanation we can see that books written by same
author are read by Henry, Paul, Hiram and Rachel. Therefore, Andrew is the
one not reading the book by same author, making option E the correct
answer.

115.

Following the Common explanation we can see that Judgement novel is


read by the person who sits opposite to Andrew.

Hence option C is correct.


Set-24

Common explanation

References:

In row-1 Sriram, Tushar, Uma, Vivek, Wasim and Xanil are seated and all of
them are facing south direction.

In row-2, Mano, Nirmal, Oviya, Parthiv, Queen and Ranjith are seated and
all of them are facing north direction.

Tushar sits at an extreme end of the line.

Xanil is not an immediate neighbor of Tushar.

Tushar does not face the one who likes Vijay.

Only two people sit between Tushar and the one who likes Dhanush.

Nirmal sits second to the left of the one who faces the one who likes
Dhanush.

Nirmal does not like Ajith.

Tushar does not like Prabhu.

Inferences:

From above information,

• Tushar sits either extreme right end or left end of Row-1. So we get two
possibilities.

• Remaining all the above statements are directly given and can be filled
with respect to Tushar's position.
• Nirmal sits second to the left of the one who faces the one who likes
Dhanush. Here, Nirmal sits 2nd from left end in Row-2 [Case: 1] & Nirmal
sits extreme left end in Row-2 [Case: 2]

• Based on all above conditions, initial arrangement is as follows,

Case: 1
[If Tushar at extreme right end]
Row-1 (Person) ↓ Tushar Xanil
Actors Prabhu Dhanush
Actors Vijay Ajith
Row-1 (Person) ↑ Nirmal

Case: 2 [If Tushar at extreme left end]


Row-1 (Person) ↓ Xanil Tushar
Actors Dhanush Prabhu
Actors Ajith Vijay
Row-1 (Person) ↑ Nirmal

References:

No two persons, whose names start with consecutive letters of English


alphabet, are immediate neighbors of each other. For example, Binu is an
immediate neighbor of neither Aman nor Chitra and so on.

The one who likes Vijay sits second to the right of Oviya.

Inferences:

From above information,

• Nirmal & Oviya can’t be the neighbors (consecutive alphabetic name).


Therefore Oviya must sit in front of the one who likes Dhanush in both
cases as shown in seating.
Case: 1 [If Tushar at extreme right end]
Row-1 (Person) ↓ Tushar Xanil
Actors Prabhu Dhanush
Actors Vijay Ajith Vijay
Row-1 (Person) ↑ Nirmal Oviya

Case: 2 [If Tushar at extreme left end]


Row-1 (Person) ↓ Xanil Tushar
Actors Dhanush Prabhu
Actors Ajith Vijay Vijay
Row-1 (Person) ↑ Nirmal Oviya

References:

Only two persons sit between Parthiv and Ranjith.

Ranjith likes Karthi.

Parthiv sits second to the right of the one who likes Jeeva.

Inferences:

From above information,

• Parthiv and Oviya can’t be the neighbors (consecutive alphabetic names)


as per the condition.

• In Case-1, Parthiv sits at extreme right end and Ranjith sits on the
immediate left of Oviya. With respective to above 3 statements, Ranjith
likes Karthi, Oviya likes Jeeva and Parthiv likes Vijay.

Case: 1 [If Tushar at extreme right end]


Row-1 (Person) ↓ Tushar Xanil
Actors Prabhu Dhanush
Actors Vijay Ajith Karthi Jeeva Vijay
Row-1 (Person) ↑ Nirmal Ranjith Oviya Parthiv
• In Case-2, Parthiv sits at 2nd from right end and Ranjith sits on the
immediate left of Oviya. With respective to above 3 statements, Ranjith
likes Karthi, Oviya likes Jeeva and Parthiv likes Vijay.

Case: 2 [If Tushar at extreme left end]


Row-1 (Person) ↓ Xanil Tushar
Actors Dhanush Prabhu
Actors Ajith Karthi Jeeva Vijay Vijay
Row-1 (Person) ↑ Nirmal Ranjith Oviya Parthiv

References:

Vivek faces the one who likes Prasanth.

Only three people sit between the ones who like Prasanth and Rajni.

Inferences:

From above information,

• Vivek sits in the row-1 & faces south direction. Therefore the ones who
like Prasanth and Rajni come in row-2 & face north direction.

• In Case-1, Vivek must sit at 2nd from left end in row-1, by this the one
who like Rajni comes at extreme left end of Row-2 (Only possibility as per
condition)

Case: 1 [If Tushar at extreme right end]


Row-1 (Person) ↓ Tushar Xanil Vivek
Actors Prabhu Dhanush
Actors Rajni Ajith Karthi Jeeva Prasanth Vijay
Row-1 (Person) ↑ Nirmal Ranjith Oviya Parthiv

• In Case-2, There is no place for Prasanth and Rajni with 3 gaps in Row-2.
Hence this case can be eliminated as shown in seating.
Case: 2 [If Tushar at extreme left end] [Eliminated]
Row-1 (Person) ↓ Xanil Tushar
Actors Dhanush Prabhu
Actors Ajith Karthi Jeeva Vijay Vijay
Row-1 (Person) ↑ Nirmal Ranjith Oviya Parthiv

References:

One of the immediate neighbors of Mano faces the one who likes Arya.

Inferences:

From above information,

• Mano and Nirmal can’t be the neighbors (consecutive alphabetic names)


as per the condition.

• So Mano sits on the immediate right of Oviya and likes the actor Prasanth.
One of the immediate neighbors of Mano is Parthiv and Parthiv faces the
one who likes Arya as per given statement.

• Finally, Queen (last person in row-2) sits at extreme left end and likes
Rajni. Thus we get the following seating.

Case: 1 [If Tushar at extreme right end]


Row-1 (Person) ↓ Tushar Xanil Vivek
Actors Prabhu Dhanush Arya
Actors Rajni Ajith Karthi Jeeva Prasanth Vijay
Row-1 (Person) ↑ Queen Nirmal Ranjith Oviya Mano Parthiv

References:

Only three people sit between the one who like Vikram and Prabhu
respectively.

Uma does not sit at an extreme end of the line.


The one who likes Kamal sits to the immediate right of Sriram.

Inferences:

From above information,

• In Row-1, Tushar likes Vikram and Vivek likes Prabhu (Only possibility with
respect to the condition)

• Tushar and Uma & Uma and Vivek can’t be the neighbors (consecutive
alphabetic name) as per condition. Therefore Uma sits second to the left of
Tushar.

• Wasim and Vivek can’t be the neighbors (consecutive alphabetic name) as


per condition. Therefore Wasim sits on the immediate left of Tushar.

• The one who likes Kamal sits to the immediate right of Sriram. Here,
Sriram likes the actor Dhanush & Uma likes the actor Kamal.

• Finally, Xanil (last person in row-1) sits at extreme left end and likes the
actor Arya.

• Finally, Nirmal likes the actor Surya and Wasim likes the actor Ajith.
Nirmal & Wasim are opposite each other. Thus we get the completed
seating as shown.

Case: 1 [If Tushar at extreme right end]


Row-1 (Person) ↓ Tushar Wasim Uma Sriram Vivek Xanil
Actors Vikram Ajith Kamal Dhanush Prabhu Arya
Actors Rajni Surya Karthi Jeeva Prasanth Vijay
Row-1 (Person) ↑ Queen Nirmal Ranjith Oviya Mano Parthiv

Answers:

116.

Following Common explanation, we get "Wasim - Ajith".


Hence, option B is correct.

117.

Following Common explanation, we get 'Surya-Nirmal' is the odd one out


here.

Remaining 4 actors are liked by the person, who sits at any of the 4
extremes of two rows.

Hence, option C is correct.

118.

The following Common explanation, we get “Sriram-Dhanush".

Hence, option C is correct.

119.

The following Common explanation, we get "None of the above".

Hence, option E is correct.

120.

In the following Common explanation it is clear that Wasim sits immediate


right of the person who sits opposite to Ranjith.

Hence, option B is correct.


Set-25

Common explanation

Reference:

The total number of rows does not exceed 7 and the total number of seats
was five times the total number of rows.

The number of persons to the right of Hema who was at seat number 3 in
the row number 5 was 1 more than the total number of rows.

Tina was seated at the left most seat in the last row, which was not prime
numbered.

There were three seats between Saba and Tina.

Inference:

With reference to the first three hints it is clear that the total number of
rows was a composite more than 5 but less than 7.

So, the total number of rows will be 6 and total number of seats will be 30.

The number of persons to the right of Hema will be 7.

Seat number→ Total number


1 2 3 4 5 6 7 8 9 10
Row number↓ of seats
1
2
3
4
5 Hema 10
6 Tina Saba NA NA NA NA NA 5
Grand Total 30
Reference:

Mihir’s row number and seat number were same and it was an even prime
number.

Ranu sits second to the left of Shreya.

Shreya is on the immediate right of Sanu who sits in row number 4.

Total number of persons in the top 3 farthest rows (from screen) was equal
to 18.

Inference:

The only even prime number from 1 to 6 is 2, So, Mihir’s row number and
seat number will be 2.

The top 3 farthest rows are row numbers 4,5 and 6.

Sum of the seats in row number 5 and 6 is 15, thus there must be only 3
seats in row 4.

Total
Seat number→
1 2 3 4 5 6 7 8 9 10 number of
Row number↓
seats
1
2 Mihir
3
4 Ranu Sanu Shreya NA NA NA NA NA NA NA 3
5 Hema 10
6 Tina Saba NA NA NA NA NA 5
Grand Total 30
Reference:

Kavya was on the last seat of the first row.

The total number of persons in row 1 was thrice of the total number of
persons in row 4.

Shagun was seated in an odd numbered row except row 1.

Inference:

As total number of persons in row 4 are 3, so as per the second hint total
number of persons in row 1 would be 9 and Kavya sits at seat number 9.

The only odd numbered row from 1 to 3 except row 1 is row 3, thus Shagun
sits in row 3.

Thus we get all the 30 seats.

Total
Seat number→
1 2 3 4 5 6 7 8 9 10 number
Row number↓
of seats
1 Kavya NA 9
2 Mihir NA NA NA NA NA NA NA NA 2
3 Shagun NA NA NA NA NA NA NA NA NA 1
4 Ranu Sanu Shreya NA NA NA NA NA NA NA 3
5 Hema 10
6 Tina Saba NA NA NA NA NA 5
Grand Total 30

Answers:

121.

The total number of seats in the multiplex was 30.


Hence option C is correct.

122.

Seat number of Kavya is 9 and seat number of Sanu is 2.

Required difference = 7.

Hence option D is correct.

123.

Shagun sits immediately ahead of Ranu.

Hence option A is correct.

124.

Sanu is the one who sits on an even numbered seat among the given
options. Thus the odd one out.

Hence option E is correct.

125.

The total number of persons in the top two nearest rows (row number 1
and 2) is 11.

Hence option B is correct


Set-26

Common explanation

References:

At most 13 people sit around the circular table.

Distance between any two adjacent persons is same.

Akhil is facing Pravin, who is sitting second to the right of Tahir.

Inferences:

From above statements,

Note: If two people facing each other in circular seating means, equal
number of seats should be there on either side of that people. From
3rd reference point, we conclude that either 8 or 10 or 12 people maximum
sit around the circular table.

Let the position of people around the circular table are numbered as 1 to
8/10/12 in clock wise direction.

Keep this information in mind while solving this puzzle.

References:

Akhil is facing Pravin, who is sitting second to the right of Tahir.

Sriman is facing Lokesh, who is sitting fourth to the right of Chahal.

None among Sriman, Chahal and Lokesh is an immediate neighbor of


Pravin.
Inferences:

From above statements,


Given, Akhil is facing Pravin. Pravin sits second to the right of Tahir.

Given, Sriman is facing Lokesh. Lokesh sits fourth to the right of Chahal.

Case-1: If 8 people sit around the circular table

Tahir and Pravin sit at 1st and 7th position respectively. Akhil sits at
3rd position (reference point-1)

Then there is no place for Sriman, Lokesh and Chahal as per condition.
Hence this case become invalid and it can be eliminated.

Case-2: If 10 people sit around the circular table

Tahir and Pravin sit at 1st and 9th position respectively. Akhil sits at
4th position (reference point-1)

Lokesh, Chahal and Sriman sit at 2nd, 6th and 7th position respectively (only
possibility, reference point-2 & 3)

By using this information we get the following seating as shown,


Case-3 & 3-A: If 12 people sit around the circular table

Tahir and Pravin sit at 1st and 11th position respectively. Akhil sits at
5th position (reference point-1)

With respect 2nd and 3rd reference point we get two possibilities as shown,

Lokesh, Chahal and Sriman sit at 3rd, 7th and 9th position respectively
(1st possibility, Case-3)

Lokesh, Chahal and Sriman sit at 2nd, 6th and 8th position respectively
(2nd possibility, Case-3-A)

By using this information we get the following seating as shown,


References:

The person, who is sitting four places away from Tahir, is not immediate
neighbor of Chahal.

Manoj, who is sitting seven places away from Tahir, is not immediate
neighbor of Akhil.

Inferences:

From above statements,

Case-2: Here 10 people sit around the circular table

As per 1st reference point, people who sit at 5th and 7th position [4 places
away from Tahir on either side] are not the neighbor of Chahal. But in this
arrangement Chahal sit at 6th position i.e. Chahal is neighbor of both 5th and
7th position. Hence this case become invalid and it can be eliminated.

Case-3: Here 12 people sit around the circular table

1st reference point gets satisfied itself in 12 people seated circle.

Manoj is sitting seven places away from Tahir. It means there are six people
sit between Manoj and Tahir. Manoj is not the neighbor of Akhil.
Here, if Manoj is seated at a gap of 6 people to the right of Tahir, then
Manoj is the neighbor of Akhil which is no possible as per condition.

Therefore Manoj is seated at a gap of 6 people to the left of Tahir i.e.


Manoj sits at 8th position. All conditions get satisfied and we get the
completed seating as shown,

Case-3-A: Here 12 people sit around the circular table

In this case-3-A, if we count left of Tahir, Akhil sits 4 places away from Tahir
and Akhil is the immediate neighbor of Chahal which is not possible as per
1st reference point. Hence this case become invalid and it can be
eliminated.
Answers:

126.

Following the Common explanation, we get "None of these".

12 people are sitting around the table.

Hence, option E is correct.

127.

Following the Common explanation, we get "Two".

Two people are sitting between Manoj and Akhil, when counted from left
of Akhil

Hence, option B is correct.

128

Following the Common explanation, we get "Chahal sits to the immediate


right of Manoj".

Hence, option D is correct.

129.

Following the Common explanation, we get "Chahal".

Relation: Akhil sits 2nd to the right of Chahal

Tahir sits 2nd to the right of Lokesh


Similarly, Chahal sits 2nd to the right of Sriman

Hence, option A is correct.

130.

In the Following Common explanation it is clear that Manoj sits at even


numbered place.

Hence, option A is correct.


Set-27

Common explanation

Reference:

Hina’s husband sits third to the right of Bihu who is a male.

Bihu belongs to the third generation.

Bihu is not adjacent to any female.

Esha sits adjacent to her father Charu.

Hina sits second to the right of her daughter Esha.

Inference:

With the first hint it is clear that Hina is female.

With reference to the last two hints, we can conclude that Esha is daughter
of Hina and Charu both, since Hina is female, so Charu would be male.
Bihu belongs to third generation and is a male, that signifies that Bihu is the
son of Charu and Hina, because from the given relations, ‘son’ is the only
relation that denotes male from third generation.

Thus following seating arrangement and family tree can be observed.

Reference:
Geet sits immediate right of Hina.

Disha is the granddaughter of Anil but not the daughter of Charu.

Inference:

Thus Disha is the daughter of Hina’s brother i.e. Hina’s niece.

At this point we cannot determine the gender of Anil.

Reference:

Only one person sits between Farah, who is the son of Geet and Anil.

Hina’s mother sits to the immediate left of Disha.

Inference:

As no female can sit adjacent to Bihu, thus the only place left for Disha is
second to the left of Bihu.

Thus with reference to second hint, Geet is Hina’s mother and sits
immediate left of Disha.

Position of Farah and Anil can be fixed as immediate right and left of Bihu.
Answers:

131.

If Hina's brother (Farah) sits immediate right of Bihu then Hina's mother
(Geet) sits second to the left of Anil.

Hence option C is correct.

Final Arrangement and Family tree:

132.

Anil is the father-in-law of Charu.

Hence option B is correct.

133.

HIna is the mother of the person sitting opposite to her.

Hence option D is correct.

134.

In the Following Common explanation it is clear that Geet sits third to the
right of person who sits opposite to disha.
Hence, option C is correct.

135.

In the Following Common explanation it is clear that Hina sits Opposite to


Bihu.

Hence, option D is correct.


Set-28

Common explanation

Reference:

Below given is the distribution of soldiers as per descending order of their


ranks.

H(24) > C(68) > F(74) > B(31) > A(79) > D(20) > E(52) > G(82)

Inference:

While solving the puzzle we will keep this information in mind.

Reference:

The second highest ranked soldier was sitting opposite to the one who
killed the highest number of terrorists.

A is neither adjacent to the one whose rank is the lowest nor to the one
whose rank is the second highest.

Inference:

The second highest ranked soldier is C and the one who killed the highest
number of terrorists is G, thus, C and G sit opposite to each other.

As per the second hint we have only two positions left for A as shown in
Case 1 and Case2.
Reference:

The one who killed the fourth lowest number of terrorists was sitting
between G and the one who was third highest ranked.

F was immediate right of the soldier who was third lowest in rank.

Inference:

The one who killed fourth lowest number of terrorists is E and the one who
is third highest ranked is F.

So, E sits between G and F.

The third lowest ranked soldier is D.

Case 2 fails here, as there is no place for D to sit as per the second hint.

Reference:
D and the one whose number of terrorists killed is a prime number were
sitting opposite to each other.

Inference:

There are two terrorists whose number of terrorists killed is prime number
viz. B (31) and A (79).

But we have already positioned A as second to the left of C, thus B will be


seated opposite to D.

The only vacant place is occupied by H. Thus the final seating arrangement
looks like below.

Answers:

136.

E (the one whose rank is second lowest) was third to the left of the one
who killed second highest number of terrorists i.e. A.

Hence option B is correct.


137.

A-F is the odd one out as F is fourth to the right of A, whereas in all other
options the second person is third to the right of first person.

Hence option D is correct.

138.

The one who killed second lowest number of terrorists i.e. H sits opposite
to E.

Hence option A is correct.

139.

H sits adjacent to the one who killed lowest number of terrorists is false
statement.

Hence option B is correct.

140.

C and F are the immediate neighbours of the one who killed the lowest
number of terrorists i.e. D.

Hence option C is correct.


Set-29

Common explanation

Reference:

(b) The person from Banking and G are opposite each other. H is not from
Banking.

(d) Only one person sits to the left of G.

(e) H who is not from Medicine is the neighbour of the persons from Army
and Software.

(g) C sits to the right of the person from Army.

Inference:

Following two are the possible cases in which the above hints can be used.

Case 1 When G faces North


Field Banking
South facing
Person H
Person G H C
North facing
Field Army Medicine Software

Case 2 When G faces South


Field C H G
South facing
Person Software Medicine Army
Person H
North facing
Field Banking

Reference:

(a) B and the person from Teaching are adjacent to each other and one
among them sits at one of the ends.
(c) Either C or the person from Navy sits at one of the ends and both sit in
the different rows.

Inference:

Here we can see that in both the cases , C is sitting at an extreme end and
only one out of C and the one who works in Navy sits at one of the extreme
ends.

Thus the one related to Navy will sit in the middle of the row where people
are facing south because his/her row is different from that of C.

As we can see that there are no two adjacent vacant seats in the row facing
North, thus B and the person from teaching will sit in the other row in case
1 and vice versa in case 2.

Case 1 When G faces North


Field Banking Navy Teaching
South facing
Person H B
Person G H C
North facing
Field Army Medicine Software

Case 2 When G faces South


Field C H G
South facing
Person Software Medicine Army
Person B H
North facing
Field Teaching Navy Banking

Case 1A When G faces North


Field Teaching Banking Navy
South facing
Person HB
Person G H C
North facing
Field Army Medicine Software
Case 2A When G faces South
Field C H G
South facing
Person Software Medicine Army
Person HB
North facing
Field Navy Banking Teaching

Reference:

(f) The person from Business who is not D is adjacent to A and opposite E,
who faces north.

Inference:

Case 1 When G faces North


Field Business Banking Navy Teaching
South facing
Person A B D
Person G H C
North facing
Field E Army Medicine Software

Case 2 When G faces South


[Eliminated] E faces South
Field C H G E
South facing
Person Software Medicine Army
Person D B A
North facing
Field Teaching Navy Banking Business

Case 1A When G faces North


[Eliminated] No place for E to sit
Field Teaching Banking Navy
South facing
Person HB
Person G H C
North facing
Field Army Medicine Software
Case 2A When G faces South
[Eliminated] No place for E to sit
Field C H G
South facing
Person Software Medicine Army
Person B
North facing
Field Navy Banking Teaching

Hence the arrangement will be completed from Case 1 in the following


manner.

Case 1 When G faces North


Field Business Banking Navy Teaching
South facing
Person F A B D
Person Medicine G H C
North facing
Field E Army Agriculture Software

Answers:

141.

The following Common explanation, we get "Agriculture".

H is from the field of Agriculture.

Hence, option C is correct.

142.

The following Common explanation, we get "The person from Teaching, E".

The person from Teaching, E are at the ends of the row among the given
options.

Hence, option D is correct.

143.

The following Common explanation, we get "The person from Banking".

A, the person from Banking, sits opposite the person from Army.
Hence, option C is correct.

144.

In the following Common explanation we can say that A is from banking


field.

Hence, option C is correct.

145.

The following Common explanation, we get C sits opposite to D.

Hence, option D is correct.


Set-30

Common explanation

References

Only the person whose age is in even number is facing inside the circle.
The one whose age is 17 years old sits second to the left of Padmesh.
Two persons sit between Padmesh and Quincy.

Inferences

From above statements,

"The person whose age is in even number faces inside the circle and the
person whose age is in odd number faces outside the circle." We will use
this hint throughout the explanation.

Padmesh age was not known, thus either he faces inside or outside the
circle. Thus we get two possibilities.

The one whose age is 17 years old (odd number) faces outside & sits
second to the left of Padmesh.

Two persons sit between Padmesh and Quincy. Here we get two more
possibilities as shown in figure.
References

The one who sits immediate right of Quincy is 41 years old.

One of the immediate neighbors of Quincy is facing inside.

Udvita sits second to the right of Quincy.

Inferences

From above statements,

It is clearly understood that in Case: 1-A & 2, one of the immediate


neighbors of Quincy is 17 year old person and other one is 41 year old
person. Both are facing outside the circle (odd number ages). This violates
the given condition. Hence it can be eliminated.

But Case-1 & Case-2-A satisfy the given hints as shown in figure. The age of
Quincy is not known. Thus she faces either inside or outside the circle.

The one whose age is 41 years will face outside the circle (odd number).

One of the immediate neighbors of Quincy is facing inside. By using this


statement the direction of immediate neighbors of Quincy can be
determined as shown in figure.
Further two more cases namely- case: 1-B and case: 2-B arise.

References

Quincy’s age is neither 33 nor 19.

Wimal is an immediate neighbor of Quincy, but not an immediate neighbor


of Udvita.
Three persons sit between the one whose age is 33 years old and the one
whose age is 19 years.

Inferences

From above statements,

Three persons sit between the one whose age is 33 years old and the one
whose age is 19 years. Here we get only one possibility in each case i.e. the
person whose age are 33 and 19 sit opposite to each other and one of them
is Udvita.

Thus Udvita and the other person both face outside as both the ages are
odd numbers.

References

Yatin is 22 years old.


The one who sits second to the left of Udvita is 25 years old.

Inferences

From above statements,

As we know Yatin is 22 years old and there is no vacant place in the


arrangement in cases 1 and 2A.

Since, there is no place for Yatin to sit in both case 1 and case 2A, these got
eliminated.

In case: 1-B & Case: 2-B. Udvita's second left is Quincy. Thus, Quincy's age is
25 (Odd numbered age & Quincy faces outside in both cases).

The only vacant place is filled by Yatin in both the cases.

By using above statements, we get the following cases,


References

Padmesh is 7 years younger than Quincy.

Neither Yatin nor Wimal is an immediate neighbor of Zenith.

Rachna is 3 years younger than Wimal, but not the youngest person in the
group.

Two persons sit between the one who is the youngest and Udvita.

No two persons have same age.

Inferences

From above statements,


Quincy age = 25 (in both cases)

Given, Padmesh's age = Quincy – 7

Therefore, Padmesh's age = 25 – 7 = 18 years [Even numbered age]

Thus, Padmesh must face inside the circle. By this Case-2-B gets
eliminated.

Now only Case-1-B is left to continue.

Neither Yatin nor Wimal is an immediate neighbor of Zenith. Here, Zenith


sits exactly between Udvita and Quincy & Zenith age is 41 & he faces
outside the circle.

Rachna is 3 years younger than Wimal, but not the youngest person in the
group. Here, Rachna is not the youngest person i.e. (17 years) in the group.
Then the remaining person Xavier is the youngest person in the group.

Therefore, Rachna’s age is either 19 or 33.


No two persons have same age.

Given, Rachna = Wimal – 3. Let us check possibilities. If Rachna = 19, then


Wimal = 19 + 3 = 22

(Note: Yatin age = 22 & given, No two persons have same age.). Hence
Wimal is not 22 years old.

If Rachna = 33, then Wimal = 33 + 3 = 36 i.e. Wimal age is 36 & he is facing


inside the circle.

Therefore, Rachna's age is 33 (odd number) & faces outside the circle and
Udvita is 19 years old.

Thus we get the completed seating as shown below,

Answers:

146.

The following Common explanation, we get "Zenith-41 age-Eldest person of


the group”.

Hence, option B is correct.

147.

The following Common explanation, we get "Zenith-41 age & Xavier-17 age,
face outside the circle”.
Hence, option C is correct.

148.

The following Common explanation, we get "Three persons”.

Zenith-41 age & Xavier-17 age (both are opposite to each other).

Hence, option C is correct.

149.

The following Common explanation, we get "Wimal and the one whose age
is 33 are immediate neighbors”.

Remaining 4 options consist of persons and the age of persons sitting


opposite to them.

Hence, option C is correct.

150.

The following Common explanation, we get "The one who is 22 years old”.

2nd oldest person-Wimal (age 36 & faces inside initially).

If Wimal faces outside, his 3rd left is Yatin, whose age is 22 years old.

Hence, option D is correct.


Set-31

Common explanation

References:

1. Harish and Irfan, who likes Samsung, are diagonally opposite to each
other.

2. Diwan and the one who likes Samsung are facing each other.

3. Diwan likes Sony mobile phone.

4. The person, who is sitting third to the right of Diwan, is facing the person
or vacant seat who/which is immediate right of the one who likes Oppo.

Inferences:

From above information,

From point 1, 2, 3 we get Irfan likes Samsung phone and Diwan likes Sony
Phone & both are sitting opposite to each other [either north or south
direction]

Note: Irfan, Harish & Diwan all are sitting at extreme ends.

With reference to 4th point, Diwan can sit at either extreme left end of
Row-1 (South direction) or extreme left end of Row-2 (North direction), by
this we get two possibilities.

Case:1 Harish sits at extreme right end of row-1 and Irfan sits at extreme
right end of row-2 (diagonally opposite

Case: 1
Row – 1 (Person) ↓ Harish Diwan
Mobile Sony

Mobile Oppo Samsung


Row – 2 (Person) ↑ Irfan
Case: 1 Harish sits at extreme right end of row-2 and Irfan sits at extreme
right end of row-1 (diagonally opposite to each other) & Diwan sits
opposite to Irfan. By using above information we get the initial seating as
follows,

Case: 2
Row – 1 (Person) ↓ Irfan
Mobile Samsung Oppo

Mobile Sony
Row – 2 (Person) ↑ Diwan Harish

References:

1. Edwin is second to the right of the one who likes Sony.

2. One who likes Lenovo and the one who likes Moto are immediate
neighbors of each other.

3. One who likes Lenovo and one who likes Vivo are facing each other.

4. One, who likes Redmi, sits third to the right of the one who likes Lenovo.

Inferences:

From above information,

With reference to 4th point, in Case: 1, the one who likes Lenovo must in
Row-1 & faces south direction to satisfy the reference points 2 & 3. But we
get two possibilities as shown in seating arrangement. [i.e. Case-1 & Case-1-
A]

Case: 1
Row – 1 (Person) ↓ Harish Edwin Diwan
Mobile Redmi Moto/ Lenovo Sony

Mobile Oppo Vivo Samsung


Row – 2 (Person) ↑ Irfan
Case: 1-A
Row – 1 (Person) ↓ Harish Edwin Diwan
Mobile Redmi Moto/ Lenovo Moto/ Sony

Mobile Oppo Vivo Samsung


Row – 2 (Person) ↑ Irfan

With reference to 4th point, in Case: 2, the one who likes Lenovo must in
Row-2 & faces north direction to satisfy the reference points 2 & 3. But we
get two possibilities as shown in seating arrangement. [i.e. Case-2 & Case-2-
A]

Case: 2
Row – 1 (Person) ↓ Irfan
Mobile Samsung Vivo Oppo

Mobile Sony Lenovo Moto Redmi


Row – 2 (Person) ↑ Diwan Edwin Harish

Case: 2-A
Row – 1 (Person) ↓ Irfan
Mobile Samsung Vivo Oppo

Mobile Sony Moto/ Lenovo Moto/ Redmi


Row – 2 (Person) ↑ Diwan Edwin Harish

References:

Only one person sits between the one who likes Asus and the one who likes
Oppo.

Inferences:

From above information,

In Case-1, the one who likes Asus sits on the 2nd left of Irfan & there must
be one person sit between one who likes Asus and the one who likes Oppo
as per statement. But if we observe the case-1, 10 seats are occupied by
person or mobile phone which is not possible. Hence this case can be
eliminated.

Case: 1 [Eliminated] Total occupied seats exceed 9.


Row – 1
Harish Edwin Diwan
(Person) ↓
Mobile Redmi Moto Lenovo Sony

Mobile Oppo Asus Vivo Samsung


One person
Row – 2 should sit Irfan
(Person) ↑

In Case-1-A, the one who likes Asus sits on the immediate left of Irfan, by
this the seat which is on the immediate right of the one who likes Oppo
becomes Vacant as per given statement and it is shown in seating.

Case: 1-A
Row – 1 (Person) ↓ Harish Edwin Diwan
Mobile Redmi Moto/ Lenovo Moto/ Sony

Mobile Oppo Vivo Asus Samsung


Vacant
Row – 2 (Person) ↑ Irfan

In Case-2, the one who likes Asus sits on the 2nd left of Irfan & there must
be one person sit between one who likes Asus and the one who likes Oppo
as per statement. But if we observe the case-2, 10 seats are occupied by
person or mobile phone which is not possible. Hence this case can be
eliminated.

Case: 2 [Eliminated] Total occupied seats exceed 9.


Row – 1
Irfan One person
(Person) ↓
should sit
Mobile Samsung Vivo Asus Oppo

Mobile Sony Lenovo Moto Redmi


Row – 2
Diwan Edwin Harish
(Person) ↑
In Case-1, the one who likes Asus sits on the immediate left of Irfan, by this
the seat which is on the immediate right of the one who likes Oppo
becomes Vacant as per given statement and it is shown in seating.

Case: 2-A
Row – 1 (Person) ↓ Irfan
Vacant
Mobile Samsung Asus Vivo Oppo

Mobile Sony Moto/ Lenovo Moto/ Redmi


Row – 2 (Person) ↑ Diwan Edwin Harish

References:

1. The number of persons to the right of Edwin and the number of persons
to the left of Francis are same.

2. Ashwin, who likes Micromax, is not sitting adjacent to any of the vacant
seats.

3. Govind is an immediate neighbor of vacant seats.

4. Chandra sits right of Govind, who does not face south direction.

5. Bala likes neither Oppo nor Asus.

6. Vacant seats are not facing each other.

Inferences:

From above information,

With reference to 1st point, number of persons to the right of Edwin is 2.


Because there should be 3 vacant seats in the arrangement. Therefore
number of persons to the left of Francis must also be 2 in both case-1-A &
Case-1-B
If we observe the case-1-A & Case-1-B, 8 seats are occupied either with
person or mobile and the remaining one seat is filled by Ashwin, who likes
Micromax phone. From this we clearly understood that, Bala, Govind,
Chandra and Francis must like Oppo, Vivo, Asus and Moto but not
necessarily in the same order. I.e. No new seats can be occupied by these
4 persons.

Now up to the given condition, let check the possibilities:

Note: Chandra sits right of Govind and both are facing north direction.
Govind is an immediate neighbor of vacant seats.

Case: 1-A: To satisfy the reference point-1, Francis must like Asus; by this
number of persons to the right of Edwin is 2. Therefore number of persons
to the left of Francis is also 2. Then, Govind likes Oppo and Chandra likes
Vivo. Then Bala must like Moto (Bala doesn’t like Oppo or Asus).

With the last hint we know that two vacant seats can't face each other, so
the only left person Ashwin must face the vacant seat of row 2. So, the first
and second vacant seats are there in the row 2, but there should be a total
of 3 vacant seats as only 9 people are sitting, thus the third vacant seat is
beside Ashwin, which will violate the 2nd reference point.

Both the vacant seats should be adjacent to Govind, with reference to the
3rd reference point which says that Govind is an immediate neighbor of
vacant seats.

Case: 1-A When Bala is immediate left of Edwin


[Eliminated] No placefor Ashwin to sit
Row – 1 (Person) ↓ Harish Ashwin-X Edwin Bala Diwan
Vacant
Mobile Redmi Lenovo Moto Sony

Mobile Oppo Vivo Asus Samsung


Vacant Vacant
Row – 2 (Person) ↑ Govind Chandra Francis Irfan

Case: 1-B: To satisfy the reference point-1, Francis must like Asus; by this
number of persons to the right of Edwin is 2. Therefore number of persons
to the left of Francis is also 2. Then, Govind likes Oppo and Chandra likes
Vivo. Then Bala must like Moto (Bala doesn’t like Oppo or Asus).

Neither vacant seats are facing each other nor Ashwin is a neighbor of
vacant seat. All the conditions are satisfied.

Case: 1-B When Bala is immediate right


of Edwin
Row – 1 (Person) ↓ Harish Bala Edwin Ashwin Diwan
Vacant
Mobile Redmi Moto Lenovo Micromax Sony

Mobile Oppo Vivo Asus Samsung


Vacant Vacant
Row – 2 (Person) ↑ Govind Chandra Francis Irfan

If we observe the case-2-A & Case-2-B 8 seats are occupied either with
person or mobile and the remaining one seat is filled by Ashwin, who likes
Micromax phone. From this we clearly understood that, Bala, Govind,
Chandra and Francis must like Oppo, Vivo, Asus and Moto but not
necessarily in the same order. I.e. No new seats can be occupied by these
4 persons

Chandra sits right of Govind and both are facing north direction. Govind is
an immediate neighbor of vacant seats. But in both the cases only one seat
can be filled by either Chandra or Govind. If Chandra or Govind sit in a new
place in Row-2 (a seat which is not occupied by mobile phone), then total
seats exceed 9 which is not possible. Hence both the following cases can be
eliminated.

Case: 2-A [Eliminated]


Govind does not face south
Row – 1 (Person) ↓ Irfan Francis
Vacant
Mobile Samsung Asus Vivo Oppo

Mobile Sony Moto Lenovo Redmi


Row – 2 (Person) ↑ Diwan Edwin Harish
Case: 2-B [Eliminated]
Govind does not face south
Row – 1 (Person) ↓ Irfan
Vacant
Mobile Samsung Asus Vivo Oppo

Mobile Sony Lenovo Moto Redmi


Row – 2 (Person) ↑ Diwan Edwin Harish

Clearly, only case 1-B satisfies all the given hints, thus the final
arrangement will be obtained through Case 1-B.

Case: 1-B
Row – 1 (Person) ↓ Harish Bala Edwin Ashwin Diwan
Vacant Microm
Mobile Redmi Moto Lenovo Sony
ax

Samsun
Mobile Oppo Vivo Asus
Vacant Vacant g
Row – 2 (Person) ↑ Govind Chandra Francis Irfan

Answers:

151.

The following Common explanation, we get "Asus-Francis".

Hence, option A is correct.

152.

The following Common explanation, we get "Govind-Oppo".

Hence, option B is correct.

153.

The following Common explanation, we get "Only three persons".


Hence, option B is correct.

154.

The following Common explanation, we get "None of these, Vacant seat".

Second to the left of Francis is Govind & Govind faces vacant seat.

Hence, option E is correct.

155.

The following Common explanation, we get Irfan sits opposite to the


person who sits third to the left of Edwin.

Hence, option E is correct.


Set-32

Common explanations

Reference:

B lives on 4th floor.

G, lives on even prime numbered floor.

B, D, F and H are sitting in row-I facing North and A, C, E and G are sitting in
row-2 facing South (but not necessarily in the same order).

Inference:

The only even prime number from 1 to 8 is 2. So, G lives on second (2nd)
Floor.

We shall keep the last hint in mind while drawing both the arrangements.

Floor Person
8
7
6
5
4 B
3
2 G
1

Reference:

G sits opposite to F, who lives on 8th floor.

D, sits at the left end of the row and lives on the 6th floor.
The person who lives on 3rd floor sits between D and the person who lives
on 8th floor.

Inference:

Below floor and linear arrangement can be obtained from the above hints.

Floor Person
8 F
7
6 D
5
4 B
3 H
2 G
1

Since the person who lives on 3rd floor sits between D and F, thus B who
lives on 4th floor must sit at the right end of Row-I.

And the only person left in Row-I i.e. 'H lives on third floor.

Row- II
Facing South ↓

Row- I
D H F B
Facing North ↑

Reference:

G sits opposite to F.

E sits between A and G.

The person who lives on the first floor is not the neighbour of G in the row.

Inference:
It means E and C can't live on the first floor. Therefore, C would be living on
the 7th floor.

Now only two floors are left i.e. 1 and 5, as E also can't live on 1st floor , so
E must be living on 5th floor and A lives on 1st floor.

Thus both the arrangements get completed.

Floor Person
8 F
7 C
6 D
5 E
4 B
3 H
2 G
1 A

Row- II
A E G C
Facing South ↓

Row- I
D H F B
Facing North ↑

Answers:

156.

The following Common explanation, we get Clearly, A, C and B, D represent


the people sitting at extreme ends of both the rows.

Hence, option A is correct.

157.

The following Common explanation, we get Clearly, C sits to the immediate


left of the person who lives on 2nd floor.
Hence, option B is correct.

158.

The following Common explanation, we get C sits opposite to B

Hence, option C is correct.

159.

In the following Common explanation, it is clear that only three persons live
between C and H.

Hence, option D is correct.

160.

In the following Common explanation, it is clear that A and G are


immediate neighbours of the person who is sitting opposite to H.

Hence, option C is correct.


Set-33

Common explanation

References:

 Ishant and Rayudu were on the same sofa and there were two persons
between Pandya and Rayudu.

Inferences:

From the above reference, we can start with two cases as follows:

References:

 Rahul was to the immediate left of Pandya.


 Rahul was second to the right of Dhawan.

Inferences:

Using the above references, when we move ahead while placing the
mentioned players, we find that the second reference above violates the
case 1-A and the case 2-A.
Let's move forward with the remaining two cases.

References:

 Dhawan's sofa was neighboring Chahal’s sofa and Umesh’s sofa.


 Rahul and Umesh were on different sofas.
 Pandya is sixth to the right of Dinesh.
Inferences:

Here, 1st reference above clearly violates case 2-B as we can observe that
we can't place either Chahal or Umesh on the sofa to the right of Dhawan
as this sofa is already occupied by Pandya and Rahul. So, moving foward
with case 1-B only.

References:

Virat was third to the left of Umesh.

Umesh and Raina were seated at a gap of one person.

Dhoni was second to the right of Dinesh.

Inferences:

We get the final seating arrangement as:


Answers:

161.

Following Common explanation, we get "Rohit, Chahal sitting in different


sofas".

Remaining 4 pairs were sitting on the same sofa.

Hence, option C is correct.

162.

Following Common explanation, we get "Vijay".

Hence, option D is correct.

163.

The following Common explanation, we get "Raina and Rohit".

Hence, option B is correct.

164.

Following Common explanation, we get "Rohit".


Hence, option A is correct.

165.

Following Common explanation, we get "Six persons".

Hence, option A is correct.


Set-34

Common explanation

Reference:

Each of them was related to Preet in some manner: Mother, Father, Sister,
Wife and Brother. They were seated in a row, facing east. They were aged
differently. It is known that any child and his/her spouse would be younger
to his/her parents. Any husband would not necessarily be elder to his wife.

Inference:

We will keep this information in mind while solving the puzzle.

Reference:

Daya was seated at one of the extreme ends at a gap of 3 from Preet's
father.

Preet's father was to the immediate right of the one who was 30 years old.
The one who was 34 years old was neighboring Daya.

Inference:

Here, we have two possible scenarios in which we can use the above hints
accordingly.

Case 1:

Daya was at the extreme left end.

Case 2:
Daya was at the extreme right end.

Reference:

Preet was 3rd to the left of his sister.

Preet's wife was 32 years old.

Geet was 29 years old and was to the immediate left of Preet's brother.

Inference:

Using the above hints, we can say that Geet, who is 29 years old, cannot be
the Preet’s father as he cannot be younger than Preet’s wife, who is 32
years old.

Case 1:

At this point we cannot place Geet in the above scenario. So, we can say
that the case 1 is an invalid case.

Case 2:

Reference:

Roop was to the immediate left of Moni.


Inference:

With the help of above hints we can figure out the position of all the
persons in the row.

Case 2:

Reference:

Daya was 33 years elder to Moni.


Preet's father was 7 years younger than Preet's mother.

Inference:

Using the above hints we can say that Daya cannot be the Preet’s wife as
her age is more than 33 years and we know that Preet’s wife is 32 years old.
So, at this point Moni must be the Preet’s wife and Daya is Preet’s mother.

As we already figured out that the age of Preet’s wife i.e. Moni is 32 years.
So, the age of Daya will be 32 + 33 = 65 years.

Then, the age of Preet’s father will be 65 – 7 = 58 years.

Case 2:
Answers:

166.

Following the final solution we can say that Preet’s sister, who is 29 years
old is the youngest is the family.

Hence, the correct answer is option C.

167.

Following the final solution we can say that the ages of the persons sitting
at the extreme ends were 30 years and 65 years.

Required Difference = 65 – 30 = 35 years

Hence, the correct answer is option B.

168.

Following the final solution we can say that Preet’s wife was third to the left
of eldest person in the family i.e. Daya.

Hence, the correct answer is option D.

169.

Following the final solution we can say that Geet is sister of Preet.

Hence, the correct answer is option D.

170.
Following the final solution we can say that Preet sits second to left of Geet
if Geet changes her/his position with Moni.

Hence, the correct answer is option B.


Set-35

Common explanation

References

Vinoth doesn’t sit on the middle sides of the table.


Vinoth likes neither Black nor Orange.
Vinoth was born in January and is an immediate neighbor of Aravind and
Sachin.
Aravind was born in August month.

Inferences

From above statements,

o Vinoth sit at corner of the table & faces outside.

o Vinoth was born in January and is an immediate neighbor of Aravind and


Sachin. Here we get two possibilities as show in seating.

Based on the above statements we get the seating as shown below

Case: 1 Case: 2

References

Chinnu doesn’t face outside the table.

Chinnu likes Red and was born in June.


The one who was born in March sits on the immediate left of Chinnu.

Inferences

From above statements,

o Chinnu faces inside the circle. Thus we get two possibilities in each case.

o Based on the remaining statements, we get the following seating as


shown

Case: 1 Case: 1-A

Case: 2 Case: 2-A

References

Manju is an immediate neighbor of the one who was born in June and
Thara.
Manju was born on the month which has more than 30 days.

Manju does not like Black color.

Inferences

o Based on the above statements, we get the following seating as


shown

Case: 1 Case: 1-A

Case: 2 Case: 2-A

References

Udhay likes Yellow and was born in the month which has 30 days and is not
an immediate neighbor of Chinnu.

The one who was born in April sits opposite to the one who likes Pink.
The one who likes Pink was born in the month which has less than 30 days.
Inferences
From above statements,

o Udhay likes Yellow color & born in April month (30 days, only month
left among given)

o Udhay is not an immediate neighbor of Chinnu.

o The one who likes Pink color sits opposite to Udhay.

o The one who likes Pink was born in February (28 days i.e. < 30 days).
With this statement Case-1-A and Case-2-A gets eliminated as shown
in seating.

Case: 1

The one who likes Pink color sits opposite to Udhay.


The one who likes Pink was born in February
[Both statements are not satisfied in this case]
Case: 1-A [Eliminated] Case: 2
The one who likes Pink color sits opposite to Udhay.

The one who likes Pink was born in February

[Both statements are not satisfied in this case]

Case: 2-A [Eliminated]

References

Thara likes Blue and sits opposite to the one who likes Green.

The person who was born in the consecutive months will not sit together
i.e. The person born in January is not an immediate neighbor of the person
who was born in February.

Inferences

From above statements,

o Finally, Baskar born in February & likes Pink color in both cases.

o Case: 1 as per condition Thara or Sachin can’t born in May month (The
person who was born in the consecutive month will not sit together).
Therefore case: 1 gets eliminated as shown below.
Case: 1

o As per Case: 2 Sachin was born in May month & Thara was born in July
month

o Vinoth & Manju doesn’t like Black color. Then, Aravind likes Black color.

o Vinoth doesn’t like Orange color. Therefore Manju likes Orange color.

o Finally, Vinoth likes White color. Thus we get the case completed.

Case: 2

Answers:

171.

The following Common explanation, we get "Thara-July Month".

Hence, option B is correct.


172.

The following Common explanation, we get "The one who was born in June
and the one who was born in May".

Chinnu-June, Baskar-Pink & Sachin-May are immediate neighbors.

Hence, option D is correct.

173.

The following Common explanation, we get "Vinoth-White-Faces outside


the table".

Remaining 4 options are the ones who face inside the table.

Hence, option C is correct.

174.

The following Common explanation, we get "All of these".

All the options are true.

Hence, option E is correct.

175.

In the following Common explanation, it is clear that Thara and Chinnu are
the immediate neighbours of the person who sits third to the left of Sachin.

Hence, option B is correct.


Set-36

Common explanation

References:

Either Hendrik or the one whose age is 56 years is sitting at the extreme
end.

Bhairav sits to the immediate left of the one whose age is 56 years.

One person is sitting between Hendrik and the one whose age is 56 years.

Two persons are sitting between Chavez and the one whose age is 56 years.

The one whose age is 14 years is sitting second to the left of Chavez.

Inferences:

From above statements,

 Either Hendrik or the one whose age is 56 years is sitting at the extreme
end. If the one, whose age is 56, sit at extreme end, then he/she sits at
extreme right end.
 If Hendrik sit at extreme end, he/she sit either right or left end. If we
observe all above statements we generally get 3 initial possibilities,

Based on above information, we get 3 cases as shown in seating,

Case: 1
(If the one, whose age 56 sit at right end)
Persons
Chavez Hendrik Bhairav
(North)
Ages 14 56
Case: 2
If Hendrik at left end)
Persons
Hendrik Bhairav Chavez
(North)
Ages 56 14

Case: 3
(If Hendrik at right end)
Persons
Chavez Bhairav Hendrik
(North)
Ages 14 56

References:

There is a difference of 6 years between the ages of Faizan and Geetika and
two persons are sitting between them.

Inferences:

From above statements,

 To make the difference 6 only one possibility is there among given


number i.e. (8, 14)

 So, it is understood that, either Faizan's age is 14 & Geetika's age is 8


or Faizan's age is 8 & Geetika's age is 14.

 Two people sit between Faizan and Geetika.

With reference of above statements, let check the possibility in each cases.

Case: 1
(If the one, whose age 56 sit at right end)
[Eliminated] (No two gaps b/w Faizan/Geetika)
Persons
Faizan / Geetika Chavez Hendrik Bhairav
(North)
Ages 14 56
Case: 2
(If Hendrik at left end)
Persons Geetika /
Hendrik Bhairav Faizan / Geetika Chavez
(North) Faizan
Ages 56 14 8

Case: 3
(If Hendrik at right end)
Persons
Faizan / Geetika Chavez Geetika / Faizan Bhairav Hendrik
(North)
Ages 14 8 56

References:

There is one person sitting between Faizan and the one whose age is 44
years.

Aashvi is an immediate neighbor of the one whose age is 44 years.

Hendrik is older than Aashvi.

Inferences:

From above statements,

 In Case-2, Faizan's age is 14 and Geetika's age is 8. Here if Bhairav age is


44 and then Aashvi age become 56, who is oldest person in group. But
given, Hendrik is older than Aashvi. So this is not possible. If Chavez age
is 44, then Aashvi sits on the immediate left of Chavez. Thus given
condition is satisfied.

Case: 2
(If Hendrik at left end)
Persons
Hendrik Bhairav Faizan Aashvi Chavez Geetika
(North)
Ages 56 14 44 8
 In Case-2-A, Faizan's age is 8 and Geetika's age is 14. Then there is no
place for Aashvi, so this case can be eliminated.

Case: 2-A
(If Hendrik at left end)
[Eliminated] no place for Aashvi
Persons
Hendrik Bhairav Geetika Chavez Faizan
(North)
Ages 56 14 44 8

 In Case-3, Faizan's age is 14 and Geetika's age is 8. Here, Chavez age is


44, and then Aashvi sits on the immediate left of Chavez. Thus given
condition is satisfied.

Case: 3(If Hendrik at right end)


Persons
Faizan Aashvi Chavez Geetika Bhairav Hendrik
(North)
Ages 14 44 8 56

 In Case-3-A, Faizan's age is 8 and Geetika's age is 14. Then there is no


place for Aashvi, so this case can be eliminated.

Case: 3-A
(If Hendrik at right end)
[Eliminated] no place for Aashvi
Persons
Geetika Chavez Faizan Bhairav Hendrik
(North)
Ages 14 44 8 56

Note: We have only Case 2 & Case-3 left.

References:

Dhruvi is not sitting at the extreme end.

The one whose age is 12 is sitting second to the left of Dhruvi.


Eeshan and the one whose age is 21 years are immediate neighbors.

Hendrik is older than Aashvi.

Inferences:

From above statements,

 In Case-2, Dhruvi sits on the immediate right of Bhairav and Hendrik age
is 12. Now, Eeshan and the one whose age is 21 years are immediate
neighbors, by this condition this case gets eliminated as there are no
adjacent places. Also Last condition that Hendrik is older than Aashvi will
get violate.

Case: 2
(If Hendrik at left end)
[Eliminated] Hendrik is older than Aashvi- not satisfied
Persons
Hendrik Bhairav Dhruvi Faizan Aashvi Chavez Geetika
(North)
Ages 12 56 14 44 8

 In Case-3, Dhruvi sits on the immediate left of Hendrik and Bhairav's age
is 12. By this, Eeshan age is 56 and Dhruvi age is 21.

Finally, Hendrik is older than Aashvi. Therefore Hendrik's age is 36 and
Aashvi's age is 28 (All given conditions satisfied)

Thus we get the completed seating arrangement as shown below,

Case: 3
(If Hendrik at right end)
Persons
Faizan Aashvi Chavez Geetika Bhairav Eeshan Dhruvi Hendrik
(North)
Ages 14 28 44 8 12 56 21 36
Answers:

176.

The following Common explanation, we have "None".

Geetika = 8 & Bhairav = 12,

Difference = 12 – 8 = 4, both are immediate neighbors.

Hence, option C is correct.

177.

The following Common explanation, we have "7 years".

Faizan = 14 & Dhruvi = 21, Difference = 21 – 14 = 7 years

Hence, option B is correct.

178.

The following Common explanation, we have "Hendrik-36 years".

Hence, option C is correct.

179.

In the following Common explanation, it is clear that the name of persons


who sit at extreme end are Faizan and Hendrik and their ages are 14 years
and 36 years respectively.

Now, the sum of the ages,

14+36 = 50

Hence, option B is correct.


180.

In the following Common explanation Eashan is 56 years old.

Hence, option D is correct.


Set-37

Common explanation

References:

Tanay sits at the corner of the table.

Tanay sits third to the right of Rishab.

There are three persons sitting between Rishab and the one who was born
in February.

Tanay and the one who was born in August are immediate neighbors.

Inferences:

From above statements, we get two cases.

Case: 1

Case: 2

References:

One who was born in October and the one who was born in April are
immediate neighbors of Rishab.

One who was born in August is second to the right of Nikhil.


Liyan was born in May and sits second to the right of Tanay.

Jagat sits second to the left of Liyan.

Inferences:

By using these statements, we get

In case (1) Nikhil was born in February and facing inside the table

In case (2) Nikhil was born in February and facing outside the table.

So in both cases Nikhil’s second right is the person who was born in August.

Similarly, in case (1) Tanay face outside and Liyan (born in May) is seated
second to his right.

In case (1) Tanay face inside and Liyan (born in May) is seated second to his
right.

In Case (1) Liyan face centre and his second left is seated by Jagat (born in
October/April)

In Case (2) Liyan face outside and his second left is seated by Jagat (born in
October/April)

Remaining statements are placed as per the given information.

Case: 1
Case: 2

References:

Virak sits fourth to the right of Yash.

Virak is facing centre.

Virak and Yash are not facing each other.

One who was born in July sits third to the left of Virak.

Immediate neighbors of Rishab are facing direction opposite to Yash.

Inferences:

In case (1) Virak (given, facing centre) sits fourth to the right of Yash who
was born in August. Yash must face outside since Virak and Yash are not
facing each other.

In case (2) Virak (given, facing centre) sits fourth to the right of Yash who
was born in August. Yash must face outside since Virak and Yash are not
facing each other.

So Case (1) eliminated since No place is left to locate July, which is third to
the left of Virak.

Only Case (2) left to proceed. Tanay who was born in July sits third to left of
Virak.

Immediate neighbors of Rishab facing opposite direction of Yash.


Yash faces outside the table. Therefore the immediate neighbor of Rishab
faces inside the table.

Case: 1 [Eliminated]

No place is left to locate July, which is third to the left of Virak.

Thus we get, Case (2)

References:

Rishab was born in one of the months before the month in which Virak was
born.

Inferences:

From above statements, we conclude Rishab was born in January and Virak
was born in March.

No information on Harsha, so he/she may bear in October/April.

Thus we get the completed seating of case (2)


Answers:

181.

Following the Common explanation, we get "None of these". Third to the


left of Harsha is Virak.

Hence, option E is correct.

182.

Following the Common explanation, we get "Virak-March".

Hence, option C is correct.

183.

Following the Common explanation, we get "Nikhil-born in February, 28


days month".

Remaining persons are born in 31 days Month [January, May, July &
August]

Hence, option D is correct.

184.

Following the Common explanation, we get "Cannot be determined”.

Either Jagat or Harsha born in October. So we can’t find Nikhil’s position


with respect to October.
Hence, option E is correct.

185.

Following the Common explanation, we get Nikhil sits opposite to the


person who sits fifth to the right of Liyan

Hence, option C is correct.


Set-38

Common explanation

References:

Fazith sits third from the left end and likes RBI.

Vernish sits immediate left of the person who likes RBI.

Vernish likes Axis Bank.

Only one person sits to the left of Vernish.

No person sits between Aravind and Fazith.

Aravind sit second to the left of Ram.

Ram is not a neighbor of the vacant seat.

There is no vacant seat at the extreme ends.

Five people seated between two vacant seats.

Inferences:

From above statements,

o Some of the information is given directly and it can be placed first in the
seating as follows,
o No person sits between Aravind and Fazith. Here it may be they are
immediate neighbors or there is one vacant seat in between (two
possibilities)
o In Case (1), No vacant seat at extreme ends. Ram is not a neighbour of
the vacant seat. There is 5 people sit between two vacant seats. To
satisfy all this condition, then Case (1) is not possible. So it can be
eliminated.
Case: 1 [Eliminated]
Not satisfied: Ram is not a neighbor of the vacant seat. No
Person↑ vacant seat at extreme ends. There is 5 people sit between
two vacant seats.
Vernish Fazith Aravind Ram
Banks Axis
RBI
(North) Bank

o In Case (2), all above said conditions are satisfied and we get the
initial seating arrangement as follows,

Case: 2
Person↑
Vernish Fazith Aravind Ram
Vacant Vacant
Banks Axis Seat Seat
RBI
(North) Bank

References:

There is one vacant seat between Ragu and Nithya and no other person sit
between them.

Anbu sits third to the left of one of the vacant seats. Aruna is not a
neighbour of Anbu.

No person sits between Ram and Aruna.

Only one person sits between Aruna and one of the vacant seats.

Arivu sits second to the right of one of the vacant seats. Arivu does not like
BOB.

The person who likes BOB sits second to the right of one of the vacant seat.
The only neighbour of the person who likes Corporation Bank sits second to
the left of Atal.

Inferences:

From above statements,

o There is one vacant seat between Ragu and Nithya and no other
person sit between them. Therefore Nithya/Ragu sits 2nd or 4thfrom
the right end (only possibility).
o Anbu sits third to the left of one of the vacant seats. Here Anbu sits
at extreme left end (only possibility)
o No person sits between Ram and Aruna .Only one person sits
between Aruna and one of the vacant seats. [Here, we get two
possibilities as shown in Case 2 & Case 2-A]
o Arivu sits second to the right of one of the vacant seats. Arivu does
not like BOB. [Here, in case (2), we get only one possibility that is;
Arivu sits at extreme right end. But in case(2-A) we get one more
possibility as we don’t know the exact vacant seat and it is shown in
Case 2-B]
o The person who likes BOB sits second to the right of one of the
vacant seat. Here it is followed in each case shown.
o Now, remaining only person is Atal and he seated as per the place
left in each case.
o The only neighbour of the person who likes Corporation Bank sits
second to the left of Atal. [Based on this condition, Case 2-A & 2-B
are eliminated as shown in fig]

Note: Aravind who likes Corporation Bank and his only neighbour is
Aruna. Aruna sits second to the left of Atal. This condition not satisfied in
case 2-A and Case 2-B

Case: 2
Person↑ Nithya/ Nithya/
Anbu Vernish Fazith Aravind Aruna Ram Atal Arivu
Vacant Ragu Vacan Ragu
Banks Axis Seat Corporation Seat
RBI BOB BOB
(North) Bank Bank
Case: 2-A [Eliminated]
Not Satisfied: The only neighbour of the person who likes Corporation Bank sits
Person↑ second to the left of Atal.

Nithya/ Nithya/
Anbu Vernish Fazith Aravind Arivu Ram Aruna Atal
Vacant Ragu Vacant Ragu
Banks Axis Seat Seat
RBI BOB BOB
(North) Bank

Case: 2-B [Eliminated]

Not Satisfied: The only neighbour of the person who


Person↑ likes Corporation Bank sits second to the left of Atal.

Nithya/ Nithya/
Anbu Vernish Fazith Aravind Atal Ram Aruna Arivu
Vacant Ragu Vacant Ragu
Banks Axis Seat Seat
RBI BOB BOB
(North) Bank

Note: From above statements, Case (2-A) and Case (2-B) are eliminated.
Only Case (2) is left to continue.

References:

Ram does not like IOB. One of the immediate neighbors of Ram likes Dena
Bank.

The person who likes Dena Bank and IOB are immediate neighbors.

The person who likes LVB and Axis Bank are immediate neighbors.
The person who likes BOI and Canara Bank are immediate neighbors.

Nithya neither likes IOB nor BOI.

Inferences:

From above statements,

o Ram does not like IOB and Nithya does not like IOB. The person
who likes Dena Bank and IOB are immediate neighbors. One of the
immediate neighbors of Ram likes Dena Bank. Then Atal likes Dena
Bank and Ragu likes IOB.

o The person who likes LVB and Axis Bank is immediate


neighbors. Then Anbu likes LVB as per seating arrangement.
o Nithya does not like BOI and the person who likes BOI and Canara
Bank is immediate neighbors. Then Nithya likes Canara Bank and
Arivu likes BOI.

o Finally, Ram likes HDFC Bank. Thus we get the final table as follows,

Case: 2
Person↑
Anbu Vernish Fazith Aravind Aruna Ram Atal Nithya Arivu
Vacant Ragu Vacant
Banks Axis Seat Corporation HDFC Dena Seat Canara
LVB RBI BOB IOB BOI
(North) Bank Bank Bank Bank Bank

Answers:

186.

The following Common explanation, we get "Nithya-Canara Bank”.

Hence, option B is correct.

187.
The following Common explanation, we get " Vernish(Axis Bank) and his 3rd
right is Aravind who likes Corporation Bank”.

Hence, option D is correct.

188.

The following Common explanation, we get “Ram sits exactly between


Aravind (Corporation Bank) and Ragu (IOB)”.

Hence, option D is correct.

189.

The following Common explanation, we get "None” Arivu likes BOI sits at
extreme right end.

Hence, option A is correct.

190.

In the following Common explanation shown that Nithya likes Canara bank.

Hence, option D is correct.


Set-39

Common explanation

References:

Waha likes Freesia and sits either fourth from one of the ends.

Yaja likes Bergamot and sits at one of the extreme ends.

The one who likes Camellias sits exactly between Paya and Waha.

Paya is not an immediate neighbor of the one who likes Bergamot.

The one who likes Azalea and the one who likes Camellias are immediate
neighbors.

Only two persons sit between Baba and Daha.

Daha sits second to the right of the one who likes Camellias.

There are as many person sits between Paya and Yaja sits between Hara
and Taha.

Neither Hara nor Taha is an immediate neighbor of the one who like
Freesia.

Inference:

Based on the given hints following cases can be drawn.

Case 1 - When Yaja is at extreme left end.

Here, Paya can’t sit second or fourth to Waha’s right because then the 7th
and 8th hint which states that” Daha sits second to the right of the one who
likes Camellias” and “There are as many person sits between Paya and Yaja
sits between Hara and Taha” will contradict with each other. The position
of Paya will be sixth to the right of Waha.

Flowers Bergamot Freesia Azalea Camellias Azalea


Hara/ (Hara, (Hara, Hara/
Persons Yaja Waha Baba Daha Paya
Taha Taha)-X Taha)-X Taha

Case 1A

When Paya sits left of Waha- This case is Eliminated because here
positions of Baba and Paya coincide.

Flowers Bergamot Camellias Freesia


(Hara,Taha)- Daha
Persons Yaja Paya Waha
X (Hara,Taha)-X

Case 1B

When Paya sits right of Waha – This case is Eliminated because here also
positions of Baba and Waha coincide.

Flowers Bergamot Freesia Camellias


Persons Yaja Hara,Taha-X Waha Hara,Taha-X Paya Daha

Case 2 - When Yaja is at extreme right end.

Here Paya can’t sit second to the right of Freesia because hints that Daha
sits second to the right of Camellias” and “There are as many person sits
between Paya and Yaja sits between Hara and Taha.” will contradict with
each other.
Therefore Paya is second to the left of Waha in case 2.

Flowers Azalea Camellias Freesia Bergamot


Daha
Hara/ Hara, Hara/
Persons Paya Waha (Hara, Baba Yaja
Taha Taha-X Taha
Taha)-X
Case 2A

Here Paya can’t sit second to the left of Waha because then the condition
that only two persons are there between Daha and Baba will violate, as the
positions of Paya and Baba will coincide.

Neither Paya can sit fourth to the left of Waha because then the hint that
Daha sits second to the right of the one who likes Camellias will violate.
Positions of Waha and Daha will coincide.

Flowers Azalea Camellias Azalea Freesia Bergamot


Daha
Hara/ Hara, Hara/
Persons Paya Baba (Hara, Waha Yaja
Taha Taha-X Taha
Taha)-X

So we’ll continue with Case 1, 2 and 2A.

Reference:

There are as many persons sit between Hara and Waha sits between the
one who like Lilac and the one who like Bergamot.

Daha does not like Lilac.

Inference:

Case 1

If only 1 person is between Hara and Waha.

Flowers Bergamot Lilac Freesia Azalea Camellias Azalea Lilac - X


Persons Yaja Hara Waha Baba Daha Paya Taha

Case 2

Flowers Azalea Camellias Freesia Lilac - X Lilac Bergamot


Persons Hara Paya Waha Daha Baba Taha Yaja
Case 2A

Flowers Azalea Camellias Azalea Lilac - X Freesia Lilac Bergamot


Persons Taha Paya Baba Daha Waha Hara Yaja

Reference:

Gada and Edha are immediate neighbors .

Gada sits second to the left of Sada who likes Daisy.

The person(s) sitting between the one who likes Hyacinth and the one who
likes Crocus is same as the person(s) sitting between the one who likes
Crocus and Sada.

Edha likes Holly and is not an immediate neighbor of the one who like
Orchid or Begonia.

Inference:

Case 1

Eliminated as 2nd hint is not satisfied.

Flowers Bergamot Lilac Freesia Azalea Camellias Azalea Lilac - X


Persons Yaja Hara Waha Baba Daha Paya Taha

Case 2

It satisfies all the given hints in the following way.

Orchid/ Orchid/
Flowers Azalea Camellias Freesia Hycanith Holly Crocus Lilac Daisy Bergamot
Begonia Begonia
Persons Hara Paya Waha Daha Edha Gada Baba Sada Taha Yaja

Case 2A
Eliminated as 2nd hint is not fulfilled.

Flowers Azalea Camellias Azalea Lilac - X Freesia Lilac Bergamot


Persons Taha Paya Baba Daha Waha Hara Yaja

Thus the final arrangement can be obtained from case 2 by assigining the
only left position to Raga.

Answers:

191

Following the Common explanation, we get "Edha" Sixth from both ends.
Thus Edha is exactly in the middle.

Hence, option B is correct

192.

Following the Common explanation, we get "Cannot be determined".

Hara likes either Orchid/Begonia

Hence, option E is correct.

193.

Following the Common explanation, we get "Gada-Crocus & Baba-Lilac".

Hence, option A is correct.

194.

Following the Common explanation, we get "none of these".

Hence, option E is correct.


195.

Following the Common explanation, it is clearly shown in the table that


Gada sits between Edha and Baba.
Hence, option B is correct.
Set-40

Common explanation

References:

Different amounts to deposit are 4000, 6000, 15000, 1000, 2000, 3000,
9000 and 7000.

The one who has lowest amount to deposit sits opposite to the one who
has highest amount to deposit.

Qumila has the highest amount to deposit in the savings account.

Qumila deposited an amount which is sum of the amount deposited by


his/her immediate neighbors.

Only one person sits between Vitna and the one who has an amount 9000
to deposit.

The amount deposited by Vitna is not a multiple of 6.

Inferences:

Based on the given statements, we calculate the amount to be deposited at


bank by 8 persons.

With these statements, we get two possibilities,

Lowest Amount given = 1000 Then Qumila = 15000 Rs


Highest Amount given = 15000 sits opposite to the one
Qumila deposited highest amount who has 1000 Rs
We know Qumila = 15000 To get, Sum = 15000
(It is the sum of his/her immediate 6000 + 9000 = 15000
neighbors) (Only possible combination in given
amount)
So, the persons depositing 6000 &
9000
are the immediate neighbors of
Qumila
Case 1 Case 2

References:

The one who deposited thrice the amount of Vitna sits opposite to Vitna.

Pakija is an immediate neighbor of Neaha and Vitna.

Pakija is neither an immediate neighbor of the one who deposited 6000 nor
of the one who deposited 9000.

Inferences:

From above two cases, we see that Vitna sits opposite to Rs.6000

Given, thrice the amount (3 times) of Vitna sits opposite to Vitna. Only
amount to get 3 times is Rs.2000

So clearly, Vitna has Rs.2000 and sits opposite to the one who has (3 × 2000
= Rs.6000)

Based on remaining statements, we get

Case 1 Case 2
References:

There are as many person(s) sitting between the one who has the lowest
amount to deposit and Vitna as sitting between Neaha and Anab.

The one who has amount 7000 to deposit sits immediate left of Anab.

The sum of the amount deposited by Anab and Vitna is equal to the
amount deposited by Takbir, whose amount is multiple of 6.

The person Bayaz has only 3000 Rupees to deposit.

Inferences:

From both cases, Pakija has the lowest amount = Rs 1000

No one sits between Pakija (lowest amount) and Vitna. No one sits between
Neaha and Anab.

Thus Anab sits immediate right of Neaha (in Case 1) and Anab sits
immediate left of Neaha (case 2).

The one who has amount 7000 to deposit sits immediate left of Anab.
*Clearly, Anab doesn’t have Rs7000 to deposit in the account].

Finally, Anab + Vitna = Takbir (amount is multiple of 6)

Note: Vitna = Rs2000, Bayaz =3000 Rs Pakija =1000 Rs Qumila =15000 Rs

Remaining amounts are [6000, 9000, 4000, and 7000]

Anab= Rs 7000 (not possible), as one who has amount 7000 to deposit sits
immediate left of Anab.

If Anab =6000, Vitna =2000 Then Takbir = 6000+2000 = 8000 (not multiple
of 6, so not possible)
If Anab=9000, Vitna = 2000 Then Takbir = 9000 + 2000 =11000 (not multiple
of 6, so not possible)

If Anab= 4000, Vitna =2000 then Takbir =4000 + 2000 = 6000 (multiple of
6, which is possible)

Finally, Neaha sits opposite to 9000 (in both cases)

Then Neaha = Rs 7000 , Manov = Rs 9000 . Then we get the completed


seating,

Case 2
[Eliminated as the one who has
deposited 7000, sits to the immediate
Case 1
left of Anab.]

Thus final arrangement can be obtained from Case 1 in the following


manner:
Answers:

196.

Following the Common explanation, we get "None of these".

Pakija = 1000 Manov = 9000 Average = [1000+9000] / 2 = 10000 / 2 =


Rs.5000

Hence, option B is correct.

197.

Following the Common explanation, we get "Takbir-6000".

Hence, option A is correct.

198.

Following the Common explanation, we get "3rd left of 6000(Takbir) is


Pakija"

Hence, option B is correct.

199.

Following the Common explanation, we get "Neaha deposited Rs.7000".

Hence, option E is correct.

200.

Following the Common explanation, we get "2nd left of Neaha-Vitna (2000


Rs Deposited)"

Hence, option B is correct.


Set-41

Common explanation

Reference:

The people sitting at ends face in North direction.

Meet who is the wife of Kunj sits exactly in the middle of the row facing in
South direction.

Inference:

Using the given hints we can fix the position of meet in the middle of the
row.

Blood Chart A:

Reference:

Deep sits adjacent to Meet.

Geet’s spouse Deep faces in the North direction.

Deep who is Lata’s father sits fourth to the right of Prem who is the only
daughter of Roop and they both face in the opposite directions.

Inference:

Using the given hints we can fix the position of Deep and Prem ion the row.
Here, with the help of above hints following blood charts can also be
drawn:

Blood Chart B:

Blood Chart C:

Reference:

Deep’s neighbours are females and face in opposite directions.

Avi sits third to the right of Geet who is Prem’s grandmother and one of
them sits adjacent to
Meet.

Lata, who is male, is the only sibling of Kunj sits exactly between Avi and
Prem’s cousin who is a male.

Lata faces in same direction as that of Geet.

Inference:

At this point there are two possible scenarios in which we can fix the
positions of Geet and Avi in the row.
Case I:
Here, we cannot fix the position of Lata as per hint number 3 in case I
without contradicting the above hints. So, we can say that case I is an
invalid case.

Case II:

Here, we have one more hint that Geet is Prem’s grandmother at this point
we cannot use this information as doing this we will make our puzzle more
complex so we will keep this information in our mind and use it later.

Using the above hints we can merge our blood charts A and B as follows to
form chart D:

Blood Chart D:

Here, we will make a mental note of an information that Prem has a cousin
who is male.
Reference:

Avi is unmarried and faces in a direction similar to that of Kunj.


Roop who is the sister of Avi sits at one of the ends with a female.
Inference:

Using the above hints we can fix the positions of Kunj and Roop in the row
easily.
Case II:
After using the above hints we can merge our blood charts C and D as
follows to make chart E:

Blood Chart E:

At this point there is only place left in the row where we can fix the position
of Joy and after placing Joy at the extreme end left of the row we can say
that Joy is the cousin of Prem and the son of Kunj and Meet.

Case II:

Blood Chart E:

Here, both seating arrangement and blood chart is completed.

Answers:

201.

Following the final solution we can say that the one who sit at the extreme
right of the row i.e. Roop is the Daughter in law of Deep.
Hence, the correct answer is option E.

202.

Following the final solution we can say that no one sits third to the right of
Kunj.

Hence, the correct answer is option A.

203.

Following the final solution we can say that four females sit between Lata’s
spouse i.e. Roop and Prem’s cousin i.e. Joy.

Hence, the correct answer is option C.

204.

Following the final solution we can say that the number of males facing
south direction and the number of females facing north direction was 1 and
2 respectively.

Required Difference = 2 – 1 = 1

Hence, the correct answer is option B.

205.

Following the final solution we can say that Geet is the one who does not
belong to the group because, out of all the persons in the given options,
Geet is the only person who belongs to the first generation of the family.

Hence, the correct answer is option A.


Set-42

Common explanation

References:

Three persons sit between R and the one who likes Dravid, both of them
face each other.

R sits second to the left of V who likes Yuvraj.

Inferences:

From above statements,

R and the one who likes Dravid are sitting opposite to each other and both
are facing inside the circle.

R sits second to the left of V who likes Yuvraj. Here, V faces either inside or
outside the circle. Thus we get two possibilities as shown in initial seating.

Case: 1 Case: 2

References:

Two persons sit between the one who likes Dravid and U.

W sits third to the right of Q, who is an immediate neighbor of U.


W likes Ganguly.

Immediate neighbors of U face opposite to each other (If one facing the
center other facing away from the center or vice versa).

Inferences:

From above statements,

Two persons sit between the one who like Dravid and U. Also, Q and U are
immediate neighbors. W likes Ganguly sits third to the right of Q.

Note: R is one of the immediate neighbors of U in both cases and R faces


inside the circle. Therefore, Q is the other immediate neighbors of U must
face outside the circle as per given statement.

By using above information, we get the following case as shown below.

Case: 1 Case: 2

References:

T and P are immediate neighbors; one of them is an immediate neighbor of


V.

The ones who like Virat and Dhoni sit opposite to each other, both of them
are facing away from the center.
The one who likes Dhawan sits to the immediate right of the one who likes
Dhoni.

Inferences:

From above statements,

In case-1, there is no place for T and P (two places together and one of
them is an immediate neighbor of V). Therefore this case become invalid
and it can be eliminated.

In case-2, either T or P sits to the immediate right of V. With respect to


second reference point, either T or P likes Virat and U likes Dhoni & both of
them are facing away from the center (only possibility). Finally, R likes
Dhawan sits to the immediate right of U (likes Dhoni).

By using above information, we get the following case as shown below.

Case: 1 [Eliminated] Case: 2

References:

Not more than two persons facing same direction sit together.

U and Q do not like Sehwag.

Inferences:

From above statements,

Q does not like Sehwag and then Q likes Sachin.


Finally, S likes Sehwag and sits to the immediate left of Q. S faces inside the
circle i.e. Maximum 2 persons facing same direction sit together. Similarly,
W also faces inside the circle.

Note: Either T or P likes either Virat or Dravid. Thus we get the completed
seating as shown below.

Case: 2

Answers:

206.

Following the Common explanation, we get "Cannot be determined".

Either T or P likes Virat.

Hence, option E is correct.

207.

Following the Common explanation, we get "The one who likes Ganguly".

W (likes Ganguly) sits third to the right of Q (likes Sachin).

Hence, option A is correct.

208.

Following the Common explanation, we get "T".'


As per given condition, T likes Virat and P likes Dravid. T sits to the
immediate right of P.

Hence, option B is correct.

209.

The following Common explanation, we get "Two".

V (likes Yuvraj) sits 3rd to the right of S (likes Sehwag) i.e. only two persons
sit between V and S, if counted from the right of V.

Hence, option B is correct.

210.

Following the Common explanation, we get "None of the above".

All statements are false.

Hence, option E is correct.


Set-43

Common explanation

Reference:

Seven persons – Mani, Basu, Shiv, Teer, Jadu, Pari and Ramu, were sitting in
a straight line, facing towards north. These persons were born in different
years – 1975, 1982, 1986, 1989, 1991, 1996 and 1999, but not necessarily in
same order. There persons were born in different cities – Mandi, Jaipur,
Agra, Ajmer, Noida, Jhansi and Pune, but not necessarily in same order.

Inference:

We will keep this information in mind while solving the puzzle.

Reference:

Person who was born in 1986 in Ajmer was sitting on the extreme right end
of the row.

At most three persons were sitting on the left of Mani, who was born in a
leap year.

Three persons were sitting between Mani and Shiv, who was born in 1975.

Inference:

Here, there is only one leap year i.e. 1996 in all the given years.

At this point there are two possible scenarios in which the above hints can
be used.

Case 1:
Case 2:

Reference:

Only 1 person was born between person who was sitting on the immediate
left of Shiv and the person who was sitting in the middle of the row.

Inference:

At this point we cannot use the above hints in case 2 because the person
who was sitting on the immediate left of Shiv and the person who was
sitting in the middle of the row are same in case 2 so we can say that case 2
is an invalid case.

After using the above hints in case 1, we have:

Case 1-A:

Case 1-B:

Reference:

Jadu was born in 1999 and was born in Pune.


Pari wasn't born in 1991.

Pari, who was born after Teer, was sitting on the left of Teer, who was born
in Mandi.

Teer wasn’t sitting third from the left end of the row.

Inference:

Here, we can say that Pari wasn’t born in 1991; also Pari can’t be born in
1999 and 1986 under the given conditions. So, the possible birth years of
Pari are 1982 and 1989.

If Pari was born in 1982 then we have no possible birth year for Teer, as it is
given that Pari was born after Teer. So we can surely say that Pari wasn’t
born in 1982.

Now, if Pari was born in 1989 then only possible birth year for Teer is 1982,
as it is given that Pari was born after Teer. So we can surely say that Teer
was born in 1982 and Pari was born in 1989.

At this point we cannot fix the position of Pari and Teer in case 1-B under
the conditions so we can say that case 1-B is an invalid case.

Here, we have two possible scenarios in which above hints can be used in
case 1-A accordingly.

Case 1-A (i):

Case 1-A (ii):


Reference:

Two persons were born between the persons who were born in Noida and
Agra.

Basu, who was born in Jhansi, was sitting on the left of the person born in
Jaipur.

Shiv wasn’t born in Noida.

Inference:

Here, we cannot fix the position of Basu in Case 1-A (i) according to the
given hints so we can say that Case 1-A (i) is an invalid case.

After using the above hints, we have:

Case 1-A (ii):

Answers:

211.

Following the final solution we can say that the one born in 1989 was sitting
in the middle of the row.

Hence, the correct answer is option E.

212.

Following the final solution we can say that Shiv – 1975 – Agra is the correct
combination.
Hence, the correct answer is option D.

213.

Following the final solution we can say that one person was sitting between
Jadu and Teer.

Hence, the correct answer is option B.

214.

Following the final solution we can say that Basu was sitting on the extreme
left end of the row.

Hence, the correct answer is option B.

215.

Following the final solution we can say that the one born on 1982 second to
the left of Ramu.

Hence, the correct answer is option D.


Set-44

Common explanation

Reference:

Eight persons – Ankit, Ankush, Amjad, Akash, Akshay, Ashish, Ashad and
Arman, were sitting around a circular table, facing towards the centre.
These persons were playing cards and have won a different amount of
money.

Inference:

We will keep this information in mind while solving the puzzle.

Reference:

Every person won at least Rs. 20.

One of the persons won Rs. 20.

The person sitting opposite to Amjad won Rs. 60 which is half of the sum of
the amount won by the ones, who won the highest and the lowest amount
of money.

Inference:

Here, we can say that the least amount can be won by any of these persons
was Rs. 20. And, it is also given that one of the persons Rs. 20, it means Rs.
20 is the least amount won by these person.

We have,

⇒ (Highest amount + Least amount) = 2 × Rs. 60

⇒ (Highest amount + Rs. 20) = Rs. 120


∴ Highest amount won was Rs. 100

Here, we will make note that the highest amount won was Rs.

100 and least amount won was Rs. 20.

Reference:

Amjad was sitting neighboring Ankush and Ashish and won Rs. 40.
Ankush was sitting third to the right of the person, who won the highest
amount.

Ankit was sitting exactly opposite to the one, who won the highest amount.

Inference:

After using the above hints, we have:

Reference:

Akash won Rs. 80, and was not an immediate neighbour of the one who
won highest amount of money.
Ankush and his neighbours and the person sitting opposite to him did not
win the least amount of money.

The person, who was sitting opposite to the person neighboring Ankit, won
Rs. 30.

Inference:

After using the above hints, we have:

Reference:

One of the persons won Rs. 50.

The sum of the amount won by Ankit and his neighbors was Rs. 200.

Ankit won the less money than his neighbors.

Inference:

At this point we can say that either Ankush or Ankit won Rs. 50.

Let’s assume that one among Ankit and Ankush won Rs. X and other won
Rs. 50.

Now, Rs. (X + 50 + 80) = Rs. 200

∴ X = Rs. 70

After using the above hints, we can say that Ankush won Rs. 70 and Ankit
won Rs. 50.
Reference:

Sum of the money won by Ashad and Arman was more than the money
won by Ankush but less than the one who won the highest amount.

Arman won more money than only one person.

Inference:

After using the above hints, we have:

At this point we can easily fix the position of Akshay and can say that
Akshay won the highest amount.
Answers:

216.

Following the final solution, we can say that none of the persons were
sitting between the ones who won the highest amount and the least
amount, when counted from the left of the one who won the least amount.

Hence, the correct answer is option A.

217.

Following the final solution, we can say that the amount won by one, who
was second to the right of the one, who won the least amount is Rs. 70.

Hence, the correct answer is option C.

218.

Following the final solution, we can say that the amount won by Akash and
Ankush was Rs. 80 and Rs. 70 respectively.

Required Difference = Rs. (80 – 70) = Rs. 10

Here, we can see that none of the persons won Rs. 10.

Hence, the correct answer is option E.

219.

Following the final solution, we can say that Ankit was sitting third to the
right of the one who won the least amount.

Hence, the correct answer is option B.

220.

Following the final solution, we can say that Akshay won highest amount of
money.

Hence, the correct answer is option D.


Set-45

Common explanation

References:

Anvita is sitting second to the right of the one who likes Botany and the one
who is sitting at corner of the table.

There are three persons between the one who like Botany and Aadvika.

The one who likes Physics sits third to the right of Anvita. Anvita sits third to
the right of the one who like Physics subject.

Aadvika sits third to the right of Ashwika, who like Chemistry subject.

Aadvika likes History subject.

Inferences:

From above statements,

From the 1st reference point, we observe that, Anvita must sit at corner of
the table, by this all the above 5 reference points get placed as per the
given condition.
Thus we get the following initial seating as shown,

References:

1. The one who likes Zoology sits exactly opposite to the one who likes
Chemistry.
2. Artika sits exactly opposite to the one who likes Maths subject.

3. The one who like Hindi subject is sitting exactly opposite to the one who
is immediate left of the one who like English subject.

4. The number of people sits between the one who like English subject and
the one who like History subject count in anti-clockwise direction from the
one who like English subject is one more than the number of people sit
between the one who like Maths subject and the one who like Hindi subject
count in anti-clockwise direction from the one who like Hindi subject.

Inferences:

From above statements,

With reference to 1st and 2nd point we get two cases as shown in seating.

By using 3rd statement case-2 gets eliminated as shown in figure.

Before placing the hints based on 3rd point, just observe the 4th point i.e.

Number of people sits between Number of people sit between


the one who like English subject the one who like Maths subject
and the one who like History and the one who like Hindi
subject count in anti-clockwise subject count in anti-clockwise
direction from the one who like direction from the one who like
English subject Hindi subject
(4) (3)

Case-1: The above only possibility is obtained by placing, the one who like
Hindi is on the immediate right of Ashwika and the one who like English is
on the 2nd left of Ashwika.

By using all above statements, we get


Case: 1

Case-2: The one who like Hindi and the one like English can’t sit opposite to
each other as per statement (Refer Point-3). Hence this case can be
eliminated.

Case: 2 [Eliminated] No place for Hindi/English

References:

Anvita and Advita face opposite direction and Ankita sits third to the right
of Advita.

Anaika is facing inside the table and not an immediate neighbor of the one
who likes Hindi.

Inferences:

From above statements,

Anvita faces outside the table and then Advita faces inside the table.

Ankita (likes Botany) sits third to the right of Advita (likes Zoology).
Finally, Anaika likes English & Asvitha likes Physics subject.

Case: 1

Answers:

221.

Following the Common explanation, we get "Artika-Zoology, wrong pair".


i.e. Artika-Hindi (True).

Remaining 4 combinations are correctly paired.

Hence, option D is correct.

222.

Following the Common explanation, we get "Advita – Zoology".

Hence, option B is correct.

223.

The following Common explanation, we get "Ankita, Anaika and Anvita".

Hence, option D is correct.

224.

Following the Common explanation, we get "All are true statements".

Hence, option E is correct


Set-46

Common explanation

Reference:

Ganesh, who is a doctor, sits at the extreme end of the line and likes white
color.

Inference:

As, Ganesh sits at the extreme ends so there are two possibilities here:

Case 1

Case 2

Reference:

One who is a CA sits on the immediate left of the Doctor.

Inference:

As, CA is sitting on the immediate left of Doctor therefore case 1 fails here:

Case 2
Reference:

Avdhesh, who is a lawyer, sits third to the left of Faneesh. Neither Avdhesh
nor Faneesh sits at the extreme ends of the line.

Faneesh, who is a Banker, does not like Yellow color.

Baadal, who is a teacher likes Green color and sits on the immediate left of
Avdhesh. Avdhesh does not like either Blue or Yellow color.

Inference:

Now, two cases arise again while fulfilling conditions for Faneesh and
Avdhesh.

Case 1

Case 2

Reference:

Eshan, who likes pink color, is a businessman. Eshan is not an immediate


neighbor of either Avdhesh or Faneesh.

Inference:

As, there is no place left for Eshan to sit in case 2, so it fails here:
Reference:

One person is sitting between Harshit and Chaand, Harshit likes orange
color. Chaand, who is a pilot, likes Red color.

Inference:

As, Chaand is a pilot therefore Harshit is a CA.

Dinesh must be sitting between Avdhesh and Chaand and is an engineer.

Note: As, Avdhesh does not like Blue and Yellow color so he must be the
one who likes Black color.

Hence, Dinesh must be the one who likes Yellow color and Faneesh likes
Blue color.

Final Image
Answers:

225.

Avdhesh likes black color.

Hence option D is the correct answer.

226.

Dinesh is sitting between Chaand and Avdhesh.

Hence Option A is the correct answer.

227.

Blue color is liked by the one who is a Banker.

Hence Option C is the correct answer.

228.

Dinesh is an Engineer by profession.

Hence Option C is the correct answer.

229.

Harshit is sitting third to the right of one who likes yellow color

Hence Option A, is the correct answer.


Set-47

Common explanation

References:

X and Z are faces outside the circle.

One who sits immediate left of X likes Yamuna River and faces inside the
circle.

D sits second to the left of X.

Two persons sit between D and E.

E doesn’t sit adjacent to X and do not like Ganga River.

Both D and E are facing same direction.

E faces opposite direction to X.

Inferences:

From above statements,

Given, X faces outside the circle and E faces opposite direction to X.

Therefore, E faces inside the circle. Given, Both D and E are facing same
direction.

Thus, both D and E are facing inside the circle.

With respect to above reference points, we get the initial seating as


follows,
References:

C sits opposite to Y who sits to the immediate right of D and both are facing
each other.

The one who likes Saraswati River sits between E and Y.

The one who likes Ganga River doesn’t sit to the immediate right of E.

Inferences:

From above statements,

Given, C and Y are facing each other i.e. faces inside the circle. Y sits to the
immediate right of D.

With respect to above reference points, we get the following seating,

References:

X and Z are faces outside the circle.

Z sits second to the left of G.


Only two persons sit between G and H.

The one who likes Mahanadi River sits second to the right of the one who
likes Saraswati River.

The one who like Godavari River faces the one who like Mahanadi River.

Inferences:

From above statements,


Given, Z faces outside the circle. Here we get two possibilities.

Case 1: If Z sits to the immediate left of C, then G (likes Saraswati River) sits
to the immediate right of Y and faces outside the circle. Also, H sits to the
immediate left of X i.e. two persons sit between G and H.

Now as per statement, D likes Mahanadi River sits second to the right of G,
who likes Saraswati River. Finally, D faces Z as per seating. Here, Z likes
Godavari River and faces outside the circle which violates the given
statement i.e. one who like Godavari River and one who like Mahanadi
River is sitting opposite to each other and faces inside the circle. Hence this
case become invalid and it can be eliminated.

Case: 1 [Eliminated]

Case 2: If Z sits to the immediate right of Y and likes Saraswati River. Then,
G sits to the immediate left of C and faces inside the circle. Also, H sits to
the immediate left of X i.e. two persons sit between G and H.
Now as per statement, D likes Mahanadi River sits second to the right of Z,
who likes Saraswati River. Finally, D faces G as per seating. Here, D and G
are faces inside the circle i.e. one who like Godavari River and one who like
Mahanadi River is sitting opposite to each other and faces inside the circle.
All the given conditions get satisfied.
Case: 2

References:
The one who likes Narmada River sits immediate right of the one who likes
Kaveri River but not sitting opposite to the one who likes Saraswati River.
Inferences:
From above statements,
As per statement, the one who likes Narmada River and the one who likes
Saraswati River is not sitting opposite to each other. Therefore, C likes
Narmada River and X likes Kaveri River to satisfy the given condition. Now,
E doesn’t like Ganga River and then he/she likes Gomti River. Finally, Y likes
Ganga River and we get the completed seating as follows,

Case: 2
Answers:

230.

The following Common explanation, we get "Gomti River".

Hence, option D is correct.

231.

The following Common explanation, we get "C-Narmada River".

Hence, option B is correct.

232.

The following Common explanation, we get "Three".

The one who like Saraswati River (Z) and the one who like Kaveri River (X) is
sitting opposite to each other. 3 persons sit between them in either
direction.

Hence, option C is correct.

233.

The following Common explanation, we get "Y sits opposite to C who likes
Narmada River".

Hence, option A is correct.

234.

The following Common explanation, we get "None of these".

All statements are false.

Hence, option E is correct.


Set-48

Common explanation

References:

All the persons, who are not sitting at any end of the row, are not
equidistant from their immediate neighbors.

Distance between two consecutive persons is among 5m, 6m, 10m and
12m.

Mahesh, who is seated at one end of the row, is 15m away from Tahir.

Distance between Mahesh and its immediate neighbor is not 10m.

Inferences:

From above statements,

All the persons, who are not sitting at any end of the row, are not
equidistant from their immediate neighbors.

Here, the persons who are sitting in the row (except both ends) are not
equidistant from their immediate neighbors.

Mahesh, who is seated at one end of the row, is 15m away from Tahir.

Given, Distance between two consecutive persons is among 5m, 6m, 10m
and 12m.

It is clearly understood that, Mahesh and Tahir can’t be the immediate


neighbors as there is no one is seated with 15m between any two persons.

Therefore the possible combinations are (5m + 10m =15m). Therefore,


Mahesh is seated at one of the extreme ends and one person sits between
Mahesh and Tahir.

Distance between Mahesh and its immediate neighbor is not 10m.

Here, it is clearly understood that, Mahesh and its immediate neighbor


distance is 5m.

Note: Mahesh is seated either at extreme right or left end. Thus we get two
possibilities. Also we placed only three persons in the seating initially & we
will continue based on the other statements in following explanation.

By using above information, we get the initial seating as follows,

Case 1

Case 2

References:

Baskar is seated 23m to the left of Manoj.

Tarun is seated 27m to the left of Fiyaz.

The total length of the linear row is 50m.

Distance between two consecutive persons is among 5m, 6m, 10m and
12m.
Inferences:

From above statements,

Baskar is seated 23m to the left of Manoj.

Here also some persons seated between Baskar and Manoj. The possible
combinations for 23m among distance are

I. 6m + 5m + 12m = 23m &

II. 6m + 5m + 6m + 6m =23 m

Tarun is seated 27m to the left of Fiyaz.

Here also some persons seated between Tarun and Fiyaz. The possible
combinations for 27m among distance are

I. 10m +12m + 5m =27m

II. 5m + 5m + 6m + 6m +5m =27m

III. 5m + 5m + 12m + 5m=27m

IV 10 m + 6m +6m +5m =27 m

The total length of the linear row is 50m.

To make total length 50m with six gaps (for 7 persons) among given
distance (5m, 6m, 10m and 12m) we have only one possible combination as
follows,

5m + 5m + 6m +10m + 12m +12m = 50 m (Only possibility)

Note: From above combinations Only I and III are possible.

I.e. Baskar is seated 23m to the left of Manoj. Here two persons seated
between Baskar and Manoj. The distance between them is 6m + 5m + 12m
= 23m in any order.

Similarly Tarun is seated 27m to the left of Fiyaz. Here two persons seated
between Tarun and Fiyaz. The distance between them is 10m +12m + 5m
=27m in any order.

Now check the above combinations in seating arrangement,

Tarun is seated 27m to the left of Fiyaz [10m + 12m + 5m =27m in any
order]

Note: From above 50m length, only one 10m distance is there between any
two adjacent people. We know that, only 10m distance gap is there in
between Mahesh and Tahir.

Therefore, Tarun must be seated in between Mahesh and Tahir. Also, 2


persons seated between Tarun and Fiyaz. Fiyaz sits to the right of Tarun.
Thus case: 2 gets eliminated.

Thus we get the seating arrangement as follows,

Case 2 [Eliminated]

Case 1
Case 1-A

References:

Baskar is seated 23m to the left of Manoj.

Distance between Harish and the one who is seated at one end of the row
is 6m.

Distance between Fiyaz and Manoj, is not 22m.

Inferences:

From above statements,

Baskar is seated 23m to the left of Manoj

I.e. Baskar is seated 23m to the left of Manoj. Here two persons seated
between Baskar and Manoj. The distance between them is 6m + 5m + 12m
= 23m in any order.

From above seating (Case: 1 & 1-A), total 5m +10m +5m + 12m = 32m are
covered.

We know, 5m + 5m + 6m +10m + 12m +12m = 50 m (Only possibility)

Remaining distances left are, 6m + 12 m =18m to cover total 50m

With this possibility, we understood that Manoj should sit at extreme right
end. Based on this condition Case-1 gets eliminated as shown in figure.
Distance between Harish and the one who is seated at one end of the row
is 6m.

Here, it is understood that the distance between Manoj (sits at extreme


right end) and Harish is 6m.

Distance between Fiyaz and Manoj, is not 22m.

Here, the distance between Fiyaz and Manoj is 18m.

Case 1: Here there is no way to make distance between Manoj and Baskar
with 23m gap. We know only 18m left in the row i.e. Out of total length
50m; 32m are covered in this figure. Hence this case can be eliminated.

Case: 1

Case 1-A: Here Baskar sits third to the left of Manoj with 23m gap. All the
remaining conditions get satisfied & we get the completed seating.

Case 1-A:
Answers:

235.

Following the Common explanation, we get "Baskar, who sits 2nd to the left
of Harish".

Hence, option A is correct.

236.

Following the Common explanation, we get "Three persons".

Hence, option A is correct.

237.

Following the Common explanation, we get "50m".

Hence, option D is correct.

238.

The following Common explanation, we get "Harish".

Hence, option B is correct.

239.

Following the Common explanation, we get "12m".

Hence, option B is correct.


Set-49

Common explanation

Reference:

H sits 2nd to the right of the friend who likes basketball.

B is the neighbour of the one who likes basketball.

B likes hockey.

Inference:

For the position of B there could be 2 possible cases-

Case 1:

Reference:

The one who likes football sit between C and E.

E likes cricket and sits immediate right of F.

F don’t like football.

Inference:

For the position of E and C there are 3 possible sub cases:


Case 1.a:

Reference:

G likes kho kho and sits 2nd to the right of the one who likes volleyball.

The friend who likes table tennis sits opposite to G.

Reference:

A and D both don’t like neither badminton nor table tennis.

Inference:

Since A and D both don’t like badminton this case is invalid.

Case 1.b:
Reference:

G likes kho kho and sits 2nd to the right of the one who likes volleyball.

The friend who likes table tennis sits opposite to G.

Inference:

This case is invalid as position of G and the one who likes table tennis
cannot be fixed.

Case 1.c:

Reference:

G likes kho kho and sits 2nd to the right of the one who likes volleyball.

The friend who likes table tennis sits opposite to G.


Inference:

Position of G and the one who likes Football can be fixed.

Reference:

A and D both don’t like neither badminton nor table tennis.

Inference:

Since A and D both don’t like Table tennis this case is invalid.

Case 2:

Reference:

The one who likes football sit between C and E.

E likes cricket and sits immediate right of F.


F don’t like football.

Inference:

For the position of C and E there are 2 sub cases:

Case 2.a:

Reference:

G likes kho kho and sits 2nd to the right of the one who likes volleyball.

The friend who likes table tennis sits opposite to G.

Inference:

Position of G and the one who likes Football can be fixed.


Reference:

A and D both don’t like neither badminton nor table tennis.

Inference:

From the above condition A and D either likes volleyball or football and H
likes badminton.

Case 2.b:

Reference:

G likes kho kho and sits 2nd to the right of the one who likes volleyball.

The friend who likes table tennis sits opposite to G.


Inference:

From the above condition position of G and the one who likes volleyball
cannot be fixed. Hence this case is invalid.

Case 2.a is the final arrangement.

Answers:

240.

From the given solution H likes Badminton.

Hence the correct answer is option C.

241.

G sits between the one who likes Basketball and the one who likes Football.

Hence the correct answer is option A.

242.

From the given solution A likes either Volleyball or Football.

Hence the correct answer is option E.


243.

The following Common explanation, we get

Either A or D sits 2nd to the right of F.

Hence the correct answer is option E.

244.

The following Common explanation, we get

From the given solution 2 people sit between F and the one who likes
Football.

Hence the correct answer is option B.


Set-50

Common explanation
References

Therefore in the given seating arrangement each members seated in a row


faces another members of the other row.
In Row-1, all are facing south and in Row-2 all are facing north direction.

Two persons sit between the one who like Reuben and the one who like
Moose, who sits at extreme end.
Neel sits second to the left of Isaac, who sits opposite to the one who like
Reuben.
The one who like Almond sits immediate right of Isaac.
Inferences
From above statements,

 Note: Given, all the persons are facing each other, then vacant seats also
opposite to each other.
 Also, we don’t know who are all in each rows. So we have to solve based
on the possibility.
 The one who like Moose, who sits at extreme end. Here we get 4
possibilities

Based on the other statements, we get following initial seating as shown,

Case: 1
Row-1 (South)
Moose Reuben
Food
Persons

Persons Neel Isaac


Food
Almond
Row-2 (North)
Case: 1A
Row-1 Reuben Moose
(South)Food
Persons

Persons Neel Isaac


Food Almond
Row-2 (North)

Case: 2
Row-1 (South)
Almond
Food
Persons Isaac Neel

Persons
Food
Reuben Moose
Row-2 (North)

Case: 2-A
Row-1 (South)
Almond
Food
Persons Isaac Neel

Persons
Food
Moose Reuben
Row-2 (North)

Note: All the above cases are possible; only if two people are sit between
the one who like Reuben and the one who like Moose. If there is vacant
seat in between, then the position of the one like Reuben gets changed,
with that all the others position also gets changed.

References

The one who like Fish is sit at extreme end.


The one who like Fish and the one who like Bread sits diagonally opposite
to each other.

The one who like Corn sits second to the right of the one who like Fish.

The one who like Corn and the one who like Ostrich sits opposite to each
other.

Vacant seats are not in the extreme end.

Inferences

From above statements,

o By combining all above information, the following seating we


obtained are
o In Case (1), the one who like Fish is sit at extreme end.
v. If the one who like Fish is sit at extreme left end of Row-1, then the
one who like Corn sits second to the right of the one who like Fish is
not possible.
vi. If the one who like Fish is sit at extreme right end of Row-2, then the
one who like Corn sits second to the right of the one who like Fish is
not possible.
vii. If the one who like Fish is sit at extreme left end of Row-2, then the
one who like Corn sits second to the right of the one who like Fish is
possible. But all the 5 seats (In Row-2) from left are filled by this
statement. Then the vacant seat comes at extreme right end, which
is not possible. So Case-1 is not possible with respect to above
statements.

Note: All the above conditions explained (the one who like Fish, at end) is
applicable for all above 4 cases. It means that, we can’t continue all the
above 4 cases as it is. From, here we clearly observe that the vacant seat is
in between the one who like Reuben and the one who like Moose. So we
have to change all above 4 cases (1, 1-A, 2 & 2-A) with respect to the one
who like Reuben.
o In Case (1), the one who like Fish is sit at extreme left end of Row-2.
Then Neel likes Corn. The one who like Bread is sit at extreme left
end of Row-1 and the one who like Ostrich sits opposite to Neel.
[Note: Position of the one who like Reuben & others are changed
from initial solving]

Case: 1
Row-1 (South)
Moose Ostrich Reuben Bread
Food
Persons

Persons Neel Isaac


Food
Fish Corn Almond
Row-2 (North)

In Case (1-A), both possibilities as above explained are not possible. If we


not shifted the position of the one who like Reuben, then there is no place
for Ostrich & Bread as shown in table. If we shift the position of the one
who like Reuben, then there is no place for Neel. So this case is not possible
in either way, hence it can be eliminated.

Case: 1-A [Eliminated]


Row-1 (South) Reuben/ Moose/Br
Food Ostrich ead
Persons

Persons Neel Isaac


Food
Fish Corn Almond
Row-2 (North)

o In Case (2), the one who like Fish is sit at extreme left end of Row-1.
Then Neel likes Corn. The one who like Bread is sit at extreme left
end of Row-2 and the one who like Ostrich sits opposite to Neel.
[Note: Position of the one who like Reuben & others are changed
from initial solving ]
Case: 2
Row-1 (South)
Almond Corn Fish
Food
Persons Isaac Neel

Persons
Food
Bread Reuben Ostrich Moose
Row-2 (North)

o In Case (2-A), both possibilities as above explained are not possible. If


we not shifted the position of the one who like Reuben, then there is
no place for Ostrich & Bread as shown in table. If we shift the
position of the one who like Reuben, then there is no place for Neel.
So this case is not possible in either way, hence it can be eliminated.

Case: 2-A [Eliminated]


Row-1 (South)
Almond Corn Fish
Food
Persons Isaac Neel

Persons
Food Moose/ Reuben /
Row-2 (North) Bread Ostrich

Note: Out of 4 cases, two are eliminated and two (Case-1 & 2) are left to
continue.

References

Laksh sits second to the right of Row-1, vacant seat.

Jason sits third to the right of the one who like Toast.

The one who like Toast and the one who like Gumbo sits opposite to each
other.
Inferences

From above statements,

o In Case-1, the vacant seat in Row-1 is 3rd from the left end and vacant
seat in Row-2 is 3rd from the right end.
o Laksh sits on the immediate right of the one who like Ostrich.
o Jason sits third to the right of the one who like Toast. Here no place
for Jason as per the statement. Hence this can be eliminated.

Case: 1 [Eliminated]
Jason sits third to the right of the one who like Toast.
No place for Jason in either of the rows
Row-1 (South)
Moose Ostrich Reuben Bread
Food Vacant
Persons Laksh

Persons Neel Isaac


Food Vacant
Fish Corn Almond
Row-2 (North)

o In Case-2, we get two possibilities for vacant seats in both rows as


shown in table. The vacant seat in Row-1 is 4th from the left end and
vacant seat in Row-2 is 4th from the right end. Laksh likes Almond sit
extreme right end of Row-1. Jason sits third to the right of the one
who like Toast. Here no place for Jason as per the statement. Hence
this can be eliminated.

Case: 2[Eliminated]
Jason sits third to the right of the one who like Toast.
No place for Jason in either of the rows
Row-1 (South)
Almond Corn Fish
Food Vacant
Persons Laksh Isaac Neel

Persons
Food Vacant
Bread Reuben Ostrich Moose
Row-2 (North)
o In Case-2-B, the vacant seat in Row-1 is 2nd from the left end and
vacant seat in Row-2 is 2nd from the right end. Laksh sits on the
immediate right of Neel. Here Jason likes Moose sit at extreme right
end of the Row-2. Laksh like Gumbo and sits opposite to the one who
like Toast.

By using above all information, we get the following case-2-B as shown


below.

Case: 2-B
Row-1 (South)
Almond Gumbo Corn Fish
Food Vacant
Persons Isaac Laksh Neel

Persons Jason
Food Vacant
Bread Reuben Toast Ostrich Moose
Row-2 (North)

References

Samar sits second to the left of Kabir and does not like either Reuben or
Gumbo.

Only one person sits between Amol and Ranbir.

Ranbir sits opposite to the one who is an immediate neighbor of Farhan.

Inferences

From above statements

Samar likes Bread & sit at extreme left end of Row-2 and Kabir likes Toast
(Only possibility)

o Ranbir likes Reuben and Amol likes Ostrich (Only possibility with
3rd reference pint)
o Farhan likes Almond and his immediate neighbor is Isaac & Isaac
faces Ranbir. So the given condition satisfied.
o Finally, Isaac likes Hot Dogs and Darsh likes Fish. Thus we get
completed arrangement of all persons.

Case: 2-B
Row-1 (South)
Almond Hot Dogs Gumbo Corn Fish
Food Vacant
Persons Farhan Isaac Laksh Neel Darsh

Persons Samar Ranbir Kabir Amol Jason


Food Vacant
Bread Reuben Toast Ostrich Moose
Row-2 (North)

Answers:

245.

The following Common explanation, we get "Ranbir".

Amol like Ostrich and his/her 2nd left is Ranbir.

Hence, option B is correct.

246.

The following Common explanation, we get "One".

Laksh like Gumbo & Darsh (Opposite of Jason).

In between Laksh and Darsh, 1 person sitting & 1 vacant seat is there.

Hence, option A is correct.

247.

The following Common explanation, we get "Kabir".


Farhan like Almond & Opposite of Farhan is Samar. Kabir is 2nd to the right
of Samar

Hence, option D is correct.

248.

The following Common explanation, we get "Isaac, Darsh, Farhan" are


seated in Row-1

Hence, option E is correct.

249.

The following Common explanation, we get "Laksh and the one who like
Bread".

Explanation: Laksh & his Opposite person is Kabir. Kabir’s 2nd left is Samar.
Samar likes Bread *i.e Laksh and Opposite Person’s 2nd left is Paired+.

Remaining 4 options are paired,

Person and Opposite Person’s 2nd Right Example: Farhan opposite is


Samar. Samar’s 2nd right is Kabir. Kabir likes Toast.

Hence, option C is correct.

You might also like